面白い問題教えて

1 名前:132人目の素数さん 投稿日:2000/10/05(木) 18:25
頭の体操になるような問題・・お願いします

2 名前:名無しさん 投稿日:2000/10/05(木) 18:34
健作さんが愚息の正明君を連れて川のほとりまでハイキングに逝きました。
正明くんが健作さんに
「パパ、あの川は何メートルくらいあるの?」
すると健作さんは向こう岸を向いて帽子のつばを少し弄ってから
30メートル程歩き
「30メートルくらいかな。」
何故わかったのでしょうか?

3 名前:132人目の素数さん 投稿日:2000/10/05(木) 18:45
>>1
どんなジャンルのどのぐらいのレベルの問題がいいの?
例えば・・・

問題;6つの連続した整数がある。(最小の数をNとする)
これらを2つのグループに分ける時、
それぞれに入っている数の積が等しくなるように分割できるNの値を
全て求めよ。

4 名前:名無しさん@お腹いっぱい。 投稿日:2000/10/05(木) 19:03
>2
前日に調べておいたから


5 名前:132人目の素数さん 投稿日:2000/10/05(木) 19:21
>>2
実際に渡ったから

6 名前:名無しさん@お腹いっぱい。 投稿日:2000/10/05(木) 20:55
>30メートル程歩き
歩いてわかるくらいの感覚の持ち主だから
見た目でも大体わかる

7 名前:>2 投稿日:2000/10/05(木) 21:17
多分、自分の目から見た帽子のつばの位置を対岸に合わせて
そこから川のほとりの道に向いて対岸と同じ距離となる目標を定める。
そこから目標地点まで歩いて距離を計測した。というとこでは。
問題文は説明不足に思えますな。


8 名前:>2 投稿日:2000/10/05(木) 22:47
向こう岸に向かって30メートルほど歩いたら、ちょうど川を渡りきったから


9 名前:132人目の素数さん 投稿日:2000/10/05(木) 23:04
>>2
昔、健作さんはそこにダンボールで家を作って住んでいてその川のあたりは
目をつぶって歩けるし、食べ物(ゴミ箱)の場所も全て知ってるくらい
だからというのはどう?

10 名前:名無しさん@お腹いっぱい。 投稿日:2000/10/05(木) 23:49
>2
健作さんはアカシックレコードを読めるから



11 名前:えんざんしほう 投稿日:2000/10/06(金) 00:02
有名なのをひとつ。あまりに有名過ぎて皆知ってるよね。?
3x3に配置された格子点がある
. . .
. . .
. . .

てなかんじ。これを直線だけをつかった一筆書きで結んでちょうだい。

あきれるのは、各種セミナーみたいなもので、
”発想を変えろ”のモチーフとしてこれを持ち出す輩が今だにいること。
某F通のセミナでも、これがでてきた。こんなもん持ち出す発想が
余程陳腐だよ。

12 名前:132人目の素数さん 投稿日:2000/10/06(金) 00:11
>>11
その問題、知らないです。
条件は直線と一筆書きだけですか?
それだけだと考える必要がないと思うんですが…

13 名前:>12 投稿日:2000/10/06(金) 00:13
ごめんなさい、直線の本数を4本以下という制約がぬけてた。

14 名前:132人目の素数さん 投稿日:2000/10/06(金) 00:16
>12
折れ曲がるのは三回まで。つまり四本の直線を一筆で書く。

15 名前:132人目の素数さん 投稿日:2000/10/06(金) 00:17
格子点が目立たないじょ。

 ・ ・ ・
 ・ ・ ・
 ・ ・ ・

>これを直線だけをつかった一筆書きで結んでちょうだい。

なんか変な出題だな。ただ一筆書きで結ぶだけなら楽勝じゃない。
「折れ線☆本で結べ」じゃなかった?(☆はいくつか忘れた。。。)

16 名前:132人目の素数さん 投稿日:2000/10/06(金) 00:19
だぶった。許して。。。

17 名前:132人目の素数さん 投稿日:2000/10/06(金) 00:52
わからん
教えてくれ

18 名前:tr > 17さん 投稿日:2000/10/06(金) 02:03
その 9つの点は、数学で言うところの点とは違うのです。(涙)

19 名前:>18 投稿日:2000/10/06(金) 02:09
数学で言うところの点です

20 名前:tr > 19さん 投稿日:2000/10/06(金) 02:16
「点」 じゃないのは、折れ線 「三本」 で結ぶ場合でしたね。失礼しました。(汗)

21 名前:132人目の素数さん 投稿日:2000/10/06(金) 06:28
有名な問題で好きなの
「3枚のカードがある。
 一枚は両面赤、一枚は両面青、一枚は片面赤でもう片面が青。
 ここから一枚取り出したところ、表は赤でした。
 さてこのカードの裏面は赤か青か。賭けるとしたらどっちが特か」


22 名前:>21 投稿日:2000/10/06(金) 07:38
どちらかが特ってことはない。等確率。

疑いの余地のかけらもないと思うけど
何かひっかけがあるのかなあ。

23 名前:名無しさん 投稿日:2000/10/06(金) 07:44
赤の方が得。

RR,RB,BBのカードうち
RBのR面を引く確立は1/6
RRのR面を引く確立は1/3。

24 名前:>23 投稿日:2000/10/06(金) 07:51
なるほど。こういう錯覚に陥るのか。

25 名前:>17 投稿日:2000/10/06(金) 08:34
直線は別に格子点で囲まれた領域の外にはみでてもいい
がヒント

26 名前:132人目の素数さん 投稿日:2000/10/06(金) 12:58
あちこちで確率を「確立」と書く人を見受けるけど、
いい加減そんな間違いはよしてほしい。

27 名前:>26 投稿日:2000/10/06(金) 13:06
よくある誤変換の事例として
確立されてるわけですな?

28 名前:23 投稿日:2000/10/06(金) 17:51
>24
一見次の2つの事象は等確率で起こるように見えるけど
その前の事象の起こる確立との関連性から
等確率では無い。(ような気がする)

29 名前:132人目の素数さん 投稿日:2000/10/06(金) 19:19
↑わざとですよね?

30 名前:132人目の素数さん 投稿日:2000/10/06(金) 20:47
21の問題をアレンジ。

(n+2)枚のカードがある。
一枚は両面赤、n枚は両面青、一枚は片面赤でもう片面が青。
ここから一枚取り出したところ、表は赤でした。

(1)その取り出したカードの裏面が赤になる確率P(n)を求めよ。
(2)Lim[n->∞]P(n)を求めよ。

特に23さん、回答よろしくです。

31 名前:23 投稿日:2000/10/06(金) 23:00
>>21 >>23


>>28
俺の答えで正解だってば。
21さん、そうだよね?

32 名前:132人目の素数さん 投稿日:2000/10/06(金) 23:50
>28

コインをなげて10回連続で表が出ました
次に表が出る確率は?


33 名前:> 投稿日:2000/10/06(金) 23:53
1) カードをトランプみたいに向きをそろえて、シャッフルしてぬく
(シャッフルー>取り出すの過程で表裏の反転がない)
2) 箱みたいなものにほりこんで、かきまぜて、そこから取り出す
(シャッフルー>取り出すの過程で表裏反転は起こる)
で答えは違うよね。

34 名前:>32 投稿日:2000/10/06(金) 23:59
(ネタ)
>コインをなげて10回連続で表が出ました
>次に表が出る確率は?
これは細工のあるインチキコインだからきっと次も表!
確率は1



35 名前:23 投稿日:2000/10/07(土) 04:20
>32
落ち着いて考えるとモノの見事にハマってました。
恥ずかしいです。

36 名前:132人目の素数さん 投稿日:2000/10/07(土) 06:24
じゃんけんで、
ぐー  で勝つと、1万円、
ちょき で勝つと、2万円、
ぱー  で勝つと、5万円、もらえます。

なにを出しますか?

37 名前:132人目の素数さん 投稿日:2000/10/07(土) 06:52
>36
じゃんけんする相手は
>>36の条件を知っているのですか?

また、
ぐー  で負けると、5万円
ちょき で負けると、1万円
ぱー  で負けると、2万円、失う。

というようなリスクは負わなくていいんですか?



38 名前:36 投稿日:2000/10/07(土) 07:00
じゃんけんで、
ぐー  で勝つと、1万円、
ちょき で勝つと、2万円、
ぱー  で勝つと、5万円、もらえます。

なにを出しますか?

<追記>
一回勝負。
負けても、失いません。
相手は、この条件を知っています。

39 名前:おいおい 投稿日:2000/10/07(土) 07:41
21 の問題は、「赤の方が有利」が答えのはずだよ。この問題は、カードを引いて
表が青だったり赤だったりするような事象を問題にしてるんだよ。

つまり「表が赤である場合の裏が赤である条件付き確率」のはなしなの。

23 さんの言う通り、まあ、23さんの言い方は悪いんだけど、
「もう赤が出ている」ということが答えに影響を与えるの。

条件付き確率が難しいのは分かるけど、みんな大いに反省するように。

特に、30 は猛反省すべし。「表が赤が出ている時に、、、」なのだよ?

この問題について扱っている数学パズルの本は多いので、ちょっと本屋で調べれば
この問題の詳しい解説を知ることができるよ。近ごろは高校で条件付き確率
をやらないせいで、こんな間抜けな話になったのかな?

40 名前:39 投稿日:2000/10/07(土) 07:43
30 が、23 をからかうつもりで出したんなら、反省しないでよいです。
いや、それならば、むしろ、すばらしいと思うよ(^^

41 名前:132人目の素数さん 投稿日:2000/10/07(土) 07:50
三個の箱があって、その中の一つに私が1000円入れました。
あたりを引けば、1000円もらえるルールです。

あなた「じゃあ、これ」
わたし「だぶる〜〜〜ちゃ〜〜〜んす!」
あなた「な、なに??」

私は、残った二つの箱から、一つの箱を開けて、中身が入ってない
ことを見せるとその箱を捨てる。

わたし「さあ、この残った一つの箱と取り替えることが出来ます!!
どうしますか!?」

どうしますか?

42 名前:??? 投稿日:2000/10/07(土) 07:54
ひっかかった人、結構いたね。

さてさて赤が有利なのはいいとして
30が猛省しなきゃならない理由だけど
39を読んでもわからなかった。

もうちょい詳しくお願いします。

43 名前:>41 投稿日:2000/10/07(土) 07:58
最近旧鯖の既出ネタが多いな。それが悪いとは思わないけど
できればオリジナルの問題をドーンと出して欲しいものです。

44 名前:29>39 投稿日:2000/10/07(土) 08:32
反省居残り組は30、32、33か?

45 名前:>42 投稿日:2000/10/07(土) 08:43
>>23
>赤の方が得。
>RR,RB,BBのカードうち
>RBのR面を引く確立は1/6
>RRのR面を引く確立は1/3。

この説明がもっとも分かりやすいと思うけど・・・
(変換ミスは置いといて)
なんで>>23は屈してしまったんだ?23も居残りか?

46 名前:>37 投稿日:2000/10/07(土) 10:26
1) 相手が払う場合は
グー
 理由
  相手は負けても、被害が小さくてすむチョキを出す。
  と予想する。

    相手 G C P
  自分
   G   0  1  0
   C   0  0  2
   P   5  0  0


2)別に胴元がいてそいつが払う場合
  (相手はフトコロいためない、俺が賞金もらうことをとめる意思がない)
  賞金折半を相手に持ちかけて、八百長が仕込めるならパー
  仕込めない場合も、(相手のGCP選択が等確率なら)パー

 相手が意地悪な人で、自分のフトコロを痛めなくても、俺に賞金やりたく
 ないと考える人なら(1)のケースと同じで グーがいい。  

47 名前:132人目の素数さん 投稿日:2000/10/07(土) 11:12
遅レスですが…

>>2
帽子のつばが正面から左方45度の位置に向くよう、帽子をずらす。
そのあと対岸を向き、今の自分の真正面の位置にある、目印になるもの1つを決める。
対岸を向いたまま右方に歩き始め、帽子のつばの延長上に、先ほど決めた目印が到達
するまで歩き続ける。この間に歩いた距離が対岸までの距離とほぼ等しくなる。


48 名前:132人目の素数さん 投稿日:2000/10/07(土) 13:54
>45
BBのカードもいれる必要は無いと思うのですが
片面アカならRB、RRという事でその裏が青の確率と赤の確率は
同じではないでしょうか?


49 名前:> 投稿日:2000/10/07(土) 14:05
シャッフル 取り出しの過程がトランプみたいだったら
1)
上 R1 R2 B2
下 B1 R3 B3
から取り出した結果
上 R1 R2
 下 B1 R3
の可能性しかないから1/2に見えます

初期状態が1)であるとは限らない(上下逆の場合もある)
2)上 B1 R3 B3
下 R1 R2 B2
が初期状態のときは、
ここから 上赤で取り出せるのは
上 R3
下 R2
だけ。この場合は裏面は100%赤

1)と2)の初期状態について、知り得ず、同じように発生するなら
1)と2)の結果を合わせて、赤の方が有利

-------------------------------------------
ていうか素直に考えれば、
シャッフル取り出しの過程で表裏ひっくり
かえっていいでしょう

そうすると
赤上になるのは
上 R1 R2 R3
下 B1 R3 R2
の3通り

このうち2つが下も赤だから赤有利。



50 名前:>46 投稿日:2000/10/07(土) 14:10
(続き)
支払い元が別で、相手も勝った場合にちらと同じ条件で
賞金をもらえるという設定の場合。

これも、相手とは利害が対立しないので、
協力しあって--談合できるならそれが一番いい選択だが、
できない場合は、運まかせでパーをだそう。


51 名前:>49 続き 投稿日:2000/10/07(土) 14:27
つまり、カードの初期の向きが固定(赤青カードは赤が上)
されていて、取り出す過程でひっくり返らない
という特殊な条件のもとでは、
赤青等確率という答えになります。

もちろん
もとの問題にそんな条件はついてないから、
赤有利と答える方が一般的でしょう。


52 名前:132人目の素数さん 投稿日:2000/10/07(土) 16:44
>赤上になるのは
>上 R1 R2 R3
>下 B1 R3 R2
>の3通り

こうやって考えるのはおかしくありませんか?

上 R1 R2
下 B1 R3  の場合

上 R1 R3
下 B1 R2 の場合
にわけて考えるべきだと思います。なぜなら
>ここから一枚取り出したところ、表は赤でした
この時点からの確率だからです




53 名前:蛇側の"猿"Z子法 投稿日:2000/10/07(土) 17:59
>にわけて考えるべきだと思います。

わけるんなら全部をわけなきゃおかしい

ひいたカードが
上 R1
下 B1
の状態と
上 R2
下 R3
をひっつけたままの方がおかしい

カードをひいた結果は6通りある
1) 2) 3) 4) 5) 6)
上 R1 B1 R2 R3 B2 B3
下 B1 R1 R3 R2 B3 B2

この問題のポイントは、
3)と4)は、見た目は同じ(表裏両方赤)だが、
別の状態として数えなければいけない
という所にある。




54 名前:蛇側の"猿"Z子法 投稿日:2000/10/07(土) 18:01
う−ん ずれた
 1) 2) 3) 4) 5) 6)
上(表) R1 B1 R2 R3 B2 B3
下 (裏) B1 R1 R3 R2 B3 B2

です。


55 名前:蛇側の"猿"Z子法 投稿日:2000/10/07(土) 18:04
げ、まだずれる
番号 1) 2) 3) 4) 5) 6)
上表) R1 B1 R2 R3 B2 B3
下裏 B1 R1 R3 R2 B3 B2
これで大丈夫?

56 名前:>46,51 投稿日:2000/10/07(土) 18:14
賞金が、5,2,1万円は談合は成立しやすい
パー以外で勝っても高高2万円なので、
談合に応じて2.5万円もらった方が得
と相手は考えると思われます。

賞金が、5,3,1なら、
”応じる振りをして裏切ってチョキをだせば3万円GET”
という相手側の戦略があり得るので、またちがってくるかもしれない。



57 名前:132人目の素数さん 投稿日:2000/10/07(土) 19:03
熊が、ある地点から南へ1km進み、そこから東へ1km進み、
さらにそこから北へ1km進んだら、もとの場所に戻ってしまった……
さて、この熊の色は?」


58 名前:ツキノワグマ@北極 投稿日:2000/10/07(土) 19:55


59 名前:> 投稿日:2000/10/07(土) 19:57
もとの場所は、北極点?
よってシロクマと思われるので、白

ということかな?

シロクマは”尾も白い”ね。。。

60 名前:シロクマ 投稿日:2000/10/07(土) 20:11
身をもってユークリッド幾何学が成立しない世界が
あることを体験しました。

非ユークリッド幾何学はシロクマによって形成された。


61 名前:132人目の素数さん 投稿日:2000/10/07(土) 20:21
正解は色はない
誰も見ていないところの色がどうして存在するだろうか?


62 名前:即答できるのにしよう 投稿日:2000/10/07(土) 23:36
100以下の負でない偶数を全ての積をとるといくら?

63 名前:132人目の素数さん 投稿日:2000/10/07(土) 23:50
>>62
0だね。

64 名前:>60 投稿日:2000/10/08(日) 01:49
北極点が正解ならば、
「東へ進む」は「緯度に平行に進む」と解釈したことになる。
とすると南極点から2キロ以内にも(無限個の)解が存在する。


65 名前:即答できるのにしよう 投稿日:2000/10/08(日) 01:56
地球を完全な球とします。その半径をR[km]とします
Aさんは今、赤道上の東経135度の地点にいます。
そこから 真東に向かって6.25[km]歩きました。
Aさんがいる地点の緯度はいくらですか?
円周率は3.14で近似して、小数点第5桁まで求めてください。

66 名前:即答できるのにしよう 投稿日:2000/10/08(日) 01:57
(口頭でいうとひっかるやついるかも、知れないけど
 書くとひっかからんだろうな。。)

67 名前:即答できるのにしよう 投稿日:2000/10/08(日) 02:08
>62

類題
 絶対値1000未満の全ての奇数の総和は?



68 名前:132人目の素数さん 投稿日:2000/10/08(日) 02:16
>67
負の数含めれば0

69 名前:41 投稿日:2000/10/08(日) 04:54
>>43
すまそ。でもまあ、条件付き確率つながりと言うことで、勘弁して。

70 名前:21番の問題 投稿日:2000/10/08(日) 20:07
>3枚のカードがある。
>一枚は両面赤、一枚は両面青、一枚は片面赤でもう片面が青。
>ここから一枚取り出したところ、表は赤でした。
>さてこのカードの裏面は赤か青か。賭けるとしたらどっちが特か

これで赤に賭ける方が得というのがどうしても理解できないです。
既に表が赤であること知っている状態で裏がどちらの色か、
を賭けるのですから同じような気がします。

だから30さんの問題の答えは
(1)1/2
(2)1/2
だと思います。

誰か判りやすく説明して頂けないでしょうか。



71 名前:>65 投稿日:2000/10/09(月) 00:16
>円周率は3.14で近似して、小数点第5桁まで求めてください。
有効数字3桁でやめとけよ。

72 名前:>70 投稿日:2000/10/09(月) 08:55
A)青青
B)青赤
C)赤赤
この3枚から1枚引いて表裏どちらかを見るから
A表/A裏/B表/B裏/C表/C裏
の6通りの取り方があるね。

で、“1枚引いて一方が赤”の場合は
B裏/C表/C裏
の3通りだ。
だから“1枚引いて一方が赤”だった時、
引いたカードがBである確率は3分の1
引いたカードがCである確率は3分の2
このうち
“最初に見た面の裏が赤”なのは(Cを引いた時だから)3分の2
“最初に見た面の裏が青”なのは(Bを引いた時だから)3分の1

いかがでしょう?
(よく分からなかったら実際に試してみるのもいいでしょう)

73 名前:32人目の蛸さん 投稿日:2000/10/09(月) 13:11
)72
ほんとか〜


74 名前:即答できるのにしよう 投稿日:2000/10/09(月) 13:18
16チームでトーナメントをやると、
1回戦 8
2回戦 4
3回戦 2
決勝 1

で全部で15試合やることになります。
18チームだと、上の例の1回戦の前に2チームを振り落とす試合
2つをやれば、バランスのとれたトーナメントになりますね。
したがって試合数は17になります。

これをふまえて、
参加チーム数5001でトーナメントをやるときの試合数はいくつですか




75 名前:即答できるのにしよう 投稿日:2000/10/09(月) 13:19
(>74 これも、ひっかからんだろうな。。。。)


76 名前:>73 投稿日:2000/10/09(月) 13:24
どこが疑問だ?

77 名前:即答できるのにしよう 投稿日:2000/10/09(月) 13:31
>73
本当か?と思うんなら
72のいうように実験してみれば
なんとなくわかる。
120回ぐらい試行すれば、それなりの結果でると思う。


78 名前:132人目の素数さん 投稿日:2000/10/09(月) 13:36
>74=77
君の出してる問題は
“これが即答できたら理系”
の方がしっくりくると思うが。

79 名前:即答できるのにしよう 投稿日:2000/10/09(月) 18:44
>78
そう言われるとそうかも知れませんね。
今更、転載マルチポストってのも何だし、
今回はコッチで勘弁してください。

80 名前:132人目の素数さん 投稿日:2000/10/09(月) 19:21
>>21の問題(青赤カードのやつ)と似てる問題を(条件付き確率)
くだらねぇ問題はここへ書けスレ
http://cheese.2ch.net/test/read.cgi?bbs=math&key=967702991&st=373&to=373&nofirst=true
で書いたのですが、はっきりとした解答がもらえませんでした。
ボクなりの解答は
http://cheese.2ch.net/test/read.cgi?bbs=math&key=967702991&st=403&to=403&nofirst=true
に書いたのであってるか間違えてるかだけでも教えて下さい。
パッと見はカードの問題とは関係ないように思えますが、考えてみるととよく似た問題だと思います。

81 名前:32人目の蛸さん 投稿日:2000/10/09(月) 22:13
わかった、すまぬ。


82 名前:>80 投稿日:2000/10/09(月) 23:15
あれは、Cが死刑になる可能性は2/3のまま
つまり
80のいう
http://cheese.2ch.net/test/read.cgi?
bbs=math&key=967702991&st=403&to=403&nofirst=true
でOK てことになってんじゃなかった?



83 名前:132人目の素数さん 投稿日:2000/10/09(月) 23:50
11のがわからん 助けて

84 名前:>83 投稿日:2000/10/10(火) 00:21

ヒント  5つ点を増やしても同条件で解けます

 。  。
 ...
 ...
。...。
 。


85 名前:>83 投稿日:2000/10/10(火) 00:25
3×3の格子点を4本の直線の一筆書きで結べ、だよね。
>>11は出題が間違っているようです)
自分で答えを発見すると面白いからヒントだけ。
123
456
789
まず一本目は159を結ぶ。
残った点(234678)を三本の直線で結ぶ事を考えて下さい。

86 名前:84 投稿日:2000/10/10(火) 00:34
84は間違い。スマソ
1つよけいな点を増やしてしまった。

87 名前:132人目の素数さん 投稿日:2000/10/10(火) 00:57
\  |
|\/|
/ \|

斜め右下1
上2
斜め左下3
上4


88 名前:87 投稿日:2000/10/10(火) 00:59
159635741

89 名前:132人目の素数さん 投稿日:2000/10/10(火) 00:59
傘。

90 名前:>88 投稿日:2000/10/10(火) 02:06
2と8は?

91 名前:41 投稿日:2000/10/10(火) 04:12
>>80
その問題の構造は、41 の問題と全く同じなんだよん。

死刑じゃないのを「あたり」として考えてみ?

3本くじがあって、君が一本くじを引いて、残り2本を袋に入れたとするね。
で、その袋から一本はずれくじを取り出したとしよう。
さあ、その袋と、君が持ってるくじとどちらが当たる確率が大きいかな?

くじが100本で、99本入れて98本のはずれを抜くとかなると、
気がつくはずなんだけど、ここで「全部が3」という微妙な数字が
生きてくるのだねえ。

92 名前:>89 投稿日:2000/10/10(火) 05:37
うまい。まさに。

93 名前:132人目の素数さん 投稿日:2000/10/10(火) 08:18
>>91
なるほど、単純化すればそういうことになるのか。
死刑囚Cのいっている
C:「ボクは明日3人のうち2人が死刑になることは知っている。
  つまり、A、Bのうちどちらかは確実に死刑になることは知っているんだ。
知っていることだから話しても教えたことにはならないよ」
って理屈はCの死刑になる確率でいえばまったくその通りなんですね。
でもこの問題の解答として「Cが最初言ってた事はは正しいよ」って答えじゃ
納得できない人が多いと思います…
それに構造が「全く」同じとも思えないです。
41の問題の答えは「交換した方が得」ですね、交換すれば2/3の確率で1000円
もらえる。

94 名前:93 投稿日:2000/10/10(火) 08:32
>>91
あ すいません…
袋ってのを使って解いてくれてたんですね。
「Cが最初言ってた事はは正しいよ」ってことじゃなかったのか…
うーヒントや解法も理解するのが難しい…

95 名前:>89 投稿日:2000/10/10(火) 10:23
三本の時は


(格子点が面積をもってって
 123を斜めに結ぶ)


96 名前:132人目の素数さん 投稿日:2000/10/10(火) 11:27
昔の大数の数学鼎談(多分)に載ってた問題。
問題自体より、結論が興味深い問題。

百発百中の大砲一つと、百発一中の大砲百個、
1ターンにすべての大砲が敵に発射する。
大砲は当たれば必ず破壊できるものとすると、
どっちが有利だろうか。また互角になるのはどんなときだろうか

97 名前:132人目の素数さん 投稿日:2000/10/10(火) 12:14
>96
ランチェスター戦略ですね。

98 名前:>96 投稿日:2000/10/10(火) 12:42
敵の砲弾がヒットしても、大砲そのものは、無傷って前提でいい?


99 名前:>96 投稿日:2000/10/10(火) 13:21
> また互角になるのはどんなときだろうか
というのは、何を変数と見るのですか?

100 名前:>99 投稿日:2000/10/10(火) 13:35
A軍:一発必中の大砲一個
B軍:確率αで当る大砲n個

これで互角になるαとnの関係式を出せばいいんじゃない?

101 名前:132人目の素数さん 投稿日:2000/10/10(火) 14:17
一発以上あたるまでのターン数の期待値、で比較していいんですかね?

102 名前:> 投稿日:2000/10/10(火) 15:48
>98
それとも
数撃てば当たる軍は、1ターン毎に1門失い、
全部失う100ターンまでに1発ヒットすれば
勝ちってルール

103 名前:132人目の素数さん 投稿日:2000/10/10(火) 17:00
大砲を狙うの?


104 名前:96 投稿日:2000/10/10(火) 18:10
問題文が曖昧だ・・・う!津田氏悩・・・

前半は
>>102という感じで。
後半は、大砲の数を変数にして。拡張してもいいけどね。
もちろん大砲は敵の大砲を狙うし、当たった大砲はもう戦闘不能ということで

あと、
敵軍の大砲をすべて破壊したときを勝利とするとき、
百発百中軍が勝利する確率はどれほどだろうか。

105 名前:132人目の素数さん 投稿日:2000/10/10(火) 20:38
>百発百中軍が勝利する確率はどれほどだろうか。
つまり100回敵軍が外すと言う事でしょ?
計算メンドそう・・・



106 名前:132人目の素数さん 投稿日:2000/10/10(火) 21:39
>つまり100回敵軍が外すと言う事でしょ?
>計算メンドそう・・・

試してみた?

>百発百中の大砲一つと、百発一中の大砲百個、

この設定で百個の方がハズシ続ける確率を計算しましょう。
1ターンごとに確実に1個撃破されていき、
各大砲が的を外す確率は1-(1/100)=99/100

1ターン   (99/100)^100
2ターン   (99/100)^99
3ターン   (99/100)^98
     〜〜〜
99ターン  (99/100)^2
100ターン (99/100)^1

これら100個の積が求める確率。
(99/100)^(100+99+98+ ・・・ +2+1)=(99/100)^5050
99%の5050乗は果たしていくらに?

100個の大砲が何%の精度なら
一発必中の大砲一個で相手になる?<これが互角の一例

107 名前:>4 投稿日:2000/10/10(火) 23:45
>99%の5050乗は果たしていくらに
9.07*10^(-23)
ほとんどゼロといっていいね。


 5050発も撃てばそらあたるだろ。
 下手な鉄砲も数うちゃ当たる。




108 名前:再掲 投稿日:2000/10/12(木) 16:28
問題;6つの連続した整数がある。(最小の数をNとする)
これらを2つのグループに分ける時、
それぞれに入っている数の積が等しくなるように分割できるNの値を
全て求めよ。

109 名前:>108 投稿日:2000/10/12(木) 17:00
昔のIMOだっけ?

110 名前:132人目の素数さん 投稿日:2000/10/12(木) 23:50
>5050発も撃てばそらあたるだろ
10000発なんだけどね



111 名前:>110 投稿日:2000/10/12(木) 23:55
ハァ?何が10000発?

112 名前:>111 投稿日:2000/10/13(金) 03:12
106は間違ってる、と言いたいのでは?

113 名前:113投稿日:2000/11/01(水) 23:20
箱の中にn枚のカードがあり、それぞれに1からnまでの自然数
が書かれている。無作為にカードを箱から一枚取ったとき、そのカード
に1が書かれている確率は1/nである。

では、仮に箱の中に”可算無限枚”カードが入っているとして、それらに
は重複することなく自然数が書かれているとする。
箱の中からカードを一枚取ったとき、1が書かれている確率は???

114 名前:96投稿日:2000/11/02(木) 00:19
確か百発一中の大砲が13基くらいの時に
互角になったように記憶しているんですがどうでしょう。

115 名前:拡張?投稿日:2000/11/02(木) 02:22
106> 100個の大砲が何%の精度なら
106> 一発必中の大砲一個で相手になる?<これが互角の一例

114> 確か百発一中の大砲が13基くらいの時に
114> 互角になったように記憶しているんですがどうでしょう。

互角にする考え方にもいろいろあるんですね。
100基の百発一中の大砲に対して
一発必中の大砲を何基に増やせば互角になるか、とか。


  A軍:一発必中の大砲がm基
  B軍:的中率p(0<p<1)の大砲がn基

  与えられたm,nに対し
  互角の勝負にするにp=f(m,n)を求めよ


こんな問題だったらどうなっちゃうんでしょう?(^^;

m>1のとき
B軍は戦略を練る必要があるんでしょうか?
A軍のm基を狙うのに一基ずつ集中放火を浴びせるのが最適か否か。
てきとうに分散させても同じなのか。

余裕がある人は考えてみてください。
あちしには無理〜

116 名前:>96=114投稿日:2000/11/02(木) 02:37
106の考え方であっているのなら・・・・・

Σ(1〜11)=66
Σ(1〜12)=78
Σ(1〜13)=91

11基 (0.99)^66≒51.51%
12基 (0.99)^78≒45.66%
13基 (0.99)^91≒40.06%

たった1基の百発百中側を有利にするには
百発一中側の大砲を11基まで減らさないといけない?

117 名前:がんばる君投稿日:2000/11/02(木) 07:19
互角にするために百発一中側の精度を下げてみる。

百発百中軍1基

1発P中軍100基
の戦闘が互角になる時は・・・

(1-P)^5050=0.5
P=0.000137247・・・
∴百発百中軍1基は万発1.37中軍100基と互角
または[1/P≒7286.11]を利用して
∴百発百中軍1基は7286発一中軍100基と互角

#百発百中軍をm基として考えるととたんに難しい問題になるなぁ。

118 名前:KARL投稿日:2000/11/04(土) 03:03
出席番号1〜nの生徒たちを1列にでたらめの順番に並べたとき、1,2とか15,16と
いうように、続き番号の生徒がその順に並んでいるところが1ヶ所もない並び方にな
る確率はいくらか。


119 名前:132人目の素数さん投稿日:2000/11/04(土) 03:06
>おやつのKARL

その問題のどこが面白いの?

120 名前:132人目の素数さん投稿日:2000/11/04(土) 03:18
>>119
どうやらその問題はものすごくすっきりした形になるらしい。
KARLさん、気が向いたら答え教えてね。

121 名前:>118投稿日:2000/11/04(土) 04:26
n≧2で帰納法かにゃー

122 名前:自称京大医学部生投稿日:2000/11/04(土) 04:45
う〜む。

123 名前:tr > 118=KARLさん投稿日:2000/11/04(土) 04:49
P(n=1)=1/2!, P(n=2)=3/3!, P(n=3)=11/4!, P(n=5)=53/5! まで確認。

漸化式の嵐にまず座礁。
チャート (2項係数を求めるみたいなの) を書くもふたたび座礁。
今日の海は大荒れです。(涙)

124 名前:自称京大医学部生投稿日:2000/11/04(土) 05:26
なーるほど。
帰納的に求められそうやなあ。
n=k+1の時の場合の数は、n=kの時の場合の数から求められるよなあ。
n=kの時、続きにならないようにうまく並べて、んで、次にk+1人目をkの後ろ以外のk個所の場所のどれかに入れたげればいいから、(あ、一列って縦列だよね?)
N(k+1)=kN(k)かな?
N(2)=1から、N(n)=(n-1)!
また、n人を一列に並べるのはn!通り
よって、P(n)=(n-1)!/n!=1/n
あってるかなあ??問題の取り違いしてるかもしれないけど...

125 名前:tr投稿日:2000/11/04(土) 05:53
>>124=自称京大医学部生さん
m人が 1列に並んだときに、
順序を保つ組の数が n である確率を P(m,n) で表すと
  P(m+1,0) = P(m,1)*{1/(m+1)} + P(m,0)*{(m/(m+1)}
です。右辺の第1項がクセモノなんですよ。(涙)

126 名前:132人目の素数さん投稿日:2000/11/04(土) 06:06
>118
e_n=1/2!-1/3!+1/4!+…+(-1)^n/n!
としたとき、求める確率は
P(n)=e_n*(n+1)/n
ではないかと(n≦5で確認済み。trさん感謝)

極限値が1/eに収束するだろうと予想して
10分ででっちあげた式ですので証明はありません。
どなたかフォローを。

127 名前:tr > 126さん投稿日:2000/11/04(土) 06:16
すごーい♪ (極限値 1/e と予想できるだけの知識がほしい!)
これで安心して眠れます。(笑)

128 名前:132人目の素数さん投稿日:2000/11/04(土) 06:24
118の問題ですけど
120を読んで実験して
n≧2のときP(n)=1/2と予想してみたんですけど
私は根本的になんか間違ってるらしいです。(題意の取り違え?)

n=2
× 12
◎ 21
P(2)=1/2!=1/2

n=3
× 123
◎ 132
◎ 213
× 231
× 312
◎ 321
P(3)=3/3!=1/2

129 名前:自称京大医学部生投稿日:2000/11/04(土) 06:26
>125
ああ、、そっかあ。
第一項を忘れてた〜
なるほどなるほど。

130 名前:128投稿日:2000/11/04(土) 06:32
n=4

× 1234
× 1243
◎ 1324
◎ 1342
× 1423
◎ 1432
× 2134
◎ 2143
× 2314
× 2341
◎ 2413
◎ 2431
× 3124
◎ 3142
◎ 3214
◎ 3241
× 3412
× 3421
× 4123
◎ 4132
◎ 4213
× 4231
× 4312
◎ 4321
P(4)=12/4!=1/2

131 名前:132人目の素数さん投稿日:2000/11/04(土) 06:36
>130
1342がアウトっす。

132 名前:128投稿日:2000/11/04(土) 06:37
さて私は題意のどこを取り違えてるんでしょうか?(^^;

>続き番号の生徒がその順に並んでいるところが1ヶ所もない

これは(1,2)と並ぶのは不可で(2,1)と並ぶのはOKという意味。
・・・ではないのかな?(^^;

133 名前:128>131投稿日:2000/11/04(土) 06:40
やってしまいました。(^^;
ご指摘感謝。

134 名前:128投稿日:2000/11/04(土) 06:47
というわけで私の間違いがわかりました(恥)

123のtrさん> P(n=1)=1/2!, P(n=2)=3/3!
123のtrさん> P(n=3)=11/4!, P(n=5)=53/5! まで確認。

nが1個ずつズレてます。ササイなことですが。(^^

135 名前:126投稿日:2000/11/04(土) 07:01
>>126の式
P(n)=e_(n+1) * (n+1)/n
に修正です。添え字がズレてました。

証明は私には無理そうです。白旗掲げてもう寝ます。

136 名前:132人目の素数さん投稿日:2000/11/04(土) 07:48
126=135さんのを書き直しただけですが。

118の答え
n≧2,P(n)={(n+1)/n}{Σ[k=1->n](-1)^(k+1)/(k+1)!}

これ以上簡単にはならないんでしょうか。

137 名前:132人目の素数さん投稿日:2000/11/04(土) 11:00
n 人が 1列に並んだときに、順序を保つ組の数が k であるような
並び方の総数を A(n,k) で表すと
  A(n,0) = (n-1)*A(n-1,0) + A(n-1,1)
だけど
  A(n,1) = (n-1)*A(n-1,0)
だから、結局
  A(n,0) = (n-1)*A(n-1,0) + (n-2)*A(n-2,0),  A(1) = A(2) = 1

求める確率は P(n) = A(n,0)/n! だから
  P(n) = {(n-1)/n}*P(n-1) + {(n-2)/{n(n-1)}}*P(n-2),  P(1) = 1, P(2) = 1/2
ここで
  Q(n) = {n/(n+1)}*P(n)
と定義すると
  Q(n) - Q(n-1) = {-1/(n+1)}*{Q(n-1) - Q(n-2)},
  Q(1) = 1/2 = 1/2!, Q(2) = 1/3 = 1/2! - 1/3!
だから
  Q(n) - Q(n-1) = (-1)^{n-2}/{(n+1)n(n-1)…4}*{Q(2) - Q(1)} = (-1)^{n+1}/(n+1)!
よって
  Q(n) = Σ[k=1,n] (-1)^{k+1}/(k+1)!

以上より
  P(n) = {(n+1)/n} Σ[k=1,n] (-1)^{k+1}/(k+1)!

138 名前:137投稿日:2000/11/04(土) 11:05
この問題は、以前に解いた事がありました。

ランダムCDチャンジャーが、
どの2曲も元の曲順に演奏しない確率。

139 名前:132人目の素数さん投稿日:2000/11/04(土) 22:53
相撲の巴戦の問題って知ってる?
三人のなかで誰が有利かってやつなんだが。

140 名前:>投稿日:2000/11/04(土) 23:21
そもそも 巴戦ってどんなシステムだっけ?


141 名前:>140投稿日:2000/11/05(日) 00:40
a,b,c三人の力士がいて
まずaとbが対戦し、その勝者とcが対戦する。
最初に二連勝する力士が決定するまで、これを繰り返す。
三人の実力が同じだとすると、このルールは三人にとって公平か?

ってことです。

142 名前:tr投稿日:2000/11/05(日) 00:41
>>128さん:ご指摘感謝です♪

>>137さん
> A(n,1) = (n-1)*A(n-1,0)
この式へ至る変形 (考え方) がわかりません。(汗)
  A(n,1) = A(n-1,0) + (n-2)*A(n-1,1) + 2*A(n-1,2) (n≧4)
から、すぐさま導けるのでしょうか?

> P(n) = {(n+1)/n} Σ[k=1,n] (-1)^{k+1}/(k+1)!
P(2) から P(6)=309/6! まで、別な方法で得た確率と一致しました。
ほぼ間違いなく正解ですね♪

143 名前:自称京大医学部生投稿日:2000/11/05(日) 00:43
>>137
なーるほど。
納得しました。すごいですね。
A(n,1) = (n-1)*A(n-1,0)
これを思いつきませんでした。
考えればそういうことですね。ありがとうございました。


144 名前:自称京大医学部生投稿日:2000/11/05(日) 00:55
>>142
(n-1,0)となるようになれべてみる。
ここで、たとえば、出席番号n番の人を出席番号n-1番の後ろに
いつでも配置する(二人でひとつ)と考えると(n,1)になる。
また、同様にして、k番の後ろにk+1番の人を配置しても、(n,1)
となる。(出席番号をスライドさせればよいのであるから)
このように、すべての場合を考えるとn-1通り考えられる。
よって、A(n,1)=(n-1)*A(n-1,0)
でいいかなあ??
われながらわかりにくい文章になってしまってごめんなさい。

145 名前:tr > 自称京大医学部生さん投稿日:2000/11/05(日) 01:01
ああっ 漸化式にとらわれすぎていたのですね。
ようやく納得できました。ありがとうございます♪

146 名前:自称京大医学部生投稿日:2000/11/05(日) 01:11
>>142
144の説明はちょっとわかりにくいんで、もう1個の考え方を書いておきましょう。
さっきと似てるけど、イメージが湧きやすいかと思います。

(n-1,0)となるようにならべてみる。
その中の一人(出席番号k)を選んで分身させて、自分の後ろに出席番号k+1番とする。
はじめに並んでいた人のうち、出席番号k+1番以降の人、全員の出席番号を1づつ後ろにスライドさせる。
(例えば、8→9、15→16)
すると、その並びは(n,1)である。
ここで、kは1〜(n-1)が考えられるから(n-1)通りある。
よって、A(n,1)=(n-1)*A(n-1,0)
こっちのほうが、わかりやすいかもしれません。

147 名前:自称京大医学部生投稿日:2000/11/05(日) 01:14
>146
ごめんなさい。なんか、変な文章でした。訂正します。
>自分の後ろに出席番号k+1番とする
「自分の後ろに一人作り、その人の出席番号をk+1とする。」
です。
すいませんでした。


148 名前:137投稿日:2000/11/05(日) 01:45
自称京大医学部生さん、説明ありがとうございます。
> A(n,1) = (n-1)*A(n-1,0)

同じことなんですが、私の発想は順序が逆。

A(n,1) 連続している組は
  1→2,2→3,3→4,・・・,n-1→n
の n-1 通りある。それぞれの場合について、連続している
人たちをくっつけて1人として扱うと、そのほかでは連続
してはいけないのだから A(n-1,0) 通りある。


149 名前:自称京大医学部生投稿日:2000/11/05(日) 01:57
>148
なーるほど。
そっちのほうがわかりやすいね。
分けるかくっつけるかの違いだけどね。うんうん。

150 名前:tr投稿日:2000/11/05(日) 02:06
たいへん勉強になります。柔軟な発想が正答への道標なんですね。(感涙)

151 名前:KARL投稿日:2000/11/05(日) 02:11
118番KARLです。
自称京大医学部生さん、trさん、その他名無しの皆さん、楽しんでいただけたようでうれしいです。
137番さん、お見事。

さて、問題の続きです。(やさしいかもしれませんが面白いと思います。)

一般にn 人が 1列に並んだときに、順序を保つ組の数が k であるような 並び方の総数を A(n,k) で表すと きA(n,k)はどういう式で表されるでしょう。また、順序を保つ組の数の期待値はいくらになるでしょう。

152 名前:tr投稿日:2000/11/05(日) 04:51
期待値は E(n) = (n-1)/n です。

A(n,k) の一般形は求めていないのですが、
処理して帰納法で証明できました。

153 名前:tr投稿日:2000/11/05(日) 05:38
仮定 : E(2) = 1/2, … , E(n) = (n-1)/n = {(n-1)*(n-1)!}/n!

(n,k) の状態から (n+1,k), (n+1,k+1), (n+1,k-1) への変化が
それぞれ (n-k), 1, k パターンであることに注目して、

n!*E(n+1) = 納k=0,n] k{(n-k)*A(n,k)} + 納k=0,n](k+1){1*A(n,k)}
                       + 納k=1,n](k-1){k*A(n,k)} …(1)
       = 納k=0,n](kn+1)*A(n,k)
       = n*納k=0,n] k*A(n,k) + 納k=0,n]A(n,k) …(2)
       = n*{(n-1)*(n-1)!} + n!
       = n*n!
したがって、
  E(n+1) = n*n!/(n+1) = n/(n+1)

結論 : E(n) = (n-1)/n (n≧2)

# (1) 形式的に k=0 の項を加え、次段の にまとめてます
# (2) 仮定より 納k=0,n] k*A(n,k) = (n-1)*(n-1)! です

154 名前:tr投稿日:2000/11/05(日) 05:50
ミス発見!以下に訂正願います。<(_ _)>
  n!*E(n+1) -> (n+1)!*E(n+1)
  E(n+1) = n*n!/(n+1) -> E(n+1) = n*n!/(n+1)!

155 名前:自称@ム−民谷の住民投稿日:2000/11/08(水) 15:35
直径1Cmの円を、幅2Cm、長さ10Mの領域の中に何個詰め込む事が出来るか?。
と言う、パラドックス問題が「ム−」誌でかつて有りましたが…。
正解は、はたして出ているのでしょうか?。

気になってますので…。

156 名前:>155投稿日:2000/11/08(水) 16:48
なにがパラドックスなの?

157 名前:132人目の素数さん投稿日:2000/11/08(水) 17:27
>>155-156
201枚入るとか?

158 名前:156>157投稿日:2000/11/08(水) 17:44
2001枚のつもりで言っているんだろうけど、
もっと入ると思うよ。

159 名前:辻希美ぽてんしゃる投稿日:2000/11/08(水) 18:16
おお、こんなところでもんだいにでてるとわ・・・
さげとくのれす。

160 名前:133人目の素数さん投稿日:2000/11/08(水) 20:05
>155-157

 ちょっと考えてみれば2011個入るのはわかります。
証明は中学レベル。
 2013個は入らないという証明があるようです。
この証明は見ていませんし、私の力では思い付くことも出来ません。
誰か知っていたら教えてください。

だから2011個か2012個のどちらかが答えですね。


161 名前:132人目の素数さん投稿日:2000/11/08(水) 23:18
2011個の入れ方きぼーん

162 名前:132人目の素数さん投稿日:2000/11/08(水) 23:21
>>155
cmとmって書け! 大文字じゃ分かりづらいぞ。

163 名前:133人目の素数さん投稿日:2000/11/09(木) 03:10
>161
 2000個入れる場合は、
****…
****…
 って感じで入れますが、

** …(a)

**
*  …(b)
というような、3個を三角形型に接したものを基本にします。
で、(a)を下辺に接するように左端に置く、
(b)を上辺に接するように(a)の右側にくっつけて置く、
また(a)を下辺に接するように(b)の右側にくっつけて置く、
…と繰り返して置きます。
***
****
↑  ↑
最初の(a)と次の(a)との距離は三平方の定理から√(4√3−3)+1=2.981969…
この長さに6個入るので、1000/2.981969*6=2012.09…
図を描いてみると、最後の1個ははみ出すので、結局2011個入る。

 わかりにくかったら図を描いて見てください。

164 名前:133人目の素数さん投稿日:2000/11/09(木) 03:17
>163
 ありゃや、行頭の1バイトスペースってなくなっちゃうのか。
直角三角形みたいな図になっているけど、図の(a)(b)は3つの円を正三角形型に接して置いたものです。
一円玉をたくさん用意してやってみるとよくわかります。

165 名前:161投稿日:2000/11/09(木) 03:33
>>163-164
ありがとん

166 名前:132人目の素数さん投稿日:2000/11/09(木) 06:24
 ●●   こうやって置いてくの?
.  ●
 ●●
.  ●
 ●●
.  ●
 ●●
 ↓↓

167 名前:132人目の素数さん投稿日:2000/11/09(木) 06:56


囚人A,B,C,Dがいる。それぞれ、白、黒、白、黒の帽子をかぶっている
AとBの間には壁がある
DからはB,Cが見える。
CからはBが見える。
A,Bからは誰も見えない。
ある時、自分の帽子の色がわかれば牢屋を出してくれることになった。
囚人たちにわかっているのは、帽子が黒が2、白が2とういうこただけ

牢屋をでれるのは、誰?



168 名前:>167投稿日:2000/11/09(木) 12:57
だれもでれない。



169 名前:132人目の素数さん投稿日:2000/11/09(木) 14:42
CはDが出られたかどうかを知ることが出来れば出られる。

・・・ていうかこの問題、ホントに条件はこれでいいの?

170 名前:168>169投稿日:2000/11/09(木) 15:14
問題がこれであっているなら、
CからDは見えないし、CはDに見られてることも知らないはず。

171 名前:132人目の素数さん投稿日:2000/11/09(木) 18:49
Aと看守が通じてると見た

172 名前:133人目の素数さん投稿日:2000/11/10(金) 05:17
>166
いや、下のように。*は下辺に+は上辺に接して、左に詰める。

  *++*++*++*++…
 **+**+**+**+ …



173 名前:うきゃ>167投稿日:2000/11/10(金) 10:06
*ほかの人が出ていくのが分かる場合

最初,誰も出れない

ただCは,もし自分が黒だとしたらDはすぐに出られるはず.
それなのにDが出ないと言うことは,自分は白.
Cが出る.

そういうことかな?
それとも,出ていった人の帽子の色は全員見ることができる?(^^;

174 名前:168>173投稿日:2000/11/10(金) 11:32
>囚人たちにわかっているのは、帽子が黒が2、白が2とういうこただけ
こう書いてあるんだから、CはDに見られていることは知らない。
Dが見えていないのだから、Dが出ていったかどうかもわからい。

175 名前:169>174投稿日:2000/11/10(金) 11:59
なるほど。
Cは「Dが自分とBを見ている」事も知らないはずだから
Dがどう行動しようと自分の帽子の色は分かるはずがないし、
そもそもDの行動は見えないのか。

引っかかっちゃったのかな?<俺

176 名前:うきゃ投稿日:2000/11/10(金) 12:49
そーいうひっかけ問題なの?
167さんの意図は,そうじゃないと思ったんだけど・・・.

168さんが正解だったら,見事に引っかかってるな,僕(;_;)

177 名前:168投稿日:2000/11/10(金) 12:50
こういうひっかけだったらやだな。

AとBの間には壁がある
BとCの間にはマジックミラーがある
CとDの間にはマジックミラーがある
鏡を見れば自分の帽子の色はわかる

178 名前:チョーヤ梅酒投稿日:2000/11/11(土) 02:46
ずいぶん遅レスで申し訳ありませんが、
41の問題を考えた人いませんか?
93さんは交換したほうが得と言ってますが、違う気がします。
なぜなら「あなた」が選んだ箱が当たりでもはずれでも
同様に「わたし」は箱を一つ捨てることができるからです。
つまり情報量ゼロ。結局、どちらが得かわからないと思うんですけど。
なんか違います?


179 名前:名無しさん@お腹いっぱい。投稿日:2000/11/11(土) 03:28
14歳で二人産むとする

双子の女を産むとする。そして27歳まで毎年それを繰り返す。2かける14で
合計28人の子供を一人の母体が産むとする。
子供は、成長して14歳になったら同じことを繰り返す。死ぬことはないと
する。
そうして、初代の母体が112歳になった時点で、その曾曾曾曾曾孫にあたる
人数は一体何人であるか。
こういうことを考えると夜も寝られません。


180 名前:169>177投稿日:2000/11/13(月) 16:22
って、それじゃ意地悪問題(汗

そろそろ正解(もしくは真相)をおしえて>167

181 名前:132人目の素数さん投稿日:2000/11/13(月) 16:34
3億と3、どっちが1億に近いか。

182 名前:>投稿日:2000/11/14(火) 09:13
3 !
3の方が問題文中の1億に近い場所に出現

”3と3億、どっちが1億に近いか。"
なら3億!

183 名前:132人目の素数さん投稿日:2000/11/14(火) 09:49
>>182
面白い!


184 名前:132人目の素数さん投稿日:2000/11/16(木) 05:14
皆さんこの問題は知ってるでしょうか?僕には解りません。
どなたかうまく説明してください。

死刑囚がいた。かれは来週の月曜日から土曜日までのどこかで処刑されることが決まっていた。その国の法律で、処刑日は彼には知らされないが、もし処刑される朝、今日処刑されることを予告できたならば、かれは罰を免れることが出来ると言う。
死刑囚は考えた。自分は土曜日に処刑されることはない。なぜなら、もし自分が金曜日まで生きていたならば、自分は土曜日に処刑されることがわかり、罰を逃れることが出来る。このような処置はしないはずだ。
それならば自分は金曜日までに処刑されるはずだ。しかし、金曜日に処刑されるということはない。なぜなら、自分は土曜日には処刑されないということがわかっているから、もし自分が金曜日まで生きていたならば、金曜日に処刑を予告することが出来る。
従って自分は木曜日までに処刑されるはずだ。
同じ考え方でいけば、木曜日にも水曜日にも処刑されることはなく、自分を処刑できる曜日はなくなってしまう。
かれは自分の論理が完璧であると思い、満足していた。が、結局彼は水曜日に処刑され、そのことを彼は予告できなかった。死刑囚の考え方のどこに誤りがあったのだろうか?

185 名前:132人目の素数さん投稿日:2000/11/16(木) 05:26
>>184
状況そのものがパラドックス。
死刑囚の推論は正しいし、処刑執行者も正しい。

186 名前:132人目の素数さん投稿日:2000/11/16(木) 07:37
>181
3億。
理由:
 3は「億」を加えるだけで3億になるが、「1億」にするには全部書き換えなければならない。


187 名前:132人目の素数さん投稿日:2000/11/16(木) 10:53
>184
抜打ちテストのパラドクスのパクリだ(藁

188 名前:辻希美ぽてんしゃる投稿日:2000/11/16(木) 11:58
>>187

「きべんろんりがく」というほんでも
しけいしゅうのおはなしだったとおもうんれすけど・・・

どっちがさいしょなんれすかね?

189 名前:辻希美ぽてんしゃる投稿日:2000/11/16(木) 12:10
>>139

そういえばともえせんのもんだい。じゅけいずかいてしらべてみました。

さいしょにたたかうほうのふたりが5/14 で、
そのつぎにたたかうひとは4/14=2/7 でふりになるんれすね。

でも、じゅんばんをくじできめたらもんだいないれす。


190 名前:132人目の素数さん投稿日:2000/11/16(木) 12:33
>>188 >>189
漢字を使って書け。
短小包茎がっ

191 名前:132人目の素数さん投稿日:2000/11/16(木) 12:38

【基礎データ】

Name:辻希美ぽてんしゃる

LV:今井級 (要注意人物です)
HP:ktsurut級 (しつこいです)
MP:MilkTea級 (トンデモです)


192 名前:132人目の素数さん投稿日:2000/11/16(木) 15:08

【基礎データ】

Name:みちたか

LV:ごりら級 (要注意人物です)
HP:ktsurut級 (しつこいです)
MP:michitaka級 (トンデモです)

てへてへ

193 名前:132人目の素数さん投稿日:2000/11/16(木) 20:11
LV:今井級 (要注意人物です)
HP:ktsurut級 (しつこいです)
MP:MilkTea級 (トンデモです)

↑すべて同一人物です。

194 名前:132人目の素数さん投稿日:2000/11/16(木) 20:17
>>193
似たような連中だけど、違うと思う

195 名前:★ 要注意人物トンデモリスト ★投稿日:2000/11/16(木) 20:26
【Yahoo】
imaigrjp
ktsurut
MilkTea

【2ch】
今井弘一
辻希美ぽてんしゃる


196 名前:132人目の素数さん投稿日:2000/11/16(木) 20:55
>184
「自分は土曜日に処刑されることはない」が仮定にすぎないところがへん?
土曜日に処刑されるかもしれないし、されないかもしれないのでは。

197 名前:132人目の素数さん投稿日:2000/11/16(木) 22:21
>辻希美ぽてんしゃる
おめえみたいなヲタじゃねんだから
ののたんハァハァなんていわねえよ、童貞が。

198 名前:132人目の素数さん投稿日:2000/11/17(金) 07:47
>>184
土曜日に関する考察を、

P「自分が金曜日まで生き残る」
Q「土曜日は処刑されることはない」

と置くと「PならばQ」と書ける。

この命題が「真」になる条件は、

1、Pが真であり、Qが真である。
2、Pが偽である

のどちらかになるが、
死刑囚は1しか考えずに「Pが真」と考えてしまったことが間違い。

199 名前:数学わかんねー投稿日:2000/11/19(日) 21:59
>>184
囚人は処刑日を予告する権利を1回ぶんしか持っていないのですよね。

土曜日まで処刑が伸びて、その朝に予告をすればもちろん逃れることができるけど、
その為には「予告する権利を土曜日まで使わずに残している」ことが必要です。

囚人は、仮にこのように土曜日から作戦を立て始めるのなら、金曜日の作戦を練る前に、
どうやって土曜まで権利を残しておくかも熟慮しなければなりませんね。


200 名前:184投稿日:2000/11/20(月) 04:25
>>188
ありがと
>>196
そうなんですよねぇ。もっかい考えたらそう思えてきました。
>>198
きっとその論理式で説明されてるんだと思うんですけど
どうもしっくりこなくて。
>>199
おっしゃる通りです。熟慮すればするほど眠れなくなる。。

自分の睡眠時間の為、こう言い聞かせるようにしました。
間違いがあれば教えてください。
この処刑を「勝てば生負ければ死」というゲームだとすると
処刑人はなんとしてでも殺そうとしてくる。
ところが囚人の推論によると「処刑人には必勝法はない」
それを囚人が勝手に「処刑出来ない」と早合点して処刑されてしまった。と
どうでしょうか?

201 名前:132人目の素数さん投稿日:2000/11/20(月) 13:19
>>200
はっきりさせて欲しいんだけど、死刑囚の予測行為は一回に限るの?
一回に限るなら死刑囚の推論は成り立たないよ。
「明日と明後日のどちらかに死刑を執行する場合」を考えてみてよ。

202 名前:132人目の素数さん投稿日:2000/11/21(火) 13:45
>>201
そりゃあ、1回きりにきまってるじゃん。
何回でもOKだったら、囚人は毎朝「今日が執行日!」と言うでしょ。

203 名前:201投稿日:2000/11/21(火) 17:46
>>202
そうっすよね。
でも、予測が一回のみなら、囚人の推論は
「運良く土曜日の朝まで生き残り、まだ一回も予測行為をしていなかったら
土曜日の朝に予測行為をする事で生き延びる」
となるはず。よって囚人の推論が誤り。

204 名前:KARL投稿日:2000/11/22(水) 22:51
a,b,c,dに関する次の方程式には無限個の整数解があることを証明せよ。

  a^2+b^3+c^4=d^5



205 名前:>204投稿日:2000/11/23(木) 00:59
a,b,に関する次の方程式には無限個の整数解がある

a^2+b^3=0

206 名前:KARL投稿日:2000/11/23(木) 01:12
204について
訂正です。

a,b,c,dに関する次の方程式には無限個の自然数解があることを証明せよ。
                   ~~~~~~
  a^2+b^3+c^4=d^5



207 名前:KARL投稿日:2000/11/23(木) 01:15
†206

~~~~~~の位置ずれてしまいました。
自然数です。
~~~~~~


208 名前:名無しさん@お腹いっぱい。投稿日:2000/11/23(木) 01:59
>206
tを任意の非負整数として、
a=3^(30t+12) b=3^(20t+8) c=3^(15t+6) d=3^(12t+5)
a^2=3^(60t+24) b^3=3^(60t+24) c^4=3^(60t+24) d^5=3^(60t+25)より
a^2+b^3+c^4=3^(60t+24)+3^(60t+24)+3^(60t+24)=3^(60t+25)=d^5
となる。



209 名前:名無しさん@お腹いっぱい。投稿日:2000/11/23(木) 02:06
ごちゃごちゃして、ごめん。やりなおし

a=3^(30t+12),b=3^(20t+8),c=3^(15t+6),d=3^(12t+5)とおくと、

a^2=b^3=c^4=3^(60t+24),d^5=3^(60t+25)となるから、

a^2+b^3+c^4=3*3^(60t+24)=3^(60t+25)=d^5となる。

210 名前:KARL投稿日:2000/11/27(月) 01:23
次の式を計算せよ。 (分数です)

   (10^4+324)*(22^4+324)*(34^4+324)*(46^4+324)*(58^4+324)
N= --------------------------------------------------
   (4^4+324)*(16^4+324)*(28^4+324)*(40^4+324)*(52^4+324)


211 名前:132人目の素数さん投稿日:2000/11/27(月) 14:34
>>210

   164038991325531565288789120000
N= ---------------------------------- = 373
   439782818567108754125440000

全然おもしろくない問題ですねぇ。
出題者が落ちこぼれかしら。。。


212 名前:132人目の素数さん投稿日:2000/11/27(月) 15:03
>>211
この口調は本物???

213 名前:132人目の素数さん投稿日:2000/11/27(月) 17:19
>>212
ちがいますねぇ。。。
本物は1+1が精一杯で分数の計算はできません。


214 名前:>210投稿日:2000/11/27(月) 17:31

P(k)≡(12k-2)^4+(4*3^4)
Q(k)≡(12k-8)^4+(4*3^4)
N(n)≡Π[k=1->n]{P(k)/Q(k)}

((恒等式)) a^4+4b^4={(a-b)^2+b^2}{(a+b)^2+b^2}
P(k)={(12k-5)^2+3^2}{(12k+1)^2+3^2}
Q(k)={(12k-5)^2+3^2}{(12k-11)^2+3^2}

P(k)/Q(k)
={(12k+1)^2+3^2}/{(12k-11)^2+3^2}
={(12k+1)^2+3^2}/{(12(k-1)+1)^2+3^2}

R(k)≡{12(k-1)+1}^2+3^2≠0
P(k)/Q(k)=R(k+1)/R(k)

N(n)
=Π[k=1->n]{P(k)/Q(k)}
=Π[k=1->n]{R(k+1)/R(k)}
=R(n+1)/R(1)
={12n(6n+1)/5}+1

N=N(5)=12*31+1=373

215 名前:MilKTae投稿日:2000/11/29(水) 01:04
a[0]=0, a[n+1]=√(2+a[n]) とするとき、
lim[n→∞] 2^n*√(2-a[n]) を求めよ。

216 名前:MilKTae投稿日:2000/11/29(水) 01:05
↑出題したのに下げちゃった。あげ

217 名前:132人目の素数さん投稿日:2000/11/29(水) 09:38
MilKTae=ppp ?

218 名前:自称医学博士投稿日:2000/11/29(水) 19:39
ここ面白いですね。私も一題出題させてください。

A,B,C,D,E,F,G,H,I,J,Kの12個の金貨がある。そのうち1個だけ、にせものが
混じっていて、その偽物は重さが違う。但し、重いのか軽いのかは分からない。
天秤を3回だけ使って、偽物をあてよ。

219 名前:132人目の素数さん投稿日:2000/11/29(水) 20:06
>>218

30ドルホテルの問題と同じく
週1ぐらいで必ずだれかがこの問題投稿するのはなぜ?



220 名前:ビートきよし投稿日:2000/11/29(水) 23:37
私にも出題させてください。

0≤α,β,γ≤π,α+β+γ=πのときsinαsinβsinγの最大値を求めろ。

221 名前:132人目の素数さん投稿日:2000/11/29(水) 23:46
↓そういえばこの2番って誰か答えた?
http://cheese.2ch.net/test/read.cgi?bbs=math&key=973268779

222 名前:>>221投稿日:2000/11/29(水) 23:52
これって、0^0扱いじゃなかったっけ?

223 名前:132人目の素数さん投稿日:2000/11/30(木) 03:32
>>218
A〜Kだと11枚だ。
あと、↓を見とけ。
http://cheese.2ch.net/test/read.cgi?bbs=math&key=967889985&st=56&to=56&nofirst=true

224 名前:132人目の素数さん投稿日:2000/11/30(木) 07:47
>>220
見飽きた。uzai

225 名前:>215投稿日:2000/11/30(木) 13:57
Vieteの公式の逆数ですね。

226 名前:132人目の素数さん投稿日:2000/12/02(土) 00:03
次のようなゲームをします。
まずゲームに参加すると、賞金1円が与えられます。
コインを投げ、表が1回出るごとに賞金は2倍になりますが、
裏がでたらゲームは終了です。
例えば1回目に裏が出たら賞金は1円、表が3回出て4回目に
裏が出たら賞金は8円です。
さて、参加費がいくら以下ならこのゲームに参加したほうが得
でしょうか。

227 名前:132人目の素数さん投稿日:2000/12/02(土) 00:04
>>226 なんとかのぱらどっくすきゃ?


228 名前:132人目の素数さん投稿日:2000/12/02(土) 01:04
>226
これもNG問題化しつつあるな。

229 名前:132人目の素数さん投稿日:2001/02/14(水) 03:10
age

230 名前:132人目の素数さん投稿日:2001/02/14(水) 03:19
コンビニで買い物をすると、硬貨が貯まって鬱陶しいです。
1568円の買い物をしたときに、千円札2枚で払うと、
お釣りは432円で、手元の硬貨が9枚増えてしまいます。
でも、あらかじめ硬貨を用意しておいて1570円を払えば、
お釣りは2円。使った硬貨が4枚で、お釣りの硬貨は2枚だから、
差し引き硬貨が2枚減ります。
このように、あらかじめ小銭を用意しておいて、何円の買い物をしても、
持っている硬貨の数が「同じまたは減る」ようにしたいです。

さて、このときに、あらかじめ用意する必要のある小銭の、
最小枚数は何枚でしょう? その理由も示して下さい。

ただし、店員はいじわるをせずに、お釣りの硬貨を最小枚数でくれます。
また、紙幣に関しては、何枚あっても気にしないものとします。

あ、そうそう、持っていく硬貨の組み合わせには、一つだけ条件があります。
硬貨の組み合わせは、その金額における最小枚数に限定します。
硬貨を減らしたいのに、冗長な硬貨を持ち歩くのは本末転倒だからです。
つまり、例えば、75円持っていくとしたら、その硬貨の組み合わせは
「50円玉 + 10円玉2枚 + 5円玉」の組み合わせに限定されます。

231 名前:231投稿日:2001/02/16(金) 06:38
>>226
1円獲得する確率は1。
2円獲得する確率は1/2。
4円獲得する確率は1/4。
8円獲得する確率は1/8。

どれも期待値は1円だね。
期待値的には気分次第♪
じゃんけんに100万円賭けるか10円賭けるかといっしょだな。

232 名前:231投稿日:2001/02/16(金) 06:45
追加。
f(100万円)=100万円もらったときの効用−100万円失ったときの効用
f(10円)=10円もらったときの効用−10円失ったときの効用
という関数f(x)を考える。

「効用」というのは経済学用語で、「満足度」のようなもの。
経済学的には限界効用(効用の微分)は減衰する。
よってf(10円)>f(100万円)
一般的には、x<yなら、f(x)>f(y)
よってfmax(x)=0(x=0)
つまり賭けをしないのが一番満足度が高い。

どうだ。期待値の概念をふっとばす考え方だろう?

233 名前:132人目の素数さん投稿日:2001/02/16(金) 16:05
↑馬鹿発見あげ

234 名前:231投稿日:2001/02/17(土) 00:35
ナイスレス(ワラ

235 名前:132人目の素数さん投稿日:2001/02/22(木) 03:03
ABCDE÷3=FGHIに1から9を1かいずつ入れてなりたたせる。
誰かできますか?

236 名前:132人目の素数さん投稿日:2001/02/22(木) 07:07
>>235

17469/3=5823
17496/3=5832

237 名前:230投稿日:2001/02/22(木) 07:23
誰も解いてくれない。(T_T)

238 名前:↑くだらないから投稿日:2001/02/22(木) 07:56
500円玉 1枚
100円玉 4枚
50円玉 1枚
10円玉 4枚
5円玉 1枚
1円玉 4枚

239 名前:132人目の素数さん投稿日:2001/02/22(木) 21:15
age

240 名前:頭の体操になる?投稿日:2001/02/22(木) 21:18
N  N1  N2  N3     Nk-1
Σ  Σ  Σ  Σ  ・・・Σ 1 =?
N1=1 N2=1 N3=1 N4=1    Nk=1

崩れませんように。
最後のΣの上は、N の添え字が k-1 ってことだよ。


241 名前:240投稿日:2001/02/22(木) 21:21
やっぱ、崩れたか。

Σ[N_1=1,N]Σ[N_2=1,N_1]Σ[N_3=1,N_2]Σ[N_4=1,N_3]・・・Σ[N_k=1,N_{k-1}] 1 =?


242 名前:132人目の素数さん投稿日:2001/02/23(金) 00:00
>236
解いてくれてありがとう。これはいかがでしょうか?
A÷BC+D÷EC+F÷GH=1
1から9の整数を一回ずつ入れます。
誰か解いてくれませんか?

243 名前:132人目の素数さん投稿日:2001/02/23(金) 07:29
>>242
9/12+7/68+5/34=1

244 名前:132人目の素数さん投稿日:2001/02/23(金) 08:51
>>241
NHk

245 名前:132人目の素数さん投稿日:2001/02/23(金) 20:29
244さん、あたり!
じゃ、これは?

Σ[N_1=1,N+1]Σ[N_2=1,N_1+1]Σ[N_3=1,N_2+1]Σ[N_4=1,N_3+1]・・・Σ[N_k=1,N_{k-1}+1] 1 =?


246 名前:132人目の素数さん投稿日:2001/02/23(金) 23:20
問題です・・・・3・3・8・8 の 四つの数字を使い + ,−, ÷, ×,( ) の四則演算のみを使って、 答えが『24』になる計算式を作りなさい 。


247 名前:名無しさん@お腹いっぱい。投稿日:2001/02/23(金) 23:40
3*8
3*8
24が2つできましたとさ。

248 名前:8/(3-8/3)投稿日:2001/02/24(土) 00:09
>>246
アラシですか?

249 名前:さるやまはげのすけ投稿日:2001/02/24(土) 02:12
>>246
http://cheese.2ch.net/test/read.cgi?bbs=math&key=982412754

250 名前:132人目の素数さん投稿日:2001/02/24(土) 02:53
>243
あざやかですね。ありがとうございます。

251 名前:132人目の素数さん投稿日:2001/02/24(土) 19:45
流石ですね!

252 名前:どぱきのん投稿日:2001/02/25(日) 05:03
小さい頃からナンバープレートや電車の切符の4桁の数字をみると
四則演算で10を作っていた。で、2を含むとかなり高確率で10が
作れることに気づいた。そこで問題。

1〜9までの整数から重複を許して2を含む4個の数を選んだとき、
+ ,−, ÷, ×,( )の四則演算のみで10が作れない組み合わせは?

253 名前:132人目の素数さん投稿日:2001/02/25(日) 09:29
2111

254 名前:132人目の素数さん投稿日:2001/02/25(日) 22:53
11


255 名前:132人目の素数さん投稿日:2001/02/26(月) 00:21


256 名前:どぱきのん投稿日:2001/02/26(月) 01:09
多分2111のほかにあと2つ。

257 名前:132人目の素数さん投稿日:2001/02/26(月) 01:35
>>252
あとこれだけわからん。
1112
1122
1277
1289
2257
2267
2299


258 名前:132人目の素数さん投稿日:2001/02/26(月) 01:36
あげわすれた

259 名前:132人目の素数さん投稿日:2001/02/26(月) 01:38
ついでに、3を含むやつはこれ以外はできた。
1113
1399
3444
3466
3478
3669
3779
3999


260 名前:132人目の素数さん投稿日:2001/02/26(月) 01:50
1277は
(7-1)/2+7=10
だな。

261 名前:132人目の素数さん投稿日:2001/02/26(月) 03:07
2267は2*7+2-6=10
3478はできるぞい(有名)。

262 名前:132人目の素数さん投稿日:2001/02/26(月) 03:35
1289は1*2*9-8=10,2299は(2+9+9)/2=10,3466は(4*6+6)/3=10
つーかここ見れ
http://susumuoda.tripod.co.jp/2000/nov/20/res3.txt


263 名前:どぱきのん投稿日:2001/02/28(水) 04:48
1〜9で4つが全て異なる場合は必ず10にできたはず。
1199、1158、3478あたりはやや難しくて面白い。



264 名前:132人目の素数さん投稿日:2001/03/02(金) 02:31
どんどんさがれ。

265 名前:132人目の素数さん投稿日:2001/03/02(金) 20:57
kokoko



266 名前:132人目の素数さん投稿日:2001/03/04(日) 13:35
sage

267 名前:ろうさんかんざんらん投稿日:2001/03/07(水) 17:09
さげ

268 名前:132人目の素数さん投稿日:2001/03/07(水) 20:53
2つのサイコロがある。
そのサイコロを・・・・・・・、グハッ。

269 名前:132人目の素数さん投稿日:2001/03/07(水) 21:06
↑どうした?

270 名前:ろうさんかんざんらん投稿日:2001/03/10(土) 02:46
無理して下げることないか、という考えに傾きつつある郎三勘山嵐でした。

271 名前:132人目の素数さん投稿日:2001/03/10(土) 17:58
飲み込もうとして喉に詰まった>>269

272 名前:132人目の素数さん投稿日:2001/03/16(金) 23:36
あげ

273 名前:132人目の素数さん投稿日:2001/04/11(水) 04:31
lim_{n→∞} {e^(-n) Σ_[k=0,...,n] (n^k)/(k!)} = ?

274 名前:132人目の素数さん投稿日:2001/04/12(木) 13:02
あげ

275 名前:名無しさん@お腹いっぱい。投稿日:2001/04/13(金) 15:29
 1、1、5、8の4つの数字があります。
これらの数を全て一回ずつ用い、四則演算(+、−、×、÷)
と括弧を用いて10にせよ(キップの発券番号でよくやる
パズル)。
 ただし、1と1をくっつけて11(十一)としたり、
1と8に小数点を付けて1.8としたりするのはなし
とする。また、≠も使ってはいけない。
 決してトンチ問題ではなく、まともな問題なので
ふるってご参加してください。


276 名前:8/(1-1/5)投稿日:2001/04/13(金) 16:13


277 名前:132人目の素数さん投稿日:2001/04/14(土) 00:24
この問題何回目?

278 名前:132人目の素数さん投稿日:2001/04/15(日) 10:56
>>252
ユニークな趣味だな。
友達になれそうだ!!

279 名前:132人目の素数さん投稿日:2001/04/15(日) 12:05
>>273
わからん!こたえおしえてたも。


280 名前:132人目の素数さん投稿日:2001/04/16(月) 01:07
>275
252〜263見れ。

281 名前:132人目の素数さん投稿日:2001/04/17(火) 15:06
>>273

A(n)=e^(-n)Σ[k=0,n](n^k)/(k!) とおく。
次の式は、右辺を部分積分すれば明らか。

A(n)=∫[n,∞](t^n)e^(-t)/(n!)dt

t=(√n)x+n と置換積分する。f_n(x)=[{1+x/(√n)}^n]e^{-(√n)x} とおくことにすると、

A(n)=[{n^(n+1/2)}e^(-n)/(n!)]×∫[0,∞]f_n(x)dx

初めの項については、lim[n→∞]{n^(n+1/2)}e^(-n)/(n!)=√(1/2π) である(スターリングの公式)。

g(y)={log(1+y)-y}/(y^2) とおく。このとき次のことが成立。

(1) g(y) は、y>0 で単調増加。
(2) lim[y→0]g(y)=-1/2

(1)は、g'(y)>0 を示せばよく、高校3年レベル。(2)もロピタルの定理から容易。

さて、log{f_n(x)}=(x^2)g(x/(√n)) だから、次のことが言える。

関数列 {f_n(x)} は単調減少列で、lim[n→∞]f_n(x)=e{-(x^2)/2} である。

このとき、“lim” と “∫” の入れ替えは許されるから(たとえばルベーグの項別積分定理による)、

lim[n→∞]∫[0,∞]f_n(x)dx=∫[0,∞]e{-(x^2)/2}dx=√(π/2)

よって、

lim[n→∞]A(n)={√(1/2π)}×{√(π/2)}=1/2

282 名前:132人目の素数さん投稿日:2001/04/18(水) 20:05
>218

2回でできたはずだよ。

283 名前:132人目の素数さん投稿日:2001/04/19(木) 06:33
>>282
あり得ないよ。
12個のうちどれが軽いか重いかで、24通りの結論があり得る。
天秤は一回あたり、右に傾く、左に傾く、釣り合うの3通りしかない。
よって、2回では、3^2=9 通りの結果しか判別できない。

金貨が、3^n 枚の時は、n+1回が答えになる。
3^n<2(3^n) より、n 回で不可能なことは上に書いたことと同様。
n+1 回で可能なことは、n についての帰納法による(金貨を3枚ずつ一組にすると考えれば容易)。

一般に、金貨 n 枚の時、3^(k-1)<2n≦3^k となる k をとれば、答えは k 回でいいのかな?
k 回必要なことはわかるけど、k 回で可能かどうかわからない。

284 名前:283投稿日:2001/04/22(日) 05:44
少し考えてみたんだけど・・・。

[問題] n 枚の金貨のうち一枚だけ重さが違う(n>2)。
次のそれぞれの場合、天秤を最低何回使うことになるか。
(a) 偽物を見分けるだけでよい場合。
(b) 偽物を見分け、それが重いか軽いかも判定する場合。

283 の投稿は、(b) のケースで考えていた。
で、答えはこうなるみたい。

3^(k-1)<2n≦3^k となる k をとる。
(a) の場合は、k 回。
(b) の場合は、n=(3^k-1)/2 の場合は k+1 回。その他の n では k 回。

証明の細かい所をよく詰めていないので、もしかすると間違っているかも知れないけど、
たぶんこれでいいと思う。

285 名前:不動産屋投稿日:2001/04/23(月) 16:31
この板に迷い込んだがこのスレだけは面白かった。
他にも教えてくれ。

286 名前:132人目の素数さん投稿日:2001/04/23(月) 16:58
inf{2^m/10^n | m,n は正の整数、2^m > 10^n} = 1 をしめせ。
なんてどうよ。

287 名前:でじこ@数学板投稿日:2001/04/25(水) 07:25
>>286
出典はどこにょ?
面白かったけど、ここに書き切れないくらい
長くなってしまったにょ…。

288 名前:土木作業員投稿日:2001/04/25(水) 08:43
>>286-287
問題の意味は↓でいいんでしょうか?

「m,n は正の整数、2^m > 10^n」をみたす任意のm,nにたいして
1 < (2^M/10^N) < (2^m/10^n) をみたす正の整数M,Nが存在することを示せ

289 名前:132人目の素数さん投稿日:2001/04/25(水) 09:45
>>287
わすれました。どっかの院の入試問題だったはず。
>>288
ちがいます。ただしくは
e を任意の正の数とするとき整数 m,n で 1<2^m/10^n<1+e なるものが
存在することをしめせ。
です。

290 名前:132人目の素数さん投稿日:2001/04/25(水) 20:04
>>287
証明きぼーん
1-e<2^m/10^n<1+eは言えそうだけど
常に>1であるようにとる方法が判らん

291 名前:132人目の素数さん投稿日:2001/04/26(木) 07:32
log(10)/log(2)≒m/n となる m/n を連分数近似でとればいいんじゃないの。
連分数近似なら、交互に上から下からと近づくので、ひとつおきにとれば、
0<m/n-{log(10)/log(2)}<(1/n)^2
が成り立つでしょ。

292 名前:土木作業員投稿日:2001/04/26(木) 08:19
>>289
1+e と書くべきだったのですね。なるほど、、、

>>291
これはまたあっさりと!

293 名前:132人目の素数さん投稿日:2001/04/26(木) 10:55
>>167の問題は文章だけじゃ状況が伝わりにくいでしょう。

 |
 |       _○_
○|   _○_|
 |○|

左から、黒、白、黒、白の帽子をかぶっています。
○は囚人で、みな左を向いています。

一番左の囚人は別室にいて、他の情報を
得ることはまったくできません。
それぞれ自分が何色の帽子をかぶらされている
かは知らされておらず、声を出したり身動きを
することは禁止されています。
「白い帽子が二人、黒い帽子が二人」である
ことは、囚人には伝えられています。
看守が「このなかで誰か一人でも、自分の
帽子の色がわかった物は自由にしてやる。
しかし、間違えたら全員銃殺だ」と言います。
その状況の中、答えられたのは誰でしょう?

294 名前:293投稿日:2001/04/26(木) 10:56
あわわ、図がズレた。
階段みたいな段差があるところに
立たされてると思ってください。

295 名前:132人目の素数さん投稿日:2001/04/26(木) 11:39
もし、一番右の人(D)が二つの同じ色の帽子(B,C)を見ていれば、
その逆の色を言っているはずだから、沈黙している以上、自分の色は
左から2番目の人(B)とは違う筈だ!!!!!

と考えて、右からニ番目の人(C)が叫ぶでしょう!!!!!『黒。これで自由だ!』

296 名前:293投稿日:2001/04/26(木) 12:07
>>295
大正解。簡単だった?

297 名前:295投稿日:2001/04/26(木) 12:11
>>296
ちゃんと紙を使って考えましたよ。

298 名前:鯖が移転する前にあったヤツ投稿日:2001/05/06(日) 22:52
次の無限数列には素数の項が存在しないことを示せ。
10001, 100010001, 1000100010001, ...

299 名前:名盤さん投稿日:2001/05/07(月) 03:55
結局

(10001 - 1) ^ n + .... + (10001 - 1) ^ 3 + (10001 - 1)^ 2 + 10001

見たいな式になる。はい、今日はここまで。

10001 ^ n + a1 * 10001 ^ (n - 1) + .... + an * 10001 ^ 0

みたいになって、結局、anがどうなるかが問題で、



300 名前:名盤さん投稿日:2001/05/07(月) 04:02
12個の玉があります。
その中で一個だけ重さが違うやつがあります。
てんびんを3回使ってその玉を見つけてください。


301 名前:132人目の素数さん投稿日:2001/05/07(月) 04:04
>>299
後半は考えすぎ。そんな難しい問題じゃないよ。

302 名前:132人目の素数さん投稿日:2001/05/07(月) 04:07
>>300
がいしゅつ。
ってゆうか >>284 で一般化されている。

303 名前:132人目の素数さん投稿日:2001/05/08(火) 15:27
メジャーな問題です。
イーサン・ケイニンの小説より抜粋

「ランスロットとガウェインがそれぞれ1ドルずつ賭け金を出す。次ぎに、それぞれ任意の付け値を紙に書いて封印して提出する。封を開けて、付け値の高い方が賭け金を得るが、と同時に低い値を付けた相手に、その低い付け値の額を払わねばならない。もし付け値が同額なら、賭け金は2人で折半する。ランスロット、君はいくらの値を付けるか?」

分かる方、根拠と共に正解をお願いします。
既出?

304 名前:303投稿日:2001/05/08(火) 15:31
同じくイーサン・ケイニンの小説より抜粋。

「12枚あるコインのうち、1枚は偽金で、他11枚と重さが異なる。他のコインは全て同じ重さである。天秤を3回だけ使って、どれが偽金かを決めよ。」

どうやったらできる?
既出っぽいな。

305 名前:ixi投稿日:2001/05/08(火) 15:40
よくピーターフランクルがネタにしてるやつか。
「大数」でやってたぞ。

306 名前:132人目の素数さん投稿日:2001/05/08(火) 17:47
以下の分を述語論理式で書き表せ.述語記号は適宜定義せよ.
(1)P(x)を真とするxが高々一つ存在する.
(2)nを3以上の整数とする時,x^n+y^n=z^nを満足する正の整数x,y,zは存在しない.
(3)a,b,cを任意の整数,n,mを任意の正整数とするとき,ax^n+bx^m+c=0を満足する実数xが3個以上あ
  ることはない.

307 名前:132人目の素数さん投稿日:2001/05/08(火) 17:52
>>304
激しく既出だが良問だと思う。

308 名前:132人目の素数さん投稿日:2001/05/08(火) 19:28
>>303
1ドル。三方一ドルの損。

>>304
トレカをフルコンプリートする場合の金額もわからないけど、これもわかりません。
偽金が、金より重いか軽いか、わかればわかるんですけど。
トレカフルコンプリートも、いくらかかるっぽいのかも教えてください。

309 名前:308投稿日:2001/05/08(火) 19:30
あるトレイディングカードを集めたいと思っているのですが、
カードの種類がx種、
一つのトレーディングカードのパックにy枚入っている
一つのパックがz円の場合、
x種類をあつめるのにかかるお金(期待値?)を求める式は、
どのようなものになるのでしょうか。

これです。


310 名前:308投稿日:2001/05/08(火) 20:47
100種類で全種類として、1パックに10枚はいって100円のトレーディングカード
とすると、買いだして最初の方はわかるのですが、一つのパックに、重複したカード
の入っている場合とか、ある程度あつまってからの重複分を考慮に入れてを計算する
方法がわかりません。

純粋なカードなら、買ってもお金を失うだけですが、チョコエッグとか、ビックリマンシ
ールやライダースナックのばあい、お菓子をたべないと、もったいないオバケに襲撃さ
れそうなので食べてしまい、健康も一緒に失ってしまいます。

それはそれとして、せめて、いくらぐらいかかるかを求める式があれば、新しい種類の
カードやおまけ付き玩具・菓子が商品として新しく出た場合にも、その式に、カードや
付録が何種類あるか、1パックがいくらでどれぐらいの種類のカードがはいっているか、
の数をいれれば、あつめるまえにいくらぐらいのお金がかかるかわかるのでうれしいです。

ところが、自分ではわからないのです。
どなたか、式を教えていただけないでしょうか。




311 名前:わーい ヽ(´ー`)ノ投稿日:2001/05/08(火) 21:58
5手詰め 持ち駒なし

┌──┬──┬──┬──┬──┬──┬──┬──┬──┐
│__│__│__│__│__│__│__│__│__│
├──┼──┼──┼──┼──┼──┼──┼──┼──┤
│__│__│__│__│▲竜│▲銀│__│▽王│▲角│
├──┼──┼──┼──┼──┼──┼──┼──┼──┤
│__│__│__│__│__│__│__│__│__│
├──┼──┼──┼──┼──┼──┼──┼──┼──┤
│__│__│__│__│__│▽歩│▽歩│▲桂│▲香│
├──┼──┼──┼──┼──┼──┼──┼──┼──┤
│__│__│__│__│__│__│__│__│__│
├──┼──┼──┼──┼──┼──┼──┼──┼──┤
│__│__│__│__│__│__│__│__│__│
├──┼──┼──┼──┼──┼──┼──┼──┼──┤成歩−个
│__│__│__│__│__│__│__│__│__│成香−仝
├──┼──┼──┼──┼──┼──┼──┼──┼──┤成桂−今
│__│__│__│__│__│__│__│__│__│成銀−全
├──┼──┼──┼──┼──┼──┼──┼──┼──┤成角−馬
│__│__│__│__│__│__│__│__│__│成飛−竜
└──┴──┴──┴──┴──┴──┴──┴──┴──┘

312 名前:132人目の素数さん投稿日:2001/05/08(火) 22:25
Hに直線を3本引いて、3角形を7個つくれ
ただし3角形同士は重なってはいけないとする

(啓発セミナー問題)

313 名前:>>303 1j1k投稿日:2001/05/08(火) 22:37
 

314 名前:132人目の素数さん投稿日:2001/05/08(火) 22:44
>>311
3一銀不成 3三玉 2三角成 同玉 2二竜まで

315 名前:314投稿日:2001/05/08(火) 22:52
間違えた。これじゃ1四に逃げられちゃう。


316 名前:わーい ヽ(´ー`)ノ投稿日:2001/05/08(火) 22:57
>>314
残念ながら、6手目、1四玉で詰みません。

317 名前:わーい ヽ(´ー`)ノ投稿日:2001/05/08(火) 22:58
かぶったスマソ。

318 名前:わーい ヽ(´ー`)ノ投稿日:2001/05/08(火) 23:10
>>312
これどうやって答えたらいいの??

319 名前:314@ギブアップ投稿日:2001/05/08(火) 23:29
>>311は不詰??

(TдT)マズー


320 名前:132人目の素数さん投稿日:2001/05/08(火) 23:40
>>318
問題教えてスレッドだから答えを書く必要は全くないと思われ

321 名前:308投稿日:2001/05/08(火) 23:41
よくわかりませんが、適当に
>>311
3四角成 2一王 3二歩 2二王 3三銀不成

>>309
>>310のほうをひとつお願いします。
お風呂にはいてきて寝ます。

322 名前:132人目の素数さん投稿日:2001/05/08(火) 23:46
>3四角成
ハァ?

323 名前:132人目の素数さん投稿日:2001/05/08(火) 23:53
>>322
ごめんなさい。将棋のルールも知らないで。

324 名前:わーい ヽ(´ー`)ノ投稿日:2001/05/09(水) 00:07
>>319
詰まないように見えますがちゃんと詰むんですよ。
5手詰のなかでは一番難しい部類かもしれませんね。有名な作品です。
答えはもう少しおいときます。

>>320
うーー、せっかく分かったのに。答えたいよ〜〜。

>>321
ワラタ

325 名前:132人目の素数さん投稿日:2001/05/09(水) 00:08
分からないときはどんな問題でも
1一歩打
と書いておけばいいよ
それだけで、みんな分かってくれる

326 名前:132人目の素数さん投稿日:2001/05/09(水) 00:38
持ち駒:飛
┌──┬──┬──┬──┬──┬──┬──┬──┬──┐
│__│__│__│__│__│__│▽歩│▽王│__│
├──┼──┼──┼──┼──┼──┼──┼──┼──┤
│__│__│__│__│__│__│__│__│▽銀│
├──┼──┼──┼──┼──┼──┼──┼──┼──┤
│__│__│__│__│__│__│__│__│__│
├──┼──┼──┼──┼──┼──┼──┼──┼──┤
│__│__│__│__│__│__│__│__│__│
├──┼──┼──┼──┼──┼──┼──┼──┼──┤
│__│__│__│__│▲角│__│__│__│__│
├──┼──┼──┼──┼──┼──┼──┼──┼──┤
│__│__│__│__│__│__│__│__│__│
├──┼──┼──┼──┼──┼──┼──┼──┼──┤
│__│__│__│__│__│__│__│__│__│
├──┼──┼──┼──┼──┼──┼──┼──┼──┤
│__│__│__│__│__│__│__│__│__│
├──┼──┼──┼──┼──┼──┼──┼──┼──┤
│__│__│__│__│__│__│__│__│▽馬│
└──┴──┴──┴──┴──┴──┴──┴──┴──┘

327 名前:132人目の素数さん投稿日:2001/05/09(水) 00:43
>>326
それなら3秒で分ったぞ!
22飛 11玉 28飛成まで
19馬がぁゃιすぎ

328 名前:132人目の素数さん投稿日:2001/05/09(水) 00:54
詰め将棋は囲碁・将棋板へ逝け

329 名前:132人目の素数さん投稿日:2001/05/09(水) 00:55
>>311は出題ミスって無い?

330 名前:132人目の素数さん投稿日:2001/05/09(水) 01:09
▲3一桂 △同玉 ▲4一龍 △4三玉 ▲2一角成

331 名前:132人目の素数さん投稿日:2001/05/09(水) 01:11
▲3一桂 → ▲3二桂成

332 名前:132人目の素数さん投稿日:2001/05/09(水) 01:12
>>328
たまにはいいじゃん。少しぐらいマターリ息抜きしよーよ。
数学好きな人こういうのも好きでしょ?

333 名前:314投稿日:2001/05/09(水) 01:15
分った!。。。先に答えられてた。。。
最後空き王手なんだね!

334 名前:わーい ヽ(´ー`)ノ投稿日:2001/05/09(水) 01:16
>>330-331

素晴らしい。2000マターリあげる。

335 名前:わーい ヽ(´ー`)ノ投稿日:2001/05/09(水) 01:30
気を取り直して数学関係に戻りましょう。

1 2 3 4 5 6 7 8 9 = 100

これが成り立つように、+−を入れてください。

例)
12+3−4+567+8−9 = 100 見たいな感じ。
(↑これは成り立ってないよ)

336 名前:132人目の素数さん投稿日:2001/05/09(水) 01:34
>304
正解を教えてください。
これって、「最低何回でできる?」って問題ですよね。
どっかで聞いたことあるけど4回までしかいきません。

337 名前:わーい ヽ(´ー`)ノ投稿日:2001/05/09(水) 01:47
3回。

理由は・・・めんどくさい。
キーワードは逐次選択法。

338 名前:132人目の素数さん投稿日:2001/05/09(水) 01:54
4個ずつ乗せる。釣り合った場合は残りは4枚。
釣り合わなかった場合は、左右の天秤から金貨を1枚ずつ取り、
4組のペアを作る。この組を1枚の金貨と見なせば4個の場合に
還元されている。
だから、4枚のケースを2回でできればよい。(ただし純粋に4枚の
ケースは2回ではできない。本物と分かっている金貨がもう一枚あれ
ば2回でできるのがミソ)。
4枚のうち、右に1枚、左に2枚乗せ、既に本物と分かっている金
貨を1枚右に乗せる。あとは易しい。
13個の場合も3回でできるようだよ。


339 名前:132人目の素数さん投稿日:2001/05/09(水) 02:02
補足。
4枚と本物1枚のケースでは偽物が重いか軽いかも分かることをいっておく
必要がある。(そうでないと組の場合、どっちか分からない)。

なお、12枚を3回の場合も偽物が重いか軽いかまで分かる。
13枚を3回のケースでは、そこまでは判定できない。

340 名前:132人目の素数さん投稿日:2001/05/09(水) 02:19
>>335
確か多答問題だよね。

123-4-5-6-7+8-9=100
123+45-67+8-9=100
12+3-4+5+67+8+9=100

341 名前:132人目の素数さん投稿日:2001/05/09(水) 07:24
>>338
ハァ?? それじゃうまくいかないだろ。


342 名前:がんばる君投稿日:2001/05/09(水) 07:48
>>335
全通り発見したい!
1+...+9=45なので確実に2桁以上の数(連結)が必要。

■■(A)連結が1ヵ所の場合
左辺を偶数にするためには(奇,偶)の連結しか許されない。
また、
1+2+3+4+56+7+8+9=90
であるので連結は(7,8)に限られる。
さて、
1+2+3+4+5+6+78+9=108
であるので左辺を100にするパターンは2通り、
4または(1,3)の符号をマイナスに変える方法である。
以下、これを
1+2+3+4+5+6+78+9=108・・・~[4],~[1,3]
と表現する。

343 名前:がんばる君投稿日:2001/05/09(水) 07:49
■■(B)連結が2ヵ所の場合(3桁になるパターンを除く)
左辺を偶数にするためには(奇,偶),(偶,奇)各一回づつの連結しか許されない。
以下最小連結部別に調査。

●1,2連結
12+3+45+6+7+8+9=90・・・×(;左辺のどの項の符号を-に変えても100にならない)
12+3+4+5+67+8+9=108・・・~[4]
12+3+4+5+6+7+89=126・・・~[3,4,6],~[6,7]
●2,3連結
1+23+4+56+7+8+9=108・・・~[4]
1+23+4+5+6+78+9=126・・・~[4,9]
●3,4連結
1+2+34+5+67+8+9=126・・・~[5,8]
1+2+34+5+6+7+89=144・・・×
●4,5連結
1+2+3+45+6+78+9=144・・・×
●5,6連結
1+2+3+4+56+7+89=162・・・×

344 名前:132人目の素数さん投稿日:2001/05/09(水) 07:49
■■(C)連結が3ヵ所の場合(3桁になるパターンを除く)
左辺を偶数にするためには
(奇,偶)一回(偶,奇)二回 もしくは (奇,偶)三回の連結しか許されない

●(奇,偶)一回(偶,奇)二回
12+3+45+67+8+9=144・・・×
12+3+45+6+7+89=162・・・×
12+3+4+5+67+89=180・・・×
1+23+45+6+78+9=162・・・×
1+23+4+56+7+89=180・・・×
1+2+34+5+67+89=198・・・×
●(奇,偶)三回
12+34+56+7+8+9=126・・・×
12+34+5+6+78+9=144・・・×
12+3+4+56+78+9=162・・・×
1+2+34+56+78+9=180・・・×

■■(D)連結が4ヵ所の場合(3桁になるパターンを除く)
左辺を偶数にするためには
(奇,偶)一回(偶,奇)三回、(奇,偶)三回(偶,奇)一回の連結パターン。

12+34+56+7+89=198・・・×
12+3+45+67+89=216・・・×

345 名前:がんばる君投稿日:2001/05/09(水) 07:51
■■(D)連結が4ヵ所の場合(3桁になるパターンを除く)
左辺を偶数にするためには
(奇,偶)一回(偶,奇)三回、(奇,偶)三回(偶,奇)一回の連結パターン。

12+34+56+7+89=198・・・×
12+3+45+67+89=216・・・×

■■(E)3桁連結があり、それが"123"である場合
左辺を偶数にするために123以外の連結は
・(偶,奇)のみ数回
・(奇,偶)のみ二回
・連結部を持たない
である必要がある。
また、3桁連結が複数出現するパターンは無理。

●(偶,奇)のみ一回〜三回
123+45+6+7+8+9=198・・・×
123+4+5+67+8+9=216・・・×
123+4+5+6+7+89=234・・・×
123+45+67+8+9=252・・・~[67,9]
123+45+6+7+89=270・・・×
123+45+67+89=324・・・~[45,67]
●(奇,偶)のみ二回、連結部を持たない
123+4+56+78+9=270・・・×
123+4+5+6+7+8+9=162・・・~[4,5,6,7,9]

346 名前:がんばる君投稿日:2001/05/09(水) 07:51
■■(F)3桁連結があり、それが"123"以外である場合

●3桁連結部が234である場合
左辺を偶数にするための条件は
(偶,奇)のみ数回 または (奇,偶)のみ二回である
1+234+5+67+8+9=324・・・×
1+234+5+67+89=396・・・×
1+234+5+6+7+89=342・・・×
1+234+56+78+9=378・・・×

●3桁連結部が345以上である場合
無理。

■■(G)4桁以上の連結部がある場合
もっと無理。

結論;求める数式は11パターンである
1+2+3-4+5+6+78+9=100
-1+2-3+4+5+6+78+9=100
12+3-4+5+67+8+9=100
12-3-4+5-6+7+89=100
12+3+4+5-6-7+89=100
1+23-4+56+7+8+9=100
1+23-4+5+6+78-9=100
1+2+34-5+67-8+9=100
123+45-67+8-9=100
123-45-67+89=100
123-4-5-6-7+8-9=100

(゚д゚)ウマー

347 名前:わーい ヽ(´ー`)ノ投稿日:2001/05/09(水) 10:38
>>340
そう、多答問題。正解です。

>>342
すっすごい・・・。
さすが、がんばる君。名前だけじゃないな。

ちなみに正解は11通りですべて正解。パーフェクト。
1500マターリあげます。

348 名前:がんばる君投稿日:2001/05/09(水) 12:22
>>347
ゎ-ぃゎ-ぃ

模範解答などあればおせーて。
>>344なんて全部×だよ。。。。調べる必要のない部分なのかな?

349 名前:338投稿日:2001/05/09(水) 23:49
>>341
なんで? 書き方がわかりにくかったのかな・・・。
主張していることは次の2つ。

(1) 4枚の金貨(本物か偽物か不明)と、1枚の本物があるとき。
天秤2回で、偽物を見分けそれが重いか軽いかわかる。

(2) 12枚のとき、一回天秤を使うことで (1) の問題に還元できる。

350 名前:341じゃないけど投稿日:2001/05/10(木) 16:34
>>349
(1)はどうやって分かるの??詳細説明きぼんぬ。

351 名前:341?投稿日:2001/05/10(木) 16:45
>>350
4枚のうちの2枚を1個ずつ乗せてつりあったら次の2枚。
これでわかるかと。


352 名前:132人目の素数さん投稿日:2001/05/10(木) 16:53
>>351
偽者が重いか軽いかわかんないのに、
2枚天秤で量っても偽者は特定できないでしょ?

353 名前:132人目の素数さん投稿日:2001/05/10(木) 17:14
>>351,352
338は「本物とわかってるのが1枚あれば」って断わってるよ。

354 名前:132人目の素数さん投稿日:2001/05/10(木) 18:21
>>353
だから、「本物と分かってるのが1枚」あった場合でも、
特定できないじゃない?
手順教えて。

355 名前:132人目の素数さん投稿日:2001/05/10(木) 20:39
>>354
http://cheese.2ch.net/test/read.cgi?bbs=math&key=967889985&st=76&to=76&nofirst=true

356 名前:132人目の素数さん投稿日:2001/05/10(木) 21:11
>>354
だから>>338>>339を読みなさいって。

357 名前:354投稿日:2001/05/10(木) 21:18
>>355
あっ、ほんとだね。
失礼しました。勉強なりました。
ありがとう。

358 名前:ゲボ投稿日:2001/05/11(金) 00:24
298、やらせていただきます。
299より
(10001−1)^n+・・・(10001−1)^2+10001
二項展開すると、mを自然数として,つぎのようにあらわせる。
10001m+10001+(1−1+1・・・・)
ここで、
nが奇数のとき,1−1+1−・・・1−1=0
 よって、10001m+10001は10001の倍数だから、素数ではない。
nが偶数のとき,1−1+1・・・・+1=1
 よって、与式=10001m+10002
10001と10002は公約数をもたないから、これは素数である。

ところで、10001と10002の公約数ってないですよね(自信なし)。
見つけようとしてがんばっても、見つからなかったもので。
公約数があれば、どんなnにたいしても素数ではないことがいえるのですが。

359 名前:132人目の素数さん投稿日:2001/05/11(金) 00:29
>>358
>nが偶数のとき,1−1+1・・・・+1=1
> よって、与式=10001m+10002
>10001と10002は公約数をもたないから、これは素数である。

        ∫
   ∧,,∧ ∬      / ̄ ̄ ̄ ̄ ̄
   ミ,,゚Д゚ノ,っ━~  <  ハァ?
_と~,,, M ~,,ノ___. ∀  .\_____
    .ミ,,,/~),  | ┷┳━
 ̄ ̄ ̄ .し'J ̄ ̄|.. ┃
 ̄ ̄ ̄ ̄ ̄ ̄ ̄   ┻

360 名前:ゲボ投稿日:2001/05/11(金) 00:47
359
もっ、申しわけない。mと10002が公約数をもっていれば素数にはなりませんね。

361 名前:132人目の素数さん投稿日:2001/05/11(金) 04:10
>>298

>次の無限数列には素数の項が存在しないことを示せ。
> 10001, 100010001, 1000100010001, ...

(1) 1が偶数回現れるとき
この場合は 10001 の倍数で、10001 も 73 で割り切れる。

(2) 1が奇数回現れるとき

題意の数は m を 1 以上の整数として

10^(2m*4) + … + 10^(2*4)+ 10^4 + 1

で表せる。これが 10^(2m) +…+ 10^2 + 10 + 1 で割り切れる
(例えば 10001000100010001 は 11111 で割り切れる)
ことを示すため、(3)の補題を用いる。

(3)x の多項式
x^(2m*4) + … + x^(2*4) +x^4 + 1
が x^(2m) + … + x^2 + x + 1 で割り切れることを示そう。

それぞれに(x-1)をかける事により
A(x) =(x^(2m*4) + … + x^(2*4) + x^4+1)(x-1)
が B(x)=x^(2m+1) - 1 で割り切れる事を示せばよい。

まず m が偶数2nであるときには
A(x) を展開して得た項を使って

x^(4n*4+1)−x^(4n*3) 、… 、x^(4n+1)−1

を得ることができ、これらは B(x)=x^(4n+1)−1で割り切れる。
残る項も -x^(4n*4) + x^(4n-3) 、…、-x^(4n*3+4) + x
のように組み合わせることができ、
それぞれが x^(4n*3+3)-1で割り切れ、
従って B(x) で割り切れる。

同様にmが奇数2n-1である場合も、A(x)を以下2行のように組み合わせ、
-----------------------------------------
-x^(4n*4-8)+x^(4n*3-7) 、… 、-x^(4n)+x
x^(4n*4-7)-x^(4n-4) 、… 、x^(4n*3-3)-1
-----------------------------------------


B(x)=x^(4n-1)−1で割り切れることが示せる。

(4) (3)の最初2行にある両多項式で x=10
とおいたのが、(2)の場合である。



362 名前:132人目の素数さん投稿日:2001/05/12(土) 00:22
なんでそんな面倒なことするの・・・。

与えられた数は、1+10^4+(10^4)^2+(10^4)^3+(10^4)^4+… という形。
和の公式より、{(10^4)^m-1}/(10^4-1) である(m=2,3,4,…)。

分子は、(10^4)^m-1=(10^2m)^2-1={(10^2m)-1}*{(10^2m)+1} と因数分解される。
与えられた数は整数なので、分母の 10^4-1 を、適当に 10^4-1=AB と分解し、

[{(10^2m)-1}/A]×[{(10^2m)+1}/B]

としたとき、それぞれが整数になるようにできる。
A, B は 10^4-1 以下なので、m≧3 ならば、どちらの因数も1ではない。
よって、m≧3 なら素数ではない。

m=2 について別に調べて証明終わり。

363 名前:361投稿日:2001/05/12(土) 06:13
(3)は、

x^(2m*4) + … + x^(2*4) +x^4 + 1
= [x^(2m*4+4) − 1]/[x^4 - 1]
= [(x*x)^(2m+1) + 1]/[x*x + 1] *
 [x^(2m+1) + 1]/[x + 1] *
 [x^(2m+1) − 1]/[x - 1]
(各項とも整除される)

で済む話でした。ちなみに、3項目が x^(2m) + … + x^2 + x + 1
になっています。

362さん
> なんでそんな面倒なことするの・・・。
いやまったく、何を考えていたのか。



364 名前:大一坊主投稿日:2001/05/13(日) 09:39
x^2 + y^2 = z^2
を満たす(x,y,z)の組を*すべて*求めよ。

365 名前:132人目の素数さん投稿日:2001/05/13(日) 10:21
 A B  
  □
 C D

の正方形の折り紙があるとします
そこから、どうやって最大辺の正三角形を作れますか
計算的じゃなくて、簡単に折って作れるようなのですが
また教えてください

366 名前:誰かひっかかれ投稿日:2001/05/13(日) 13:45
10mのひもがあります。
1秒毎に1m切っていくと何秒でひもが10本になる?

367 名前:132人目の素数さん投稿日:2001/05/13(日) 14:19
>>366
分かった!!10秒!

368 名前:中谷先生投稿日:2001/05/13(日) 15:37
>>367
だから君達は小学校2,3年程度の頭しか持ってないんだ!(ワラ

369 名前:132人目の素数さん投稿日:2001/05/13(日) 15:40
>>368
ネタニマジレスカコワルイ

370 名前:中谷先生投稿日:2001/05/13(日) 15:44
>>369
メール欄、見たか?(ワラ

371 名前:132人目の素数さん投稿日:2001/05/13(日) 16:18
>>370
オマエノカチ(ワラ

372 名前:132人目の素数さん投稿日:2001/05/13(日) 17:32
>>370
オマエノカチ(ワラ


373 名前:132人目の素数さん投稿日:2001/05/13(日) 22:29
どうしても分からない問題があります。
問題:ここに見た目は同じ玉が十二個ある。
   1つだけ偽物がありその重さは他と違う。
   重いか軽いかは分からない。
   天秤を三回だけ使って偽物を見つける方法を考えよ。

「とんち」とかではないようです。教えてください。

374 名前:132人目の素数さん投稿日:2001/05/13(日) 23:09
>>373
このスレちゃんと読んでから書け。アフォ。

375 名前:大一坊主投稿日:2001/05/14(月) 04:24
>>373
それたしか2000年のセンターの英語で出たんじゃなかった?

376 名前:132人目の素数さん投稿日:2001/05/14(月) 13:28
373の文章を英訳させるの?

377 名前:132人目の素数さん投稿日:2001/05/14(月) 17:09
「あるところに3人の男がいました。3人は10ドルずつ出して一部屋30ドル
の部屋に泊まりました。ところが翌朝ホテルのオーナーが実は宿泊代は
25ドルだったことに気付きボーイに5ドル返させました。ボーイは2ドル
着服して3人に1ドルづつ返しました。これで3人は9ドルづつ払ったことになるので払った金額は9×3で27ドルになります。しかしそれにボーイの2ドルを足しても29ドルにしかなりません。さて残りの1ドルは何処にいったでしょう?


378 名前:132人目の素数さん投稿日:2001/05/14(月) 17:46
>>377
>これで3人は9ドルづつ払ったことになるので
ここが落とし穴だね。
25ドルの部屋と仮定すると、一人8j33k(誰か
一人だけ34k)払ってるわけだから、ボーイが
2j着服して、あとは9j33kずつ払ってることに
なる。つまり、残りに1jの行き場所はホテル。

379 名前:132人目の素数さん投稿日:2001/05/14(月) 18:17
ゆうめいじゃん!!

380 名前:132人目の素数さん投稿日:2001/05/14(月) 19:22
>>378
男たちはそれぞれ10ドル払ってから1ドルかえってきたんだよ。
9ドルしか払ってないじゃん

381 名前:132人目の素数さん投稿日:2001/05/14(月) 19:32
客が27ドル払って
宿に25ドルいって
ボーイに2ドルいった

最後に27に2を足してるところがひっかけ
残りの1ドルとか関係ない

382 名前:132人目の素数さん投稿日:2001/05/14(月) 20:45
A君のホームページは1日に40件のアクセスがあります。
開設3日目の時点で、URLが2ちゃんねるに載せられてしまいました。
4日目から、2ちゃんねらーが毎日200人ずつ荒らしにくるようになりました。
アクセスカウンタが5000を越えた時点で、このHPは存続不可能になるとします。
A君のHPが閉鎖されるのは、開設から何日目でしょうか。
ただし、A君は毎日2回HPを更新するとします。

383 名前:378投稿日:2001/05/14(月) 21:01
>>380
あわわわ、引っかかってる...
>>381の言うとおり、何処へ行くも
なにも関係のない1ドルだった...
くそぉ、くやしいぞ

384 名前:132人目の素数さん投稿日:2001/05/14(月) 22:08
>>382
サイト更新時に毎回アクセスカウンタを0にする。永久に存続。

385 名前:384投稿日:2001/05/14(月) 22:21
ちょっと違うか。。

サイト更新時に毎回アクセスカウンタをいじれるので
存続はA君の胸先三寸。

386 名前:腐乱平太投稿日:2001/05/14(月) 22:57
高度な数学はほとんど使わない問題(多分、小学生でもいけると思う・・・)

1辺の長さが2の正三角形があります。この正三角形の内部に任意に5点を
とると、それらの点のうちお互いの距離が1以下となるような点の組が少なく
とも1組は存在することを証明してください。




387 名前:y^2=x^3+ax+b投稿日:2001/05/14(月) 23:26
>>386
正三角形に補助線をひいて小正三角形を4つつくる
 △
△△
2点間の距離が1より大きくなるように点を取っていく場合
おなじ小三角形内に2点はとれないので、全ての点の間の
距離が1より大きくなるように点を取っていくと
4点目まで点をうつった時点で4つの小三角形全てに点が打たれ、
5点目が打てない
よって全ての点の間の距離が1より大きくなるように点を取ることは
できないので、必ず距離が1以下となる点が存在する

388 名前:腐乱平太投稿日:2001/05/15(火) 07:07
>>387

正解です。鳩ノ巣原理ってやつですね。私はこの問題、初見で解けま
せんでした。この手の問題の是非ってどうなんでしょうね?この解法
を「思いつく」ことが数学的センスなのでしょうか?あまりにコロン
ブスの卵って感じですよね。なんか数学的センスというよりは何かを
発明するときのひらめきに近いような気がしますが・・・。



389 名前:大一坊主投稿日:2001/05/15(火) 07:36
黒玉と白玉を合わせて12個使って数珠を作る。
数珠は何通り作れるか。
ただし、全部が黒玉だったり、全部が白玉だったりしてもかまわない。

私はこの問題を高三の2月の上旬に思いついた。
数珠順列の基本例題を作ってみようと思ったのがきっかけである。
だが少し考えてみると、これはとんでもない難問であることに
気づいた。私程度の学力ではとうていまともに解けず、3〜4日くらいを
この問題だけのために費やした。(いいのか?)理論と計算
(コンピュータで無理矢理数えた。しかも芋アルゴリズムで。)
が一致したときの喜びは忘れられない。

と言うわけで、誰かやってみないか?

390 名前:132人目の素数さん投稿日:2001/05/15(火) 08:59
>>389 せめて5〜6個にしない?

391 名前:132人目の素数さん投稿日:2001/05/15(火) 11:44
>>389
足し算まちがってなければ 352 かな?あってっかな〜?

392 名前:391投稿日:2001/05/15(火) 11:59
>>389
ごめん。もしかしてこれ反転もかんがえんといかんの?
だったらもっとすくないんだね。やりなおします。

393 名前:132人目の素数さん投稿日:2001/05/15(火) 13:03
>>389がありならこんなんあり?
正12面体を黒3色白9色でぬりわけるぬりわけかたはいくつ?
たしか以外に少なかった気がする。

394 名前:132人目の素数さん投稿日:2001/05/15(火) 18:30
問題

 ある点から,南へ1km進み,東へ1kmすすみ,北へ1km進むと,もとの
点に戻った。これは地球のどこか?


ヒント:ちょっと考えると北極と思いやすいが,違う。
    (なぜなら北極には東西南北がないからだ。)


395 名前:132人目の素数さん投稿日:2001/05/15(火) 19:32
>>394
数学板は初めて?
あまりに概出問題なんで飽きれるよ (´ー`)ノ○ アンマーン

396 名前:132人目の素数さん投稿日:2001/05/15(火) 20:15
しかも言ってることが変

397 名前:132人目の素数さん投稿日:2001/05/15(火) 20:25
>>394
東西北はないが、南はあるぞ。一方向には定まらないが。

398 名前:>投稿日:2001/05/15(火) 21:07
一意でないにしろ南があるなら、
南をむいて右手、左手で東西もあることにはならない?
(もちろん 一意ではないが)

399 名前:132人目の素数さん投稿日:2001/05/15(火) 21:32
>>398
言葉の定義の問題になるんとちがうか?
あまり数学的な話ではないとおもう。

どっち向いても南だし、同時に東だし、同時に西だから。

400 名前:132人目の素数さん投稿日:2001/05/15(火) 22:59
>>398
ならない
南を向いて右手も左手も南

401 名前:397投稿日:2001/05/15(火) 23:01
そもそも南もあるかどうか分からなくなってきた。
方向ってなんだ?球の接線か?
北極においてはどのような規定で、東西南北を定める接線を引くんだ??

402 名前:132人目の素数さん投稿日:2001/05/16(水) 00:47

南極の近くで,緯周が1kmのところの1km北側。じゃないの?

403 名前:132人目の素数さん投稿日:2001/05/16(水) 02:10
>>401
それで当然です。
北極と南極は極座標の特異点だからね。
数学的に言えば、南極では(r,θ)の値が決まらない。
だからこのようなことが起きる。

南極と北極という2点をとおる平行な線はいくらでも
ひけることにも注意。

404 名前:132人目の素数さん投稿日:2001/05/16(水) 02:23
>>403
>南極と北極という2点をとおる平行な線はいくらでも
>ひけることにも注意。

この場合「平行な線」の定義は何ですか?

405 名前:大一坊主投稿日:2001/05/16(水) 08:15
>>392
回転と反転が絡み合って、いろいろと面白いことが起こる。
そこがポイント。

406 名前:大一坊主投稿日:2001/05/17(木) 05:42
>>393
黒の面が3つ固まっているとき:2通り
黒の面が2つ固まっていて、残り1つは孤立しているとき:2通り
黒の面が3つバラバラで互いに隣接しないとき:1通り
の合計5通り。

407 名前:132人目の素数さん投稿日:2001/05/17(木) 06:12
>>406
すごいね。正解。でもこうやるともそっとらく。
全配色=C(12,3)=220,12面体群の大きさ=60
不変群が3次巡回群の配色=40
そうでない配色=不変群が単位群=220-40=180
∴orbitの大きさ=(180*1+40*3)/60=5

408 名前:132人目の素数さん投稿日:2001/05/18(金) 10:50
age

409 名前:132人目の素数さん投稿日:2001/05/18(金) 16:58
問題!!!!!!!!!!!


  16畳の長方形の部屋がある。この部屋の対角をそれぞれ半畳ずつ切り抜いて
 置物をおかねばならない。のこりの15畳に畳を隙間無くひくことができるか?
 理由を付けて答えよ!

410 名前:132人目の素数さん投稿日:2001/05/18(金) 17:01
16畳の正方形の部屋なら、市松模様に塗った場合の同色二つを置物が占有してしまうから無理…ってので有名だけどね。

411 名前:名無しさん投稿日:2001/05/18(金) 17:17
>>410
16畳の正方形の部屋?
なにそれ?

412 名前:132人目の素数さん投稿日:2001/05/18(金) 17:23
>>411
ごめん。半畳を16枚だった。

413 名前:132人目の素数さん投稿日:2001/05/18(金) 17:31
問題!!!!!!!!!!!!!!!!!!!!!!


  あるトラックは全工程のはじめの半分を30km/hで走った。全
 体での平均速度を60km/hにするには何km/hで残りの半分を走れば
 良いか。

414 名前:132人目の素数さん投稿日:2001/05/18(金) 17:46
問題

   4本の連続した直線ですべての点を結んでください。
  ただし途中で引き返したり同じ点を二度通ってはいけない。

      ・   ・   ・
     
     
      ・   ・   ・


      ・   ・   ・

415 名前:132人目の素数さん投稿日:2001/05/18(金) 17:51
>>413
時速150キロで目的地にたどり着いてから、余った時間待機していれば平均60km/hになる。

416 名前:132人目の素数さん投稿日:2001/05/18(金) 17:59
>>414
できたけど書けない。
ちなみにはみだすよ。

417 名前:132人目の素数さん投稿日:2001/05/18(金) 18:11
>>413
全工程を60kmとすると,半分までで一時間かかってしまうから、
絶対無理じゃない!

418 名前:>414投稿日:2001/05/18(金) 18:12
超ユーメーな問題
”傘”
点に面積があれば3本でOK
"Z"

419 名前:132人目の素数さん投稿日:2001/05/18(金) 18:12
   ↑
絶対無理と言う意味です!

420 名前:>419投稿日:2001/05/18(金) 18:21
じゃあ ”乙”

421 名前:132人目の素数さん投稿日:2001/05/18(金) 18:24
問題

 1〜100までの整数の中で9という数字は何回出てくるでしょう。
   

422 名前:名無しさん投稿日:2001/05/18(金) 19:48
20???

423 名前:132人目の素数さん投稿日:2001/05/18(金) 20:49
>>414
http://cheese.2ch.net/test/read.cgi?bbs=math&key=967889985&st=105&to=105&nofirst=true

424 名前:132人目の素数さん投稿日:2001/05/18(金) 21:13
>16畳の長方形の部屋がある。この部屋の対角をそれぞれ半畳ずつ切り抜いて
>置物をおかねばならない。のこりの15畳に畳を隙間無くひくことができるか?
>理由を付けて答えよ!

★□■□■□■□
□■□■□■□■
■□■□■□■□
□■□■□■□★

★□■□■□■□■□■□■□■□
□■□■□■□■□■□■□■□★   白と黒の数が違うから無理

425 名前:132人目の素数さん投稿日:2001/05/18(金) 22:54
問題

  10本の木を5列に植えしかも1列には4本の木が植わっている
 ようにしたい。どう植えるか?


426 名前:132人目の素数さん投稿日:2001/05/18(金) 22:59
問題

  天秤を使って1〜40gまで何gでも測れるようにするには最低何
 個の重りがあれば良いか。またそれは何gか?


427 名前:1,3,9,27投稿日:2001/05/19(土) 00:17
>>426

428 名前:星?投稿日:2001/05/19(土) 00:47
>>425

429 名前:132人目の素数さん投稿日:2001/05/19(土) 08:47
>>425
下の図の4×5の桝目で、
■の中心に木を植え、
□のところは空ける。

■□□□
□■□■
■□■□
■■■■
■□□□

>>426
1、2、4、8、16、32の五つ

もっとひねったやつプリーズ


430 名前:132人目の素数さん投稿日:2001/05/19(土) 09:29
>>429
逝ってよし

431 名前:>429投稿日:2001/05/19(土) 11:28
>1、2、4、8、16、32
2進数表現でこれをかんがえたんだろうけど
(各桁の重み係数 0,1って具合)
もっといい解がある

天秤だから むこうに3gコッチに1gの分銅を載せることで
2gをはかることができる。
(各桁重みの係数として -1,0,1 を3値をとることができる。)
この発想でいけば
427
のいうように3進数を考えることができ
1,3,9,27 でよい。


427が答えって気づいてなかった?

432 名前:>429投稿日:2001/05/19(土) 12:25
■□□□
□■□■
■□■□
■■■■
■□□□

各列木が4本って条件をどうみたしているんだ?

これも 428が答えを示唆しているのだが
きづいてないのかな

433 名前:>投稿日:2001/05/19(土) 12:31
425のバリエーション

 4本の木を6列に植えしかも1列には2本の木が植わっている
 ようにしたい。どう植えるか?

428の答えを聞いたあとならえらく簡単だが、

434 名前:132人目の素数さん投稿日:2001/05/19(土) 12:40
>>433
ぴらみっど/2


435 名前:>434投稿日:2001/05/19(土) 13:07
いやむしろテトラパック

436 名前:132人目の素数さん投稿日:2001/05/19(土) 15:42
0から9の数字を1回ずつと、+−×÷を使って
1000000を作れ。
数字は繋いで2桁以上の数にしてもよい。

437 名前:132人目の素数さん投稿日:2001/05/19(土) 16:05
>>429
ネタだろ?
ネタだと言ってくれ、頼む。

438 名前:132人目の素数さん投稿日:2001/05/19(土) 17:56
>>425

     五角形の対角線の交点上じゃない? 

439 名前:132人目の素数さん投稿日:2001/05/19(土) 18:08
問題

  4枚のカードがあります。このカードは片面が赤か緑で,反対の面
 には丸か四角が書いてあり,つぎのようにテーブルに並べてあります。


     赤 | 緑 | ○ | □

全ての赤いカードの裏には四角が書かれているか?
 と言う問いに答えるには最低どのカードをめくらなくてはならないか? 

440 名前:132人目の素数さん投稿日:2001/05/19(土) 18:51
>>439
緑&刺客不要

441 名前:132人目の素数さん投稿日:2001/05/19(土) 20:22
>>439
心理学だっけ?
確か現実的な問題にすると正答率が上がるとか。

442 名前:132人目の素数さん投稿日:2001/05/19(土) 22:42
論理学でしょ? とりあえず>>440(答えを理解してないと文章の真意が読み取れないが)に一票

443 名前:132人目の素数さん投稿日:2001/05/19(土) 23:02
>>439
3

444 名前:132人目の素数さん投稿日:2001/05/19(土) 23:28
>>439
我輩も>>440に8000マターリ。てか数学ちゃんと勉強した人間には
ほぼ自明でないの?しかもこれわかんない人間をなっとくさせんの
むづかしそう。

445 名前:132人目の素数さん投稿日:2001/05/20(日) 00:37
>>442
確かに内容は論理学だが心理学で見たはず。
ノーチェックで赤と□を選んでしまう傾向が強いらしい。
正答率は10%程度という話。
あくまでも一般人を対象にした実験らしいが。
そして話を現実的にする(封筒とハガキと切手などに置き換える)
と正答率がアップするという話。

446 名前:132人目の素数さん投稿日:2001/05/20(日) 02:31
週末のひまつぶしにどうぞ。
n枚のカードのなかのk枚にマークがしてある。マークしてあるカード
をひきあてるまでカードをもどさずにひきつづける。このとき
マークしてあるカードをひくまでにかかった回数をあたえる
確率変数をXとする。Xの“分散”をもとめよ。

447 名前:132人目の素数さん投稿日:2001/05/20(日) 04:13
>>439
「赤のカードと○のカード」
でおっけーですか?
>>440の言いたいことが分らないけど)

448 名前:132人目の素数さん投稿日:2001/05/20(日) 05:17
>>447
おっけー

     「赤と○(の裏)を見る必要がある」
 ⇔ 「緑と□(の裏)はどうでもいい」
 ⇔ 「緑&刺客不要」

449 名前:132人目の素数さん投稿日:2001/05/22(火) 09:52
あげ

450 名前:132人目の素数さん投稿日:2001/05/22(火) 09:53
さげちゃった。あげなおし。

451 名前:132人目の素数さん投稿日:2001/05/22(火) 18:37
面白あげ

452 名前:132人目の素数さん投稿日:2001/05/23(水) 03:11
なんでシカクの裏を見る必要がないのか
わからん

453 名前:132人目の素数さん投稿日:2001/05/23(水) 05:02
>>452
問題文をよく読め、としか言いようが無い。

454 名前:132人目の素数さん投稿日:2001/05/23(水) 09:48
>>452
赤でも緑でもいいじゃん。

455 名前:132人目の素数さん投稿日:2001/05/23(水) 11:16
葉書 封筒 50 80とする 但し数字は切手の額面とする
全ての封筒に80切手が貼られていることを
確かめるにはどれを裏返せばいいか

456 名前:132人目の素数さん投稿日:2001/05/23(水) 12:05
でも赤めくって○だったらその時点で否定できるよね
問題の趣旨はわかるけど聞き方はちょっと微妙じゃないか?

457 名前:132人目の素数さん投稿日:2001/05/23(水) 14:43
>>456
そういうことか
「『全ての赤いカードの裏には四角が書かれている』
という事が真であることを確かめるためには
最低何枚のカードをめくらなければならないか」
とすればいいということか。

458 名前:132人目の素数さん投稿日:2001/05/23(水) 23:38
大きさが同じであるn個の立方体を用意する。
各立方体は一つずつ点が適当な面の適当な場所にうってあり、
それ以外は真っ白であるとする。

そしてこれらn個の立方体を平面の上に適当に置く。
また、平面、もしくは平面より上のm個の点から
(各点においてその点の場所からならどこの方向を見てもよいとする)
これらのn個の立方体を眺めて、それぞれの立方体の
どの場所に点がうってあるか確かめるとする。
(ただしn個の立方体は互いに触れ合っておらず、
 どんな視点から見ても点を確認できない場合は
 平面と隣接している面に点がうってあるとして
 確認できるとする。)

このとき、どのように平面上にn個の立方体を配置したとしても
mがf(n)以下に出来るf(n)を求めよ。

…問題文分かりにくくてごめん。
f(1)=2だし、f(2)=3じゃないかと思うんだけど
それ以上はもう駄目(ToT)無理。分からん。死ぬ。いっそ殺して…

正四面体や球体の方が楽かも、と考えたけど、そっちのほうが難しそう。
…平面上じゃなくて空間上の適当な場所に配置するとさらに死ねる。

459 名前:132人目の素数さん投稿日:2001/05/23(水) 23:56
>>458
こたえもってんの?

460 名前:132人目の素数さん投稿日:2001/05/24(木) 00:06
平行四辺形の一辺の中点を
定規を使って(点と点をむすぶ事だけができる)
求めよ
(1)補助線はどこに引いても良い
(2)補助線は平行四辺形の中だけ
面白いはずなのでやってみてください

461 名前:132人目の素数さん投稿日:2001/05/24(木) 00:15
>>460
対角線だけ引いて終わりなのでは?

462 名前:132人目の素数さん投稿日:2001/05/24(木) 00:17
>>460
おめ、まさかさくらスレの667じゃあるまいな。

463 名前:132人目の素数さん投稿日:2001/05/24(木) 00:17
定規は点と点を結ぶ事だけしか出来ない
って事になってます
>>461

464 名前:461投稿日:2001/05/24(木) 00:27
>>463
うん、だから平行四辺形の頂点を結ぶ。
それ以上の進展はなし。
不可能問題。

465 名前:132人目の素数さん投稿日:2001/05/24(木) 00:56
ゴメン、>>139答えの意味がぜんぜん分からないんだけど・・
□は何で要らないの?解説きぼん。

466 名前:132人目の素数さん投稿日:2001/05/24(木) 01:08
>>465
赤の裏がかならず□
⇔赤の裏が○のやつがない。
⇔赤と○のくっついてるやつがない。
かどうかをcheckするんだから赤と○のうらだけ確認すればOK

467 名前:465投稿日:2001/05/24(木) 01:20
あ、139じゃなくて439だね。スマソ。

>>466
ううーん、まだ分からん・・(厨房
□の裏の色はどっちでも良いの??

468 名前:132人目の素数さん投稿日:2001/05/24(木) 01:27
>>459
なんかスレ違いみたいですね。
でも、あっちのスレにも余り合わない気がしますし、この問題は
無かった、ということで(^^ゞ

469 名前:132人目の素数さん投稿日:2001/05/24(木) 01:27
>>467
どっちでもいい。
もし□のうらめくって
(i)赤だったとする。
“ほ〜らやっぱ赤の裏は□じゃん。”
(ii)緑だったとする。
“へ〜。緑の裏でも□が書いてあるのがあるんだ。”
どっちゃでもえ〜でしょ?

470 名前:465投稿日:2001/05/24(木) 01:34
>>469
あ、そうか!やっと分かりました・・。
なんか頭おかしくなりそう(^^;

471 名前:463投稿日:2001/05/24(木) 13:16
そうです、マルチゴメソ>>462
ほんとに答えあります
どっかの大学入試に出た問題らしいです>>464

472 名前:132人目の素数さん投稿日:2001/05/24(木) 13:27
>>471
や〜っぱりね。まっ、問題おもしろかったからもういいけど。
たしかに答えあるね。おれの答えけっこう複雑だけど
入試問題になるぐらいだからもっとスキっとできんのかな?
まあ、もういいや。

473 名前:132人目の素数さん投稿日:2001/05/24(木) 13:35
こたえおしえてください>>472

474 名前:132人目の素数さん投稿日:2001/05/24(木) 14:13
>>473
長いのでSketchだけ。
まず次を用意する。
補題 平行四辺形ABCDとAB上の点PがあたえられたときBC,CD,DA上の
   点QRSを平行四辺形になるように作図できる。
∵) AC,BDの交点をXとするときAXとBCの交点をQとするとよい。以下略
そこで本題。まずあたえられたABCDのAB上に適当にPをとりQRSを補題
からとる。ABCDの対角線の交点をOとし、PQRSの辺の交点を順にHKLMと
する。ただし、HKLMはOA,OB,OC,OD上にあるとする。平行四辺形RSHL
と点Mに対し再び補題を適用してMUVWをつくる。このときV=OでUは
SHの中点になる。AUとSRの交点をXとするとASXHは平行四辺形になり
とくにHX//AD。HXとAB,CDの交点をE,FとするときAF,DEの交点とOを
むすぶ直線がもとめる線分。

475 名前:132人目の素数さん投稿日:2001/05/24(木) 15:22
>>474
おっとちょっとまちがった。補題は
補題 平行四辺形ABCDとAB上の点PがあたえられたときBC,CD,DA上の
   点QRSを平行四辺形でかつ各辺がABCDの対角線で2分される
   ように作図できる。
でないといかん。証明はそのまま成立してる。

476 名前:132人目の素数さん投稿日:2001/05/24(木) 15:50
なるほど、ありがとう>>475

477 名前:132人目の素数さん投稿日:2001/05/24(木) 20:22
>>303
大昔の話みたいだけど、正解が出ていないようなので。
俺なら、1ドルと1セント(でいいかな?とにかく通貨の最小単位で)。
相手が1ドル未満なら自分が儲かる。
相手が1ドル2セント以上だしてきたら、1ドル1セントもらって相手に99セント渡す。
相手が1ドル、および1ドル1セントつけた場合のみ引き分け。
とゆーことで、ただ1ドルつけるよりは有利に持ってけるよ。

数学つーか、頭の体操?


478 名前:477投稿日:2001/05/24(木) 20:25
ああっ、>>313で答えてる人がいた。
鬱だ....
でも根拠示したから許してsage

479 名前:132人目の素数さん投稿日:2001/05/24(木) 21:14
始めまして。簡単だと思いますが1問。
異なる4つの数字からなる、9000未満の4桁の自然数のすべてを
Aとする。
(1)Aはいくつあるか。
(2)Aを小さいほうから順に並べた時、
2015番目の数と、2018番目の数の和を求めよ。
ある雑誌からのぬきだしです。
ちょっと面白いかと思ったんですが、どうでしょう?

480 名前:佳奈りんご投稿日:2001/05/24(木) 21:57
>>479
(1)
8×9×8×7=4032
(2)
5000以下・以上それぞれ、2016個ずつなので
5000以下の数で条件を満たすのは、
4987/4986/4985/・・・
なので2015番目は4986
5000以上の数で条件を満たすのは
5012/5013/5014/・・・
より、2018番目は5013
従って4986+5013=9999

481 名前:132人目の素数さん投稿日:2001/05/24(木) 22:23
>480 お見事です。簡単だったかな?次も同じ雑誌から。
解答知らないんですけど。
1から6までのカードを使って、X君とY君は次のゲームをする。
@まず、X君6枚のカードからでたらめに同時に3枚選び、
その3枚の数を小さい順にa,b,cとして、a〜cの範囲を、
X君の陣地とする。
A次にY君が残りの3枚から1枚を選び、そのカードの数をyとする。
ByがX君の陣地内にある(a≦y≦c)ならばY君はX君に、
捕らえられたということでX君の勝ち、陣地外ならY君の勝ちである、。
(1)このゲームはどちらが勝つ確率が大きいか?
(2)@の”同時に3枚選ぶ”を、”1枚ずつ元に戻しつつ、
3枚選ぶ”に変え、Aの”3枚”を”6枚”に変えると、
どちらが勝つ確率が大きいか。




482 名前:佳奈りんご投稿日:2001/05/24(木) 22:49
(1)とられる4枚のカードが1234とする
   ここでXが勝つのはXが124、134をとるとき
   またXが負けるのはXが123,234をとるとき
  4枚のカードがどうなっても同様なので
  XYそれぞれの勝つ確率は1/2づつ
  よって同じ
(2)Yの勝つ確率を求める
   X、111の時Yが2−6で勝利よって
   (1/216)×(5/6)=5/(6^4)
   X、222〜666のときは同じ
  
   X、1と2のみの時(4/(6^3))
   以下1−3、(6/(6^3))
     1−4、(6/(6^3))
     1−5、(4/(6^3))の時を考える
   Yの勝つ確率は
   (5+4×5+6×4+6×3+4×2)/(6^3)
   =75/216=25/72<1/2
   よってXの方が有利。

483 名前:479投稿日:2001/05/24(木) 23:02
佳奈りんごさんすごいですね。
回答が早い。よく探したら解答見つかりました。
お見事!!

484 名前:132人目の素数さん投稿日:2001/05/25(金) 00:06
一辺の長さが10cmの正方形があり、
その中に半径5cmの円と、半径10cmの扇形がある。
この2つの共通部分の面積を求めなさい。

積分したけど無理でした。。
だれかおねがいします。

485 名前:132人目の素数さん投稿日:2001/05/25(金) 00:17
>>484
無理じゃない。どーたらArcsinこーたら。

つーか既出。単位と数値が同じ。
yahooとここにスレ立てた奴か?
未解決だと思ってんのは君だけ。

486 名前:132人目の素数さん投稿日:2001/05/25(金) 00:30
すいません、対角線の数の求め方教えてください。忘れてしまって。

487 名前:132人目の素数さん投稿日:2001/05/25(金) 01:49
>>486
忘れたってさぁ
覚えるもんじゃないよ
その場で出せるものじゃん

488 名前:132人目の素数さん投稿日:2001/05/25(金) 19:59
がいしゅつかもしれないが一問
□を埋めよ
1,4,7,□,2,5,8,□,3,6,9
有名かな?

489 名前:488投稿日:2001/05/25(金) 20:01
スマソ、出題ミスった
1,4,7,□,2,5,8,0,□,3,6,9

490 名前:唐風投稿日:2001/05/25(金) 23:18
通りすがりの者ですが。
問題
英国にある三階建てのアパートでの話です。
そのアパートの住人は全部で33人です。
そのうち英語を読み書き出来ない人の割合はちょうど50%でした。
1階に住んでいる13人は全員読み書きできます。
2階の住人の読み書きが出来る人の割合は70%でした。
ある日のことそのアパートにあたらしく3人の人が入居して、
1階・2階・3階に入りました。
2階に住む人の割合は80%にかわりました。
はたしてそのアパートの三階の住人の読み書き出来る人の割合は
一体いくらになったのでしょう?

491 名前:132人目の素数さん投稿日:2001/05/26(土) 00:10
>>490
◆ わからない問題はここに書いてね 7 ◆
http://cheese.2ch.net/test/read.cgi?bbs=math&key=988952592
でがいしゅつ

492 名前:132人目の素数さん投稿日:2001/05/26(土) 19:15
>485
すんません、既出でしたか、、
知りたいのでどこにあるか教えてくだせぃ。


493 名前:132人目の素数さん投稿日:2001/05/27(日) 12:37
ひ〜ま〜じゃ〜
なんかおもろいのない?

494 名前:132人目の素数さん投稿日:2001/05/27(日) 14:08
2を越える偶数は、2つの素数の和で書ける。

495 名前:132人目の素数さん投稿日:2001/05/27(日) 17:44
1.Σ[k=1,n](1/k)
2.Σ[K=1,n](1/k)^2

496 名前:おさかなくわえた名無しさん投稿日:2001/05/27(日) 17:54
*に#だ。

497 名前:興亡投稿日:2001/05/27(日) 21:25
>>494
ゴルドバッハの予想
>>495
for(int i=1;i<=n;i++){
result+=1/i;
}



498 名前:488投稿日:2001/05/27(日) 21:36
>>496
正解。

499 名前:132人目の素数さん投稿日:2001/05/27(日) 21:45
問)
「全ての生徒はペンを持っている。」の反対の意味を持つ文を書け。

500 名前:132人目の素数さん投稿日:2001/05/27(日) 21:58
「全ての生徒はペンを持っている。」ということはない。

501 名前:132人目の素数さん投稿日:2001/05/27(日) 22:01
「not『全ての生徒はペンを持っている。』」


502 名前:みや投稿日:2001/05/27(日) 22:23
>>499
すべての生徒の誰か1人がペンを持っている

503 名前:132人目の素数さん投稿日:2001/05/27(日) 22:28
>>502
日本語を勉強しなおした方がいい
チョンなのかもしれないが

504 名前:499投稿日:2001/05/27(日) 22:42
ちなみに、とある大学生100人にこの問題を出したところ、
98人が「全ての生徒はペンを持っていない。」と答えたらしい。

505 名前:名無しさんの初恋投稿日:2001/05/27(日) 23:35
「ペンを持っていない生徒はいない」………?
ごめん、通りすがりのドキュソ文系です……。
反対になっていないよな。
おじゃましました。

506 名前:132人目の素数さん投稿日:2001/05/27(日) 23:56
ペンを持っていない生徒がいる。

507 名前:132人目の素数さん投稿日:2001/05/28(月) 01:14
生徒のクセして赤ペン先生

508 名前:132人目の素数さん投稿日:2001/05/28(月) 06:08
「全ての生徒は筆を持っている。」

509 名前:132人目の素数さん投稿日:2001/05/28(月) 06:41
「『全ての生徒がペンを持っている』わけではない」

510 名前:132人目の素数さん投稿日:2001/05/28(月) 08:40
「全ての生徒はペンを持っている?」

511 名前:132人目の素数さん投稿日:2001/05/28(月) 10:43
悪いことする時はペンを隠せ。

512 名前:132人目の素数さん投稿日:2001/05/28(月) 12:51
英語的に考えれば出来るかも。
BothとEitherとNeitherみたいに。

「ある生徒はペンを持っていない。」



513 名前:132人目の素数さん♪投稿日:2001/05/28(月) 16:32
アニヲタのA君が合コンの主役(盛り上げ役)になる確率を求めよ。尚セッティングは
B君がしたものとする。

A君>アニヲタ    B君>真性ドキュソ   C君>灯台卒
D君>仮性ドキュソ

E子>某テレビ局のAD   F子>趣味は男アサリ
G子>主婦   Z子>バツイチ 

514 名前:499投稿日:2001/05/28(月) 16:49
>>512
正解。

515 名前:132人目の素数さん♪投稿日:2001/05/28(月) 17:11
ペンは全ての生徒に持たれている。

516 名前:132人目の素数さん投稿日:2001/05/28(月) 17:19
>>514
なんで正解?
部分否定と全否定どちらでもいいのでは?
あと515は違うのでは?
「全てのペンは生徒を持っている。」

517 名前:132人目の素数さん投稿日:2001/05/28(月) 21:07
尾も白い問題キボンヌ。

518 名前:132人目の素数さん投稿日:2001/05/28(月) 22:47
>>514
つーか>>512の前に正解がいくつかあると思われ(ワラ

519 名前:132人目の素数さん投稿日:2001/05/29(火) 16:57
>>506なんかお見事に正解じゃないか。


520 名前:132人目の素数さん投稿日:2001/05/29(火) 22:21
。るいてっ持をソペは徒生のて全


てか
>>504
「とある大学生100人」
は「任意の定数」みたいで変だ

521 名前:132人目は素数さん投稿日:2001/05/30(水) 01:07
ある小学校での運動会。クラス対抗の徒競走で4人が走りました。
次の情報から4人の順位を推理してください。

Aは転んだが、2人を追い越した。
Bはズボンの尻が破れているのをDを含む2人に見られた。
CはBに追い抜かれたが、Dを抜いた。
Dは一時トップにいた。

522 名前:132人目は素数さん投稿日:2001/05/30(水) 01:16
ウイスキーの水割りを作ろうとします。ウイスキーを先に入れておき、後で水を注ぎます。
水を入れ始めて2秒後にウイスキーが全体の50%なりました、
この調子で水を入れつづけるとウイスキーが全体の1パーセントになるのは、
水を入れ始めてから何秒後でしょうか。
尚グラスの大きさは無限大です。

523 名前:132人目は素数さん投稿日:2001/05/30(水) 01:18
五分計と七分計の砂時計を使って16分を計りたい、
どういった手順なら16分を計ることができるだろうか。

524 名前:132人目は素数さん投稿日:2001/05/30(水) 01:20
8×8のマス目に、正方形がいくつ隠されているか。

525 名前:132人目の素数さん投稿日:2001/05/30(水) 01:34
すげえな、問題ラッシュだ。
>>523
5分と7分を同時にスタート
5分が終わった時点で計時開始。
2分+7分+7分で16分
あってる?

526 名前:132人目の素数さん投稿日:2001/05/30(水) 01:38
>>522
198
>>524
204


527 名前:132人目の素数さん投稿日:2001/05/30(水) 02:37
スタートを自由にできない回答が出来るような問題作る奴は山形大にでも行ってろ

528 名前:132人目の素数さん投稿日:2001/05/30(水) 03:05
だいたい砂時計だけ使える状況ってどういうときよ?(ワラ

529 名前:132人目の素数さん投稿日:2001/05/30(水) 09:46
>>525
やっぱ使いはじめ=はかりはじめでないとだめでないの?
これそうゆう解あるよ。

530 名前:132人目は素数さん投稿日:2001/05/30(水) 16:25
>>525,526正解
523の問題は他にも、
5分計と7分計を同時にスタート
5分計が終わったらそれを逆転させる
7分計が終わったところで、それを逆転させ同時に5分計を横に倒す(五分計は2分と3分に分かれている)
7分計の2回目が終わったところで14分経過、倒しておいた五分計の残り2分をスタート。
残り2分が終わったところで終了。


531 名前:132人目は素数さん投稿日:2001/05/30(水) 16:33
100キロ離れて向かいあった電車がそれぞれ時速50キロで接近しています。
ハエが電車の間を時速70キロで行ったり来たりします(電車に遭うとUターン)
電車が100キロ離れた位置からこれをはじめると、
双方の電車がすれ違うまでに、ハエはどのくらいの距離を飛ぶのでしょうか?
ハエが折り返しにかかる時間はゼロとします。

532 名前:132人目は素数さん投稿日:2001/05/30(水) 16:37
カセットテープを巻き戻しています、端から端まで巻き戻していますが、
テープの全体の長さの半分まで巻いたところで時間を見ると1分かかっていました。
テーうの全体を巻く時間は何分でしょうか?

533 名前:132人目は素数さん投稿日:2001/05/30(水) 16:43
内法がそこの直径15センチ高さ20センチの円錐形の中に、
いくらかの水が入っています、容器の側面に水面の高さの印をつけました。
この容器を上下さかさまにすると、さっきの印と同じ位置に水面が来ています。
この印から頂点までの高さはいくらでしょうか。

534 名前:132人目の素数さん投稿日:2001/05/30(水) 21:12
531を解くのに、かのフォン・ノイマンは無限級数の和を計算したらしい

535 名前:132人目の素数さん投稿日:2001/05/30(水) 22:42
>>534
もちろん暗算で、でしょ。

536 名前:132人目の素数さん投稿日:2001/05/30(水) 23:38
>>531
フォン・ノイマンはあえてそれをやったのだろうか。
ハエは一時間飛んだんだから、70キロだろう。

537 名前:132人目の素数さん投稿日:2001/05/31(木) 00:12
>>536
あ、一時間か。
100/50で2時間だと思ってしまった俺は山形大に逝ってきます

538 名前:132人目の素数さん投稿日:2001/05/31(木) 00:29
>>532
どうしても2分としか思えない
HELP!

539 名前:132人目は素数さん投稿日:2001/05/31(木) 01:47
>>536 正解。
>>538 放物線がヒントです。

540 名前:>>539投稿日:2001/05/31(木) 09:39
等加速度運動かな?

541 名前:初恋の痛み投稿日:2001/05/31(木) 13:36
√2−1 分 ぐらいだと思う

542 名前:132人目は素数さん投稿日:2001/05/31(木) 15:56
>>540 その通り!

543 名前:132人目の素数さん投稿日:2001/05/31(木) 16:22
>>538
巻いた長さ|
     |
    2|    +
     | 
     | 
    1| +
     |
     +――――――――――――> 巻くのにかかった時間
       1  X

 半分の長さを1とすると全体は2、YはXの2乗に比例、
 テープが等加速度で巻き取られていることがわかれば簡単、以上!


544 名前:132人目の素数さん投稿日:2001/05/31(木) 16:35
>>532
これだけじゃ答え決まらないだろ。

545 名前:517投稿日:2001/05/31(木) 22:51
俺があれを書いてから問題がやたら増えたな(藁

546 名前:132人目の素数さん投稿日:2001/05/31(木) 23:08
ハエって時速70キロで飛ぶの?

547 名前:132人目の素数さん投稿日:2001/06/12(火) 05:46
想城異悶打忌ボシュウ!!

548 名前:132人目の素数さん投稿日:2001/06/12(火) 16:16
ひとつの部屋に何人かがいる。それらのうち少なくとも6人が知り合いであるか、または
少なくとも6人が知り合いではないことを保証する、最小の人数を求めよ

549 名前:132人目の素数さん投稿日:2001/06/13(水) 01:12
>>548
いわゆる未解決問題をもってきたな。
設問は言い換えると「R(6,6;2)のラムゼー数を求めよ」というものだが、
現在わかっているのは、1965年のKalbfleischの出した
102 <= R(6,6;2) <= 165

たしかポリアだったとおもうが、なんでも新しいラムゼー数をひとつ
特定すれば修士論文になり、いくつかのラムゼー数において共通
した性質について論文を書けば博士論文になる、といったような
話があったと思った(うろ覚え)。


550 名前:132人目の素数さん投稿日:2001/06/14(木) 20:01
あげ!

551 名前:132人目の素数さん投稿日:2001/06/14(木) 22:30
1,6,2,3,□,0,1,0,1,1,0,6,□、2,3,2,3
□に入る数字を求めよ

552 名前:ふぬけ投稿日:2001/06/17(日) 03:33
2^2=4, 2^3=8, 2^4=16 ..... と計算していった時、
最初に右辺が「9から始まる数字」になるのは何乗したときか?
簡単でしょう?

553 名前:132人目の素数さん投稿日:2001/06/17(日) 06:53
477381560501272576

554 名前:132人目の素数さん投稿日:2001/06/17(日) 13:38
2^53=9007199254740992

555 名前:132人目の素数さん投稿日:2001/06/18(月) 04:20
A,B,Cの3人が1本ずつくじを引きます。3本のうち1本があたりです。
Aは考えました。BかCのどちらかは必ずはずれなので自分の当たる確立は2分の1だ。
Aは数学者でしょうかそれとも精神病でしょうか?

556 名前:132人目の素数さん投稿日:2001/06/18(月) 04:23
>>555
少なくとも数学者は確率を確立とは書かない(ワラ

557 名前:ふぬけ投稿日:2001/06/18(月) 23:48
>>553
なんだこの数字は???円周率の一部か?完全数とかか?
>>554
俺は小学生の頃それを鉛筆で計算したんだよ。超肥満児。

558 名前:132人目の素数さん投稿日:2001/06/19(火) 02:03
>>555
少なくてもオマエは精神病(ワラ

559 名前:132人目の素数さん投稿日:2001/06/19(火) 02:58
一時間で燃え尽きる線香を
二本つかって四十五分を計りなさい。
ただし、線香を折ったりしてはいけません。
また、時間を計測する器具(ストップウォッチなど)も
使用は認めません。

↑友達がこんな問題だしてきたんですけど・・・。
わかる人、います?
砂時計の問題に似ていますけど・・・。

がいしゅつならスマソ。

560 名前:559投稿日:2001/06/19(火) 03:03
>>559
計りなさい×
測りなさい○

ごめん。

561 名前:132人目の素数さん投稿日:2001/06/19(火) 03:04
>>560
いや、合ってるよ。
計る、が正解。

562 名前:132人目の素数さん投稿日:2001/06/19(火) 03:07
計りなさい×
謀りなさい○

563 名前:559投稿日:2001/06/19(火) 03:07
>>561
ああ、すみません!辞書見ました!無能で申し訳ない・・・。
欝・・・。

564 名前:132人目の素数さん投稿日:2001/06/19(火) 03:10
で、誰かわかった人いる?
>>559

565 名前:132人目の素数さん投稿日:2001/06/19(火) 03:13
>>564
おまえも考えろよ。

566 名前:他所で見た投稿日:2001/06/19(火) 03:58
>559
線香Aの両端と線香Bの片端の計三箇所に同時着火。
Aが燃え尽きる瞬間(30分後)にBの火がついてない方に着火。
Bが燃え尽きると45分。

567 名前:132人目の素数さん投稿日:2001/06/19(火) 04:26
>>566
なるほどお。
折れないなら燃やせ、ってことか。

568 名前:132人目の素数さん投稿日:2001/06/19(火) 04:34
>>567
燃やさなきゃ答える以前の問題かと。
恐らく燃やす時間が半分になることに
ついて言いたかったと思われるが。
ま、どうでもいいけどね。

569 名前:名無しさんだよもん投稿日:2001/06/19(火) 04:35
>>559
残念ながらがいしゅつです。

570 名前:132人目の素数さん投稿日:2001/06/20(水) 00:13
問題
30センチの棒がある。これを、ものさしとして使いたい。
1センチから30センチまで、すべての長さをセンチ単位で計れるようにするには、
最低限、どこに目盛を刻めばいいか?

571 名前:132人目の素数さん投稿日:2001/06/20(水) 00:16
1cm

572 名前:132人目の素数さん投稿日:2001/06/20(水) 00:24
……「1回で計れる」です、すみません。

573 名前:132人目の素数さん投稿日:2001/06/20(水) 01:13
1回でないのなら、場所がとれれば、目盛など1つも要らんぞ。

574 名前:132人目の素数さん投稿日:2001/06/20(水) 01:20
8cmの棒の場合
●|○○|●●●●|○
これで1〜8cmが計れる
こーいうことでしょ

575 名前:132人目の素数さん投稿日:2001/06/20(水) 02:19
1 3 7 9 15 20 25 29

ずいぶん重複してる個所があるけど、
少なくとも8ヶ所は必要なので、多分これであってると思う。
つうか>>569は葉鍵版の人だね。

576 名前:132人目の素数さん投稿日:2001/06/22(金) 07:37
↑ヴァカ発見↑

577 名前:132人目の素数さん投稿日:2001/06/24(日) 02:22
おもしろいかどうかしらんが
∫_0^2π√(1-cos^2(x))dx
こんなのやってみるとどーだ。やっぱ、つまらんかな。

578 名前:132人目の素数さん投稿日:2001/06/24(日) 03:34
∫_0^2π√(1-cos^2(x))dx=∫_0^2π|sinx|dx
=2∫_0^π(sinx)dx=2[-cosx]_0^π=2(1+1)=4

579 名前:132人目の素数さん投稿日:2001/06/24(日) 04:10
575だが、確かに俺はヴァカだった。
8ヶ所以上ってのは考え違い。
正しくは5ヶ所以上。
つうわけで、誰か正答希望。

ただ、576はコピペを貼り付けるのはやめて欲しい。。
クッキー消し忘れてヲタ板住人な事を晒された奴の煽りみたいだ。
と負け惜しみを言ってみる。

580 名前:132人目の素数さん投稿日:2001/06/24(日) 07:57
>>579
n箇所の目盛りを打つと、両端を含めた(n+2)箇所から
2箇所を選んでその差を計ることができる。
(n+2)C2≧30より、少なくとも7箇所の目盛りが必要なのでは?

581 名前:132人目の素数さん投稿日:2001/06/24(日) 12:57
579です。
俺も同じやり方で求めたんですが、
余計なことを考えて違う答えが出てしまったみたいです。
>>580の言うとおりです。
頭が腐ってるみたいなんで逝ってきます。

582 名前:名無しさんだよもん投稿日:2001/06/24(日) 13:07
>>579
呼んだ?

583 名前:132人目の素数さん投稿日:2001/06/24(日) 14:40
オモツ口イモソダイ、夕クサソオツ工テ

584 名前:132人目の素数さん投稿日:2001/06/24(日) 16:19
>>579
>ただ、576はコピペを貼り付けるのはやめて欲しい。。

あれコピペなの?っていうかあんなのどうみても手打ちだと思うんだが…
あれを保存してあって、コピってくるなら確かに別な意味で最低だが(w

585 名前:132人目の素数さん投稿日:2001/06/26(火) 23:33
ためしに試行錯誤でやってみたが8箇所が限界だった。
いい暇つぶしにはなったけど。

ほんとに8箇所以下でできるの?

586 名前:132人目の素数さん投稿日:2001/06/28(木) 02:47
問)414の問題を数学的に一般化できないか。ちなみに2*2のときは3本、3*3のときは4本、4*4のときは6本である。さて、n本のときは?
 この問題を解決するまでは、夜も眠れません。だれか助けて!
 じゃー、おやすみ...



   

587 名前:132人目の素数さん投稿日:2001/06/28(木) 22:24
「東158 西315 南413 北127」がオオサカを表すとき、
「北573 南829 北513 東211」はどこの都市を表すか。

588 名前:どぱきのん投稿日:2001/06/30(土) 23:41
ビリヤードの球(1〜15)から5つを数珠状に並べる。隣り合う1〜5個の球
を取り出したときの和で1から21がすべて作れるような並べ方は?
答えはしらみつぶしでわかるんですけど、考え方を知りたいです。

あと、n個の球で数珠を作るとき和がn^2 -n +1になるような並べ方が必ずある
ような気がするんですけど、証明してくれる人いませんか?

589 名前:132人目の素数さん投稿日:2001/07/01(日) 00:55
>>588
http://cheese.2ch.net/test/read.cgi?bbs=math&key=993571403&ls=50
で聞いたほうが(・∀・)イイんじゃないかなぁ?

590 名前:132人目の素数さん投稿日:2001/07/02(月) 18:42
>>588
某小説に出てる問題ですな。
答えは1、3、10、2、5でしたか。
結局はしらみつぶしだけど、
1)小さい三つのナンバーは1、2、3か1、2、4であることが必要
2)五つのナンバーの合計は21であることが必要
ってことであとはがんばるしかないですね。
6個のボールで1から31まで作るときは4通り答えがあるので
試してみよ。

591 名前:132人目の素数さん投稿日:2001/07/02(月) 22:24
盛岡

592 名前:132人目の素数さん投稿日:2001/07/04(水) 02:06
10メートルに木があります。その木えおある猿が夜に3メートル登り、
昼に昼寝して2メートル下がってしまう。さて何日間かかる?
今、テレビでやってた。10秒で解ける問題です。

593 名前:132人目の素数さん投稿日:2001/07/04(水) 02:26
>>592
    昼 夜
1日目 0m 3m
2日目 1m 4m
3日目 2m 5m
4日目 3m 6m
5日目 4m 7m
6日目 5m 8m
7日目 6m 9m
8日目 7m 10m
∴8日間。

594 名前:132人目の素数さん投稿日:2001/07/05(木) 03:05
2つの部屋があり、1つの部屋には3個のスイッチが、もう1つの部屋には3個
の電球がある。それぞれの部屋に1回ずつだけ入って、どのスイッチがどの
電球のものかを当てることができるか。 部屋の内部は外からみることは
できない。

友人から出された問題ですが、彼も解答を知りません。 どなたかわかる方、
おしえてください。

595 名前:132人目の素数さん投稿日:2001/07/05(木) 03:52
>>591
それは難問だと思う。「盛岡」でしょ?
たしかこれって未解決じゃなかったっけ?
ちがったかなぁ。

596 名前:tr投稿日:2001/07/05(木) 04:18
>>594 さん
「最初, 電球はすべて消えている」 って条件ありませんか?
聞いたところでは..

1) 2つのボタンを押し ON にして, 待つことしばし
2) ON にしたうちの一方を OFF にして, 電球のある部屋へ
3) 消えている電球に触ってみて暖かい方が 2) のボタンに対応

てな具合でした。

597 名前:594投稿日:2001/07/05(木) 04:41
>>596さん、
おお、こんなに早く回答をもらうとは。
最初の状態については「任意」と言われたのですが、私の記憶違い
だったかもしれません。 触ってみるは盲点でした。
とりあえず安心したのでぐっすり寝られます。

598 名前:594投稿日:2001/07/05(木) 13:32
>>596
考えてみたら、やはり最初の状態は「任意」でよさそうですね。

599 名前:132人目の素数さん投稿日:2001/07/07(土) 18:40
ファッションは数学です

600 名前:132人目の素数さん投稿日:2001/07/08(日) 01:32
594の問題はパパパパパフィーでやってた。

601 名前:1236投稿日:2001/07/11(水) 23:17
>>552
無くない?

602 名前:132人目の素数さん投稿日:2001/07/11(水) 23:24
2^53 = 9007199254740992

603 名前:しろうと132人目投稿日:2001/07/12(木) 14:18
がいしゅつだと思いますが(さすがに602を読むのは辛い)、
正六面体の1・2・3・4・5・6数字がそれぞれの面に書かれていて、
互いに平行な面の数字の合計は7であるサイコロがあります。
(はやい話が普通のサイコロ)

6人の中から1人を選ぶのには、
それぞれの人に1から6までの数字を割り当てて、
サイコロを1回振ればだれかが選ばれます。

さて、8人の場合は何回サイコロを振れば、
8人の中から1人を選ぶことができるのでしょうか。

604 名前:132人目の素数さん投稿日:2001/07/12(木) 14:48
3回くらいかなーとか思ったりしてみる。

605 名前:132人目の素数さん投稿日:2001/07/12(木) 14:54
なんかの本で見た問題なんだけど、外出かな。

問題
x^2-1=(x-1)(x+1)
x^3-1=(x-1)(x^2+x+1)
x^4-1=(x-1)(x+1)(x^2+1)
のように、x^n-1 を因数分解したとき、
それぞれの因数多項式の係数は常に0か±1である。
○か×か?

606 名前:しろうと132人目投稿日:2001/07/12(木) 15:02
>>603で問題に付け足します。
公平に8人の中から1人を選んでください。
(数学はこういうのが大きい)

>>604理由付けをおねがいします。

607 名前:132人目の素数さん投稿日:2001/07/12(木) 15:47
>>606
6^3=0(mod 8)

608 名前:132人目の素数さん投稿日:2001/07/12(木) 16:05
>>605
答えは×だけど因数分解書くのしんどい。暇があったら書きます。
(誰かが書くかもね。)
このスレでは出てないみたいだけど、私は本で見て知りました。
手計算でやろうとして途中で挫折した…指数3桁だもん。

609 名前:132人目の素数さん投稿日:2001/07/12(木) 16:14
>>605
○です。

x^n-1=(x-1){Σ[k=0,(n-1)]x^k}
因数分解をどこまですべきか、注釈が無いので

610 名前:132人目の素数さん投稿日:2001/07/12(木) 16:23
>>609
注釈がないってことは、
全ての可能性を考えなければならないので、
それじゃダメ。

611 名前:605投稿日:2001/07/12(木) 16:25
もちろん整数係数で分解できるまで分解するってことで。

612 名前:Mathematica投稿日:2001/07/12(木) 16:41
テキトーにやったら Factor[x^555 - 1] で係数に -2 が出てきた

613 名前:Mathematica投稿日:2001/07/12(木) 16:55
Factor[x^105 - 1]も×

614 名前:名無し投稿日:2001/07/12(木) 20:58
一に、二本線を引いて5にしてください。

615 名前:しろうと132人目投稿日:2001/07/12(木) 21:16
>>606
最低何回振ればいいでしょうかの付け足しです。

616 名前:132人目の素数さん投稿日:2001/07/12(木) 21:32
一回で十分
サイコロの角度とかも利用できるし

617 名前:しろうと132人目投稿日:2001/07/12(木) 21:46
>>616
はい、ピーター=フランクフルト食べたいの本ならば1回ですね。

618 名前:132人目の素数さん投稿日:2001/07/15(日) 05:31
>>606
偶数奇数で半分ふるい落としながら3回・・・かな?

619 名前:132人目の素数さん投稿日:2001/07/15(日) 05:50
8つの頂点を利用して1回。
頂点に集まる目の和が6、7、9、10、11、12、14、15の8通りあるのを使い
直方体の箱の内側にでも投げて見えてる目の和を利用すれば…
ってのはいかが?

620 名前:>614 スバラシイ投稿日:2001/07/15(日) 06:05
円錐形の容器に落とし込んでもいいね

621 名前:620投稿日:2001/07/15(日) 06:06
619 スバラシイ ・・・でした

622 名前:132人目の素数さん投稿日:2001/07/17(火) 01:16
正方形を幾つか鋭角三角形に分割せよ。
最少幾つで?

623 名前:132人目の素数さん投稿日:2001/07/17(火) 01:18
幾つか→幾つかの

624 名前:投稿日:2001/07/17(火) 01:32
はい整数問題

(1)
(2^n -1)nが整数となるnを全て求めよ
(2)
(2^n +1)/n^2 が整数となるnを全て求めよ

625 名前:投稿日:2001/07/17(火) 01:33
訂正
(1)(2^n -1)n → (2^n -1)/n

626 名前:132人目の素数さん投稿日:2001/08/01(水) 05:33
age

627 名前:>624投稿日:2001/08/01(水) 08:58
In[3]:=
Cases[Range[50000], n_ /; IntegerQ[(2^n - 1)/n]]
Out[3]=
{1}
In[7]:=
Cases[Range[30000], n_ /; IntegerQ[(2^n + 1)/n^2]]
Out[7]=
{1, 3}

かなり少ないみたいですね。

628 名前:132人目の素数さん投稿日:2001/08/07(火) 18:03
このCのプログラムをコピペしてそのままコンパイル実行してみて。
何がでてくるかな♪。
Cもってないひとはここにあるよ→
http://www.bloodshed.net/devcpp.html

#include<stdio.h>
int main() { int i;
char*a[9]={
"#include<stdio.h>",
"int main() { int i;",
"char*a[9]={",
"};",
"for(i=0;i<3;i++) puts(a[i]);",
"for(i=0;i<9;i++) {putchar(34);fputs(a[i],stdout);",
"putchar(34); if(i<8)putchar(44);putchar(10);}",
"for(i=3;i<9;i++) puts(a[i]);",
"}"
};
for(i=0;i<3;i++) puts(a[i]);
for(i=0;i<9;i++) {putchar(34);fputs(a[i],stdout);
putchar(34); if(i<8)putchar(44);putchar(10);}
for(i=3;i<9;i++) puts(a[i]);
}

629 名前:132人目の素数さん投稿日:2001/08/07(火) 19:01
>>588
>>590
少し考えればもっと絞り込めるよ。
自明だとは思うけど、

●重複する数の玉はない(∵並べ方が21通り)
●11まで取れればOK(∵それ以降は「逆」で同じ)
●11以上の数の玉はない(∵例えば「15」の玉があると残りの玉で7〜14が取れない

これで 1, 2, 10 は確定できる。
残りの玉の取り方でしらみ潰しできるかどうかだね。

630 名前:629投稿日:2001/08/07(火) 19:08
>11以上の数の玉はない
12だった。鬱だ・・・

631 名前:132人目の素数さん投稿日:2001/08/08(水) 16:50
age

632 名前:132人目の素数さん投稿日:2001/08/14(火) 19:53
11の格子点の問題が今、伊藤家でやってた
大発見だってよ・・・・相変わらず日テレは糞だな

633 名前:132人目の素数さん投稿日:2001/08/15(水) 19:05
>>11

1 2 3
6 5 4
7 8 9

この順番で通ればいい

634 名前:132人目の素数さん投稿日:2001/08/15(水) 19:26
>>633
頭悪すぎ・・・・・

635 名前:132人目の素数さん投稿日:2001/08/15(水) 20:19
>>628
korenani?

636 名前:y^2=x^3+ax+b投稿日:2001/08/15(水) 21:39
712
683
549
じゃないのかにゃ

637 名前:132人目の素数さん投稿日:2001/08/15(水) 21:40
>>635
試してみたの?

638 名前:132人目の素数さん投稿日:2001/08/15(水) 21:46
夷東家の視聴率を考えると
糞なのは・・・

639 名前:132人目の素数さん投稿日:2001/08/15(水) 23:47
>>628 試してみた。オモシロイね。

640 名前:639投稿日:2001/08/16(木) 00:16
そうそう。MSDOSのウィンドウで実行しないとダメ。
すぐ消えてしまって。

641 名前:132人目の素数さん投稿日:2001/08/16(木) 00:39
>>634
修行が足りないようだね君は。

642 名前:132人目の素数さん投稿日:2001/08/16(木) 00:51
>>628
こうしてやるとそのまま実行でもEnter押すまで消えない。

int main() { int i;
char*a[10]={
"#include<stdio.h>",
"int main() { int i;",
"char*a[10]={",
"};",
"for(i=0;i<3;i++) puts(a[i]);",
"for(i=0;i<10;i++) {putchar(34);fputs(a[i],stdout);",
"putchar(34); if(i<10)putchar(44);putchar(10);}",
"for(i=3;i<10;i++) puts(a[i]);",
"getchar();",
"}"
};
for(i=0;i<3;i++) puts(a[i]);
for(i=0;i<10;i++) {putchar(34);fputs(a[i],stdout);
putchar(34); if(i<10)putchar(44);putchar(10);}
for(i=3;i<10;i++) puts(a[i]);
getchar();
}

643 名前:132人目の素数さん投稿日:2001/08/16(木) 12:59
{■「上」}*{R「右」}*{W「右」}

{Q「left]}*{NE「下」}*{Z「右」}

解読してください。

644 名前:132人目の素数さん投稿日:2001/08/16(木) 13:25
http://members2.tsukaeru.net/makoto/suugaku.bmp
↑この図の?は、何度でしょうか
これは、有名な問題です。昔、某中学校の入試問題にでたそうです。

645 名前:132人目の素数さん投稿日:2001/08/16(木) 14:17
644の解答
左下から時計回りと逆にP、B、C、Dとする
まずこの問題でポイントになるのは
1正三角形の発見
2円の性質
3直角
である
答えは30だけど証明のヒントを書き込んどきます。
1 △PBCの外心GはBD上にある
2 GのPDに関する対称点をMとすると△PMGは正三角形
3 M、P、B、Cは共円
4 角CMG=角DMG=40
5 4よりC、D、Mは一直線
6 PBCMは中心Gの円に内接する
7 2DCP=2MCP=MGP=60

という流れです

646 名前:132人目の素数さん投稿日:2001/08/16(木) 14:25
>>645
すげー!!

647 名前:645投稿日:2001/08/16(木) 17:57
644さんどこの入試問題ですか?
こんなの中学の入試時間内でとくのは不可能だと思うんですが。
一般にこの問題は正三角形にキチャクさせればできるのですが、
角度のあたえ方によってかなりの直感てきな力が必要となります。
これが出るとしたら開成?灘?

648 名前:132人目の素数さん投稿日:2001/08/16(木) 19:44
>>647
どこで出たかわかりません。
学校の選択数学で出ただけですから
ただ、うちの中学のやつもみんな解けてなかったから、
かなり難しい問題ですね。

649 名前:132人目の素数さん投稿日:2001/08/16(木) 21:30
>>644
その問題、俺が中学の時行ってたS○Gつう塾(今では予備校状態らしいが)でも、出題されていたよ。
今をさること17年前だな。
そのときの話しでは超有名で超古典的つう話しだった。
中学受験という話は聞かなかったけどな〜

650 名前:132人目の素数さん投稿日:2001/08/16(木) 22:09
>>644
数学板10大がいしゅつのひとつ。
http://www.mitene.or.jp/~tomo-s/langley/langley10.html

651 名前:132人目の素数さん投稿日:2001/08/17(金) 00:10
>>644 が見れなかったのに
>>650 の書き込みだけで問題が分かってしまった。

652 名前:132人目の素数さん投稿日:2001/08/17(金) 00:53
ラングレーの問題か。大学教授でもわからんかったらしいからな。

653 名前:132人目の素数さん投稿日:2001/08/22(水) 20:44
数学ではないが、ちょっと面白い問題を一つ。

大きさも形も色も質感も重さも全く同じ(要するに見分けがつかない)
鉄の棒と、磁石の棒がある。
他の道具を一切使わずに(地磁気を使うのもNG)、この二本の棒を区別せよ。

654 名前:132人目の素数さん投稿日:2001/08/22(水) 20:57
覆面算。

A
+)B
--
AC

655 名前:132人目の素数さん投稿日:2001/08/22(水) 21:01

ずれた。。。
A+B=10A+C って意味ね。

これ以上ないってくらい簡単だけど、
覆面算の面白さの片鱗が味わえる良問だと
勝手に思ってます(w

656 名前:132人目の素数さん投稿日:2001/08/22(水) 21:04
>>653
T字に置く。

657 名前:132人目の素数さん投稿日:2001/08/22(水) 22:39
次のような関係式があります。「銀」はいくらでしょうか。
銀×2=金
銀−木=陶器
銀+水晶=エメラルド
ダイアモンド−ルビー=真珠
金−真珠=陶器
銀+陶器=ルビー

658 名前:653投稿日:2001/08/24(金) 01:49
やっぱり数学じゃないが。

ある物体Xを高さ10mから床に落としたら壊れたが、
高さ20mから落としたら壊れなかった。
なぜか?
落とし方、床の状況などは同じ条件とする。

多少あいまいだけど、出題者の意図を汲み取って答えてちょ☆

659 名前:132人目の素数さん投稿日:2001/08/24(金) 01:59
>>658
落とすときに終端速度以上の初速度を与えたとか。

660 名前:132人目の素数さん投稿日:2001/08/24(金) 02:09
>>658
10m〜20mの間に木の枝があってひっかかった

661 名前:132人目の素数さん投稿日:2001/08/24(金) 02:16
>>658
パラシュートが付いていて、20メートルだとそれが開く余裕があった。

662 名前:653投稿日:2001/08/24(金) 02:32
>>661
正解!!

663 名前:132人目の素数さん投稿日:2001/08/24(金) 02:39
>>662
マジで?
ただのクイズじゃ〜ん。
もっと数学とか物理の要素取り入れてよ〜。

664 名前:653投稿日:2001/08/24(金) 03:05
>>663
そっか、スマン。
俺は激しく感動したんだが。

665 名前:132人目の素数さん投稿日:2001/08/24(金) 03:52
で、>>653の答えはどうなった?

666 名前:132人目の素数さん投稿日:2001/08/24(金) 04:02
>>665
棒の中央に他方の先端を近づけてみる。
>>656に答え書いてあるじゃん。

667 名前:132人目の素数さん投稿日:2001/08/24(金) 04:20
>>657
なんか微妙に違うような・・・・
ひねってるのかな・・・・

668 名前:653投稿日:2001/08/24(金) 04:29
まだまだ出すぞ♪

ここに赤い帽子3つと白い帽子2つが存在してる。
A君、B君、C君は全員このことを知っている。
今、全員に帽子をかぶせた。
自分の帽子の色は直接見えない。
他の二人の帽子の色は分かる。
まずA君に自分の帽子の色が分かるかと聞いたが、分からないと答えた。
次にB君にも聞いたが、やはり分からないと答えた。
さて、C君の帽子の色は?

669 名前:132人目の素数さん投稿日:2001/08/24(金) 04:47
Aが分からないと答えたことを聞いた上で
Bも分からないと言ったと考えることにする。
Cが白なら、Aがわからないと言った時点で
Bの赤が確定するので、Cは赤。

670 名前:132人目の素数さん投稿日:2001/08/24(金) 04:48
white

671 名前:132人目の素数さん投稿日:2001/08/24(金) 04:50
A- r,r ->?
 - r,w ->?
 - w,w ->r
 - w,r ->?
B- r,r ->r
 - r,w ->?
 - w,w ->?
 - w,r ->r

672 名前:132人目の素数さん投稿日:2001/08/24(金) 04:52
>>668
問題の条件じゃまだ絞りきれてないような気が・・・。
B,Cがどちらも白、またはA,Cがどちらも白ってのを
否定したにすぎないでしょう?
これじゃCが赤、白どちらの場合も考えられるYO!

673 名前:132人目の素数さん投稿日:2001/08/24(金) 04:58
仮にCが白だったとする。
するとAが見るB,Cは「赤白」か「白白」のどっちか。
後者ならAは自分が赤だとわかる。
よって「赤白」しかない。
するとBは、Cが白なので自分は赤だとわかる。
よってCは赤。

674 名前:669投稿日:2001/08/24(金) 05:00
漏れの答え違うか?
Aが「わからん」と言ったのをBが聞いたときに
Cが白を被ってたら、Bの赤が確定するから、
Bが「わからん」とは言わないよな。

675 名前:669投稿日:2001/08/24(金) 05:03
>>673
スマソ
説明がかぶってしもたよ。

676 名前:132人目の素数さん投稿日:2001/08/24(金) 05:05
僕も問題出しまーす。
(以下は実話)
A君が飴玉を舐めていました。
A君曰く「飴は小さくなってくるとなくなるのが速くなってくる。」
さてA君の言ってることは正しいでしょうか?
直感でどうですか?

細かい条件は
飴は球体とし、
飴の溶ける速さはそのときの飴の表面積に比例するとします。
余裕のある人は飴の体積と時間の関係式も立ててみてネ。
まあ、ちょっとした大学入試問題にはなるかも。

677 名前:132人目の素数さん投稿日:2001/08/24(金) 05:23
溶ける速さが飴の表面積に比例するなら
小さくなると溶けるのは遅くならんか?

678 名前:132人目の素数さん投稿日:2001/08/24(金) 05:30
とける速度は遅くなるけど球の体積が減るので
結局はかわんないんじゃ?

679 名前:132人目の素数さん 投稿日:2001/08/24(金) 06:25
>>676
条件があいまいな感じもするが、溶ける速さというのが体積の
ことだとすると、それが表面積に比例する、ということから
一定の速さで径が減るということになる。
つまり、径が小さくなるほど一定時間内に溶ける分量は減るということになる。

この、体積の減る速さが減る、という状況を、人間の口のなかの感覚器と
その情報処理機構がどう解釈するかによって、
1.単純に体積に比例すると解釈して、小さくなるほど無くなるのが遅くなるように感じる。
2.ロガリズミックに認識されるので、1とは逆に速く無くなるように感じる。
3.唾液の量が一定とすると味が薄くなるので、速く無くなるように感じる。
ってところか。

単純に体積の減少を計算するのなら1となりA君の直観は間違いとなるが、
なんとなく現実的には2+3でA君の直観は正しいというところなんじゃないだろうか。

680 名前:132人目の素数さん投稿日:2001/08/24(金) 07:09
>>676
飴の体積は時間の三次関数で表される。
飴の溶ける速さは時間の二次関数。
飴の半径は時間の一次関数。

681 名前:132人目の素数さん投稿日:2001/08/24(金) 10:32
312の答えがかなり気になるんですが、教えてもらえませんか?

682 名前:132人目の素数さん投稿日:2001/08/24(金) 17:26
age

683 名前:132人目の素数さん投稿日:2001/08/24(金) 17:39
実際は、唾液の量も絡んできそうだけどな。
まあ無視するって事で。

駄レススマソ

684 名前:132人目の素数さん投稿日:2001/08/24(金) 21:56
>>668
ちょっと拡張した問題を考えてみた。
n人の人に対し赤帽n個、白帽k個を用意して同じゲームをする。
ただし0<k<nとする。
このとき最初のk人が「わからん」と答えたら残りの人は全員赤帽であり「わかる」と答える。
これを示せ。(間違ってたらスマソ)

685 名前:132人目の素数さん投稿日:2001/08/24(金) 23:09
(´`)<ののたんが中学校で習った問題解いてみてほしいのれす。
    13の8乗を144で割った余りはいくられすか?

686 名前:132人目の素数さん投稿日:2001/08/24(金) 23:20
>>685
97

687 名前:132人目の素数さん投稿日:2001/08/24(金) 23:23
(´`)<わお!正解れす。さすが数学板のひとはすごいれすね。
    さてそろそろモー板にもどります。
    おじゃましました。

688 名前:バカ投稿日:2001/08/25(土) 11:15
>>588
私もその小説読んで一時期考えてたんですが、
そういやビリヤードの球は15まででしたね。私ナインボールしかやらないから、九までで考えてました(アホや)。

で、いくら考えても「解なし」になるので、こりゃ引っ掛けだ。
「さかさにして6は9に、9は6として使える」と思いました。
しかも「数珠ごと回転させると、”6を9として使おうとしたとき、9も6になってしまう”」
と、無意味に深読みして、「玉と玉の間隔を広く取れば、任意の球だけを移動させ、反転させることが可能」
とか思ったのです(ああ、恥ずかしい)
で、答えは1・2・3・6・9になったんだったかな・・・

689 名前:132人目の素数さん投稿日:2001/08/25(土) 16:13
>>11
. . . .
. . .
. . .
.
なぜかこっちのほうが簡単だよな?

690 名前:132人目の素数さん投稿日:2001/08/25(土) 16:55
>>685
13^8≡(12+1)^8≡8*12+1≡97 (mod144)

691 名前:653投稿日:2001/08/25(土) 18:36
>>684
元ネタ提供者の意地で解いてみた♪

「1人目が分からないと言った」→「2人目以降に白帽がk人はいない」
(2人目以降に白帽がk人いたら、自分が赤帽だと分かるから)

「2人目が分からないと言った」→「3人目以降に白帽がk−1人はいない」
(3人目以降に白帽がk−1人いたら、簡単な推論により、自分が赤帽だと分かるから)

「3人目が分からないと言った」→「4人目以降に白帽がk−2人はいない」
(4人目以降に白帽がk−2人いたら、簡単な推論により、自分が赤帽だと分かるから)

…(略)

「k人目が分からないと言った」→「k+1人目以降に白帽が1人もいない」
(k+1人目以降に白帽が1人いたら、簡単な推論により、自分が赤帽だと分かるから)

従って「最初のk人が分からないと言った」→「k+1人目以降は赤帽」が成り立つ。

自分より前の人の帽子を見る必要はないのがポイントだな。
(実際、>>668でもB君はA君の帽子を気にする必要はない)
しかし、逆に言えば、必要条件だけで絞っていってるから、
本当に「最初のk人が分からないと言う」ような場合が存在するかは議論の余地がある。

692 名前:132人目の素数さん投稿日:2001/08/25(土) 18:53
>>1つかもう数時間で閉鎖するってのに何あほなこと話してんだヴォケ
シネやカス!!
お前みたいなザコが来るからサーヴァーへの負担大きくなって閉鎖になったんだ!!
氏ねヴォケ

ラウンジもしくはニュース板逝ってみろ!
のんきにあほ話してんのはここくらいなもんだぞ?

693 名前:684投稿日:2001/08/25(土) 21:33
>>691
逆に、後のn−k人がみんな赤帽なら最初のk人は「わからん」と言うんでは?
正しければ証明し、正しくなければ判例をあげよ。
(ズボラな性格丸出し)

694 名前:132人目の素数さん投稿日:01/08/27 18:50 ID:EUn8t6AQ
A「毎年10万円ずつあげよう」
B「それを今すぐ全部(毎年分)ちょうだい」

ある金額を1度に渡せば、理論上、永久に10万円ずつ渡すのと同じ事になる。
それはいくらか?

695 名前:132人目の素数さん投稿日:01/08/28 00:11 ID:78n8tUkc
次のような関係式があります。「銀」はいくらでしょうか。
銀×2=金
銀−木=陶器
銀+水晶=エメラルド
ダイアモンド−ルビー=真珠
金−真珠=陶器
銀+陶器=ルビー

↑解けないの?

696 名前:132人目の素数さん投稿日:01/08/28 00:56 ID:4Jtmx6nw
問題ってゆうより質問なんですが、
「3本のくじの中に1本だけ当たりが入っていて、
そのうち一本を選んだら、
残りの2本のうちの一本を、
こっちは確実にはずれだと教えられた。
この時点で選ぶくじを変えてもいいなら、
初めに選んだものより、もう一つのものに変えた方が当たる確率が高い。」
と聞いたんですが、どうしてでしょうか。

問題の意味がわかりにくかったら質問して下さい。

697 名前:132人目の素数さん投稿日:01/08/28 01:50 ID:cTTANqv2
>>696
ガイシュツ

698 名前:132人目の素数さん投稿日:01/08/28 03:14 ID:PnSfpVg.
>>695
その問題は、漏れが>>667でコメントしてるんだが、
微妙に違わないか?
一般的に言われてるモノとは微妙に違うよね。
調べてみると複数パターンあるみたいだけど
その計算通りになるパターンは発見できなかったよ。
それとも漏れの勘違いで、全然違ったりするのかな?

699 名前:名無しさん投稿日:01/08/28 13:28 ID:AjUyviq6
>>694
銀行に預けて利子が10万円超える分だけ渡せばいい。
しかしlimit[t→∞]#銀行(t)≠0を証明できればの話だが

700 名前:15枚の金貨投稿日:01/08/28 14:19 ID:b9FFkb4Q
意地悪な王様がいる国のお話です。
ある日、王様は家来に言いました。
王様「この15枚の金貨のうち、1枚だけ重さの違う偽物がある。お前にはわかるか?」
家来はおどおどしながら適当に1枚の金貨を王様に渡し、言いました。
家来「こ、この金貨でしょうか?」
王様「ばか者!これは本物だ!」
と、その金貨を家来に投げ飛ばしました。
王様「ここに天秤がある。それを3回だけ使わせてやるから、偽者を見分けろ!
   出来なかった場合は処刑だ!」
家来「ひぃぃ!」

さて、この家来は無事に助かることができるでしょうか?

701 名前:132人目の素数さん投稿日:01/08/28 14:23 ID:64N59Bu6
>>632
あのな〜あれは数学番組じゃねえんだよ・・・
専門知識のある奴らから見て糞じゃない番組って何だよ

702 名前:15枚の金貨投稿日:01/08/28 14:23 ID:b9FFkb4Q
残り14枚のうち偽物が1枚。
だれか挑戦してみてください。

703 名前:132人目の素数さん投稿日:01/08/28 14:24 ID:64N59Bu6
>>700
問題が悪い
助かる「可能性」があるのか「能力」があるのか
どちらを問うているんだ?

704 名前:15枚の金貨投稿日:01/08/28 14:28 ID:b9FFkb4Q
可能性の場合「さて、この家来は無事に助かることができるでしょうか?」
能力性の場合「さて、この家来は無事に助かるでしょうか?」
「できる」は可能性かと。

705 名前:132人目の素数さん投稿日:01/08/28 14:44 ID:SJfJYNSE
いずれにしても、この家来は、こんな気まぐれな理由で
処刑する王に仕えた時点で、命の保証はない。
よって、助かることは出来ない。

706 名前:132人目の素数さん投稿日:01/08/28 14:48 ID:sPwYMc12
>>700
っていうかさぁなんでこうもとても有名な問題を
何度も何度も何度も何度も何度も何度も何度も
何度も何度も何度も何度も何度も何度も何度も
何度も何度も何度も何度も何度も何度も何度も
何度も何度も何度も何度も何度も何度も何度も
書きたがる奴がいるのか?

707 名前:15枚の金貨投稿日:01/08/28 14:51 ID:b9FFkb4Q
>>706
同じではありません。
今までは12枚or13枚でした。

708 名前:132人目の素数さん投稿日:01/08/28 15:27 ID:sPwYMc12
>>707
ぉぃぉぃ、、、その歳になるまでこんな有名なのを本当に知らなかったのか?(唖然

709 名前:15枚の金貨投稿日:01/08/28 15:42 ID:b9FFkb4Q
>>708
。。。2chの中の話だが。

710 名前:15枚の金貨投稿日:01/08/28 15:59 ID:b9FFkb4Q
ていうか、>>700の問題は、ガイシュツの問題をアレンジして即行で作った
ものだが、そんなに有名だったのでしょうか?
できればソース希望。
ていうか、答え書いて。

711 名前:132人目の素数さん投稿日:01/08/28 18:05 ID:xEQkDeHg
素数を割れ。

712 名前:132人目の素数さん投稿日:01/08/28 18:38 ID:oqhEs1Nk
>>710
確かどこかのスレにn枚って話あったよ?

713 名前:132人目の素数さん投稿日:01/08/28 20:29 ID:KdduSk4Y
>>699
永久っつーか、半永久的に、というべきか。

714 名前:風見天都投稿日:01/08/28 21:35 ID:kzfh62Io
kとnを自然数とし、

k^n−1がk−1で割れることを証明せよ

715 名前:132人目の素数さん投稿日:01/08/28 22:47 ID:JpMqA1qo
>>714
(x+1)^n−1 はxで割れる
x=k-1

716 名前:715投稿日:01/08/28 22:49 ID:JpMqA1qo
間違えた…k>1の時ね(汗

717 名前:132人目の素数さん投稿日:01/08/29 00:01 ID:fXsZ/aRY
>>715
因数定理で一発でしょう。1^n-1=0だ。

718 名前:132人目の素数さん投稿日:01/08/29 00:25 ID:Qjtjk7VU
>>717
k-1が1次であることを注意すればね

719 名前:132人目の素数さん投稿日:01/08/29 00:35 ID:78Cud7Vk
前提が偽であれば
結論は常に真

の分かり易い説明きぼんぬ。

720 名前:132人目の素数さん投稿日:01/08/29 00:50 ID:Qjtjk7VU
>>719
命題:「飯を食えば眠くなる。」

飯を食って眠くなった。…真
飯を食って眠くならなかった。…偽
飯を食ってないけど眠い…真
飯を食ってないし眠くない…真

下2つは飯を食ってないのだから、命題は嘘をついてない

他に「雨が降ると川が氾濫する」等

721 名前:132人目の素数さん投稿日:01/08/29 14:47 ID:ZDRyevTs
>>694-699
人類が存在する限り、お金と言う概念は存在するので、人類が滅亡するまで、銀行と言うものはあるはずだ。

722 名前:132人目の素数さん投稿日:01/08/29 15:31 ID:/hZ7d5VY
>>721
>人類が存在する限り、お金と言う概念は存在するので、

根拠不明

>人類が滅亡するまで、銀行と言うものはあるはずだ。

お金という概念だけ歴史的記録として存続するかも知れないが
実際使いつづけられるかどうかはわからない

723 名前:132人目の素数さん投稿日:01/08/29 15:47 ID:ZDRyevTs
>>722
お金が廃止され、物々交換に戻るとは、とても思えないだろ。

724 名前:132人目の素数さん投稿日:01/08/29 16:01 ID:/hZ7d5VY
>>723
そりゃお前数学屋としては許されない発言だよ。。。

725 名前:132人目の素数さん投稿日:01/08/29 17:10 ID:ZDRyevTs
>>51
パーで負けたくないから、相手はチョキを出す、それを読んでグー。
でも、相手がそれを読んでいたら、パーを出すよ.

726 名前:132人目の素数さん投稿日:01/08/29 17:12 ID:DVP8piZ6
>>719
もう古典だけど・・・
「1=2ならば私は法王である」
証明:私と法王は2人の人間である。ここで1=2を仮定すると、これ
   は1人の人間である。よって私は法王である。

727 名前:132人目の素数さん投稿日:01/08/29 18:00 ID:/IoC86fQ
25

728 名前:132人目の素数さん投稿日:01/08/29 23:47 ID:YNIOwV3E
問題です
1
11
12
1121
次にくる数は?

729 名前:むずかしい投稿日:01/08/29 23:50 ID:UEDbX5b2
http://www.free-park.net/~netidol/imgboard2/img-box4/img20010815145017.jpg

730 名前:132人目の素数さん投稿日:01/08/29 23:54 ID:AaLxzv8c
1321

731 名前:132人目の素数さん投稿日:01/08/29 23:54 ID:cnov1ILo
>720 >726

ありがとうございました。

732 名前:132人目の素数さん投稿日:01/08/30 01:57 ID:IuAwJWM.
>>726
その場合

「1=2ならば私は法王である」
「1=2ならば私は法王でない」
「1≠2ならば私は法王でない」

の3つが真だけど下の2つはどうやって証明すんの?

733 名前:132人目の素数さん投稿日:01/08/31 19:42 ID:mKa.dXAg
>>699
定期預金だと、少なくとも1年間5%の利子がつくから、200万。

734 名前:132人目の素数さん投稿日:01/09/04 00:57 ID:BEZ5vM6o
>>622
10個じゃないか?
合ってるかどうかだけ教えてくれ。

735 名前:132人目の素数さん投稿日:01/09/14 04:36
面白い問題を、どうか面白い問題をください
一週間ぐらい潰してしまう、それでいて証明するのに必要な知識は
ほとんどいらない、そんな問題、面白い問題ください

736 名前:132匹のわんちゃん投稿日:01/09/14 11:55
>>732
私と法王が2人と前提されていることから2番目が導出される。この場合は1=2は必要ない。最後のも同様。

737 名前:132匹のわんちゃん投稿日:01/09/14 12:03
>>735
クロスワード集を買うとか。

738 名前:>投稿日:01/09/14 12:32
>>728
>問題です
>1
>11
>12
>1121

どっかでみたことある
"上の数字列のランレングスをかきくだす"だな

1121の次は
122111 (1が2,2が1,1が1)
112213
12221131


ランレングスが10以上のなるパタンがあった場合
次の段のランをどう解釈する?(例えば長さ10の場合)
1) 10 というひとつのシンボルとみなす
2) 1 と0 という2つのシンボルとみなす。
どっちだ?


関連して次の問題

この操作に
"22" 以外の不動点があるかないかを論ぜよ。

739 名前:132人目の素数さん投稿日:01/09/14 13:08
(2^(3^i))+1は3^(i-1)の倍数。

740 名前:132人目の素数さん投稿日:01/09/14 14:07
数学屋には簡単な問題かもしれんが、こういうのどう?
「1,2・・・10までの数の集合をとる。そのなかから一つ数をとって、
それより大きな数を掛け合わせたものをX、小さいのを掛け合わせてYとする。
X=Yとなる場合をすべて求めよ」
一瞬でわかった人は、解答を遠慮してね。

741 名前:132人目の素数さん投稿日:01/09/14 14:21
>>736
>私と法王が2人と前提されていることから2番目が導出される。この場合は1=2は必要ない。

よくわからんのだけど
1=2と仮定されているのだから1題目と同様に考えると私は法王ではないと断言できないというか
これだけでは真偽は定まってないのでは?

742 名前:738投稿日:01/09/14 17:08
ついでに S={ [0-9]を有限個並べた数字文字列}
f:S->S を ここでいう 一つ下の文字列を生成する写像とする

s = f の n回適用(s) と
なるような nと sが s="22"(この時はn)は任意以外にあるかないか?

---
S = { 任意の非負整数の有限個のならび }
f S-> S を
10 を 1,0 とみずに10とう一シンボルと解釈する方式では
s="2,2" 以外 f の n回適用の不動点はあるかないか?

743 名前:132人目の素数さん投稿日:01/09/14 19:58
n個の点があったとき、どの3点を選んでも、3角形を形成しないような
線分のとり方で最大の場合の線分の数を答えよ。

n=3の場合、2
n=4の場合、4

744 名前:132人目の素数さん投稿日:01/09/14 20:20
問題を普通に解釈してとくとn=3の場合,3 n=4の場合、6
一般のnの場合nC2 になるんだが俺が勘違いしてるのか?

スマンが問題をもう少し詳しく解説してくれい

745 名前:132人目の素数さん投稿日:01/09/14 21:04
3点あってそれを全部線分で結んじゃうと三角形が形成されちゃうので、
三角形ができないような線分の組は、Vみたいな2つの線分から
できたものになります。

同様に4点の場合は、□がその組み合わせの図形。

ちょっと文章が分かりにくいかったかな。

746 名前:132人目の素数さん ?投稿日:01/09/14 21:43
すみません。全く板違いの者ですが・・以前あったIDに7が三つ続けて出る
確率を計算して頂きたいのですが・・・教えて下さい・・・

747 名前:132人目の素数さん投稿日:01/09/15 03:05
IDは8文字で固定。
IDに使われる文字は、
小文字 a〜z 26−−−α
大文字 A〜Z 26−−−β
数字 0〜9 10−−−γ
その他 ,./ 3−−−δ
α+β+γ+δ=ω

ω=65個の文字からまったくのランダムに8個が選ばれ、IDが生成されるものとする。
ここで、7が3つ連続するということを、7が高々3つだけ連続することと理解する。
よって
ω^5   ω^4
---- − ---- =(各自)
ω^8   ω^8

748 名前:算法少女2001投稿日:01/09/15 03:16
正n角形の辺を延長した直線で平面はいくつの部分に分割されるか。
例えば、正三角形だと7、正方形だと9、・・・。
(出典:雑誌「数学教室10月号」(国土社)の表紙)
解答できたら、この雑誌の編集部に送ることになっている。

749 名前:132人目の素数さん投稿日:01/09/15 03:33
n≧2(∈N)のとき三辺の長さがn-1,n,n+1
の三角形Aを考えます。
Aの面積SがS∈Nであるとき、Sを小さい順に
並べるとある数列になります。
一般項を求めてください。

750 名前:132人目の素数さん投稿日:01/09/15 03:37
>>747
>ここで、7が3つ連続するということを、7が高々3つだけ連続することと理解する。
いや、その理解はおかしいだろ…。

751 名前:747投稿日:01/09/15 03:40
じゃあ
1/65^2で終了。

752 名前:132人目の素数さん投稿日:01/09/15 04:03
>>751
>1/65^2で終了。

Σ( ̄□ ̄;
ここまで馬鹿な奴がいるとは思わなかったYO!
マジで言ってる?ネタだよな?ネタと言ってくれ...頼む!
IDが8文字あるというのに...

753 名前:746投稿日:01/09/15 05:50
747さんくだらない問題につきあってくれてありがとうございます。
7が三つ以上どこかに連続しているということなんですが・・
約1/278916ということですね。すみません

754 名前:132人目の素数さん投稿日:01/09/15 07:16
>>749
S={(7+4√3)^n-(7-4√3)^n}√3/4
でいいっすか?

755 名前:132人目の素数さん投稿日:01/09/15 11:37
>754 最後のルートは不要。

756 名前:749投稿日:01/09/15 15:19
754さん正解です。
二次不定方程式の問題でした。

757 名前:132人目の素数さん投稿日:01/09/15 19:09
直円錐台(等脚台形を線対称の軸で回転させた形)を下のような平面で
切り分けます。そのときの上下の立体の体積比を、円錐台の上底の半
径r、下底の半径R、高さhで表して下さい。

切断面は、回転軸を含む平面Xと上底面の円周との交点の一つをA、下
底面の円周との交点のうちAから遠いほうの交点をBとし、Xに垂直でA、
Bを通るものです。


簡単にいうと円錐台を上下の円に接するように斜めに切って上下比べる、
という問題です。
計算はさほど難しくないでしょうが、結果が面白かったもので。

758 名前:132人目の素数さん投稿日:01/09/16 23:09
>740
そんな数あるの?

759 名前:132人目の素数さん投稿日:01/09/16 23:16
>>758
1×2×3×4×5×6=8×9×10=720。

760 名前:132人目の素数さん投稿日:01/09/16 23:59
>>759
はい、正解。わかってると思うけど、素因数分解の一意性から、
それ以外には解はありません。

761 名前:投稿日:01/09/17 03:33
任意のイプシロン>0

762 名前:132人目の素数さん投稿日:01/09/17 10:00
>>738
13213341はOKっぽい

763 名前:132人目の素数さん投稿日:01/09/18 12:29
>>762
不動点としてOKってこと?んなわけないよな?

764 名前:132人目の素数さん 投稿日:01/09/19 21:14
ちょっと知恵つけた中学生が解くくらいの
公務員とかの知能問題なんかででてきそうな感じの
友達が四苦八苦しながら解いてるのを
発想の転換だよふふんという気持ちで見れるような
面白い問題はありませんか。

765 名前:132人目の素数さん投稿日:01/09/19 21:19
南へ3000キロ
東へ7000キロ
北へ3000キロ
移動したら元の小屋に戻った
出発点の小屋で飼われている動物の名を答えよ

766 名前:132人目の素数さん投稿日:01/09/19 21:28
ポチ…ていうかどうやって答えろと。

767 名前:132人目の素数さん投稿日:01/09/19 21:37
>>765
北極だけに白熊とか・・・

768 名前:132人目の素数さん投稿日:01/09/19 21:37
>>765
北極だけに白熊とか・・・

769 名前:132人目の素数さん投稿日:01/09/19 22:32
>>767
北極以外にもこのような点は無数に存在します。

770 名前:132人目の素数さん投稿日:01/09/19 22:50
>765
ひねくれた答えだと、赤道と平行な断面の円周がきっかり7000キロ
の地点(緯度は南北共に何度ぐらいかな?)から、きっかり3000キロ
北の地点。これだって赤道と平行な地球の断面全体を指すから、
765の答えは無限にあるな。だからその地点の緯度によく生息して
いる動物。

771 名前:132人目の素数さん投稿日:01/09/19 22:55
>770
>平行な地球の断面全体
平行な地球の断面の円周上全体、だった。スマソ。

772 名前:132人目の素数さん投稿日:01/09/20 00:18
以前もどこかで見たけど、「東に進む」というのは緯線に沿う等角コース
なのか出発点から見ての東向きの大圏コースなのかで話が変わるんじゃ?

773 名前:132人目の素数さん投稿日:01/09/20 00:29
>>770
>赤道と平行な断面の円周がきっかり7000キロ
>の地点(緯度は南北共に何度ぐらいかな?)から、きっかり3000キロ
>北の地点。
問題は地球にそんな地点が存在するかということ。

774 名前:132人目の素数さん投稿日:01/09/20 01:11
>773
北半球にはないけど、南半球にならあるっぽい。

775 名前:770投稿日:01/09/20 01:18
確かに北半球では北極点をぐるりと越えてしまいそうですな。
ということは南半球の円周7000キロ地点から3000キロ北上すれ
ばいい、ということで。
そこだったら北半球・南半球どっちになるんだろうか。

776 名前:132人目の素数さん投稿日:01/09/20 01:19
でも、7000キロだろうが100000キロだろうが1周以上していけないわけでなし(!
キロインチとかキロミクロンとか言い出したらイカンか?
#その前に>772は重大だぞ

777 名前:132人目の素数さん投稿日:01/09/20 03:48
みなさん、質問は小屋に何が飼われているかですぞ。
たとえ北極だろうが南半球だろうが別の惑星だろうが
そんなのは関係ないのでは。
ていうか問題がよくない。そんな答え、数学で解けるわけがない。

778 名前:132人目の素数さん投稿日:01/09/20 04:29
>>765


779 名前:132人目の素数さん投稿日:01/09/20 06:14
389をもっと一般にした問題.
x色の数珠n個をつかってネックレスを作る.
さて、何通り作れるか.
当然、回転と鏡映によって同じになるものは同じとみなす.

780 名前:132人目の素数さん投稿日:01/09/20 06:51
765です。本当に駄問で申し訳ありません。
>>764に自分なりに応えようと思ったのですが、、、

答えは北極なので、そり引きの犬とか、白熊とかで正解です。
メール欄にも書きましたが「数学じゃない」ですね。

781 名前:132人目の素数さん投稿日:01/09/20 07:08
つまんねー
なぞなぞらしい答えを待ってたのに…

782 名前:132人目の素数さん投稿日:01/09/20 07:57
馬鹿
十分になぞなぞだろう

783 名前:-投稿日:01/09/20 08:05
モロ、無修正画像サイト発見!

http://www.sex-jp.net/dh/01/
http://www.sex-jp.net/dh/02/
http://www.sex-jp.net/dh/03/
http://www.sex-jp.net/dh/04/

784 名前:132人目の素数さん投稿日:01/09/20 10:22
>>770の考えを借りると、ある緯度での周長/赤道長がその緯度のcosに
なるはずなのでcosx=7000/40000となるxを求めるとおよそ80度。南北に
は3000/20000×180=27度動くことになり求める緯度はおよそ南緯53度
あたり。

785 名前:132人目の素数さん投稿日:01/09/20 11:10
>782
だって北極点にいる動物は?

って答えが犬とか、白熊とか定まらないのはなぞなぞとしてやっていけない・・・

786 名前:770投稿日:01/09/20 16:55
>784
計算サンクス!
770の答えは穴だらけだとは分かっていたのだけれども
北極点としか思いつかない人がいるかもしれん、っていうか、
769の具体例を挙げたかったので。
南緯53度っていうと、どのあたりになるんだろう……。
地理はよく分からん。

787 名前:132人目の素数さん投稿日:01/09/20 17:45
(x-a)(x-b)(x-c)・・・(x-z)=?

788 名前:132人目の素数さん投稿日:01/09/20 18:10
>>787
0

789 名前:132人目の素数さん投稿日:01/09/21 06:37
log3とπ/3はどっちが大きい?
(logは自然対数、πは円周率)

790 名前:132人目の素数さん投稿日:01/09/21 07:31
>785
余裕の無い奴だな、別にお遊びなんだからいいじゃねーのか
所詮頭の体操なんだから、北極だと気が付けばそれでよし
そっから先はユーモアだろ
いつからなぞなぞは答えが定まってないといけなくなったのかね
簡単過ぎるがそれなりに面白いと思ったよ、もっと心にゆとりをな

791 名前:132人目の素数さん投稿日:01/09/21 21:42
>>789

π/3 - 1 = s ≒ 0.0472 とおく。 0.04 < s < 0.05
e ≒ 2.718 なので、 2.71 < e < 2.72

e^(log3) = 3
e^(π/3) = e / e(-s) < 2.72 / (1-s) < 2.72 / 0.95 ≒ 2.86 < 3

以上より、log3 > π/3

792 名前:791投稿日:01/09/21 21:44
e(-s) は e^(-s) の書き間違い。

793 名前:>785投稿日:01/09/21 23:33
北極点には亀がいて地球にフタをしていて
南極側は象さんが地球をささえてます。

北極点にいるのは亀です。

794 名前:名無し投稿日:01/09/21 23:56
『AB=11,AC=13の僊BCがある。AB上に点PをAP=9となるように,
AC上に点RをAR=7となるようにとる。また,BCの中点をQとする。
PQ=RQ,PQ⊥QRのとき,儕QRの面積を求めよ。』
小学生にも分かるような(*)解法でお願いします。
(*) 相似比,三角形の面積=底辺*高さ÷2 を使う程度。
面白くないって批判されちゃったけど,そうなんか?

795 名前:はなう投稿日:01/09/22 00:06
>>794
んなことないぞ。十分おもしろ。出典も有名だけどね。

796 名前:はなう投稿日:01/09/22 00:12
なんでsageてんだ漏れ・・

797 名前:766投稿日:01/09/22 00:17
>>795
そうなの?解法と出典をあわせて教えてください。

798 名前:はなう投稿日:01/09/22 00:20
>>797
向こうに書いたる

799 名前:専修生2投稿日:01/09/22 00:38
A,B,Cの3人が1回射撃をして命中する確率はそれぞれ
1/2,2/3,3/4である。
A,B,C・・・の順に射撃をして行き,命中されたものが順次
抜けていって,最後に残った1人を勝者とする。
各人が自分の優勝確率が最大になるように行動する(誰を狙うか決める)
とき,優勝する確率が一番高い人と,その優勝確率を求めよ。

800 名前:はなう投稿日:01/09/22 00:43
>>799これめんどくさいからキライ。優勝確率一番高い人は明らかにB

801 名前:132人目の素数さん投稿日:01/09/22 02:04
>790
だから、765の答えは北極だけじゃなくて無限にあるんだって。
770と776の答えを読んで理解しなさい。
……出題者も分かってなかった節があるが……。

802 名前:132人目の素数さん投稿日:01/09/22 02:38
>790
だから北極点にいる動物は?でなんかこうユーモアのある回答があれば
いいんだけど、犬でも白熊でもいいとなるとなぞなぞにはならんでしょ?
なんかこう駄洒落でもいいんだけど
北極にいる動物なら何でもいいではなくさ

803 名前:132人目の素数さん 投稿日:01/09/22 03:12
>799
例えば、Cは
外そうとしても4回に1回は命中してしまうのですか?
それとも外そうとしても4回に3回命中させてしまうということですか?

804 名前:132人目の素数さん投稿日:01/09/22 08:02
1桁の正の整数を4つ(重複可)選び、+、−、×、÷と括弧だけで
10になる式を作る。
例えば1、3、4、7なら1+3×(7−4)=10

今、4つの正の整数の最小値が2で、10がつくれなかった。
このとき4つの数は何か。

805 名前:K.S投稿日:01/09/22 12:19
πが無理数であることを示せ。 

806 名前:スロット投稿日:01/09/22 16:14
誰か解いて下さい!約0.37ぐらいになるんですが、解法が分かりません。
lim_[x→∞]f(x)
f(x)=(x-1/x)^x

807 名前:yanyan投稿日:01/09/22 17:34
>>806
f(x)=((x-1)/x)^x ですよね。
これは 1/e に収束します。なぜなら
f(x)=(1+(-1/x))^x で
(1+(a/x))^x は x を大きくするとき、
e^a に収束するからです。

808 名前:803投稿日:01/09/23 00:25
>799
わざと当てないで一発使うことも可能かどうかが聞きたいのです。

809 名前:132人目の素数さん投稿日:01/09/23 02:15
2,2,5,7

810 名前:799投稿日:01/09/23 02:55
>>808
遅くなってすみません。もちろん,O.K.です。

811 名前:132人目の素数さん投稿日:01/09/23 17:49
あるホテルの受付では帽子をあずかります。
しかしこの受付の人はおっちょこちょいであずかった帽子を
でたらめに返してしまいます。
さてあずけた帽子が2人3人4人5人の時
最低1人はただしく受け取る場合の確率を高い方から順にならべてください
発展問題:N人の場合の式をつくりましょう

812 名前:132人目の素数さん投稿日:01/09/23 20:09
>805
1+1+1/2!+1/3!+…+1/n!<e<1+1+…+1/n!+e/(n+1)!
を使う.
eを有理数と仮定すれば、その(分母)=nとおく.(n>1)
全辺にn!をかければ明らかに矛盾

813 名前:名無し投稿日:01/09/24 14:43
>>811
モンモールの問題!
その受付は,おっちょこちょいなのではなく,単なる嫌がらせやろうなのです。

814 名前:132人目の素数さん投稿日:01/09/26 22:53
アメリカ人、ロシア人、タリバンの親子が連れたって川にさしかかると、
2人乗りのイカダが1つありました。子ども3人は恐れました。
子どもは自分の親がそばにいないと、他の親にマシンガンで撃たれてしまうのです。
しかし、子どもでもイカダをこげます。
さあ、この6人、川を何回横切れば無事全員が向こう岸にたどり着く
ことができるでしょう。

-------------
いや、答は言わなくていい・・・ネタとして鑑賞してくれ(w

815 名前:KARL ◆gjHKPQSQ 投稿日:01/09/27 02:35
任意の自然数nをとり、その正の約数を書き並べる。
たとえばn=12とすると、1,2,3,4,6,12という列が出来る。
次にこの列の要素のそれぞれの正の約数の個数を書き並べる。
上の例だと、1,2,2,3,4,6となる。
この最後の数列の要素のそれぞれを3乗したものを合計する。
1^3+2^3+2^3+3^3+4^3+6^3=1+8+8+27+64+216=324
これは何と、この数列の要素を合計したものの2乗に等しい。
(1+2+2+3+4+6)^2=18^2=324
このことが一般に成り立つことを証明せよ。

816 名前:132人目の素数さん投稿日:01/09/27 02:48
>>815
n=Πp(i)^e(i)とすると数列に表れる数は
Πa(i)(1≦a(i)≦e(i)+1)となることと
1以上m以下の整数の3乗の和が
1以上m以下の整数の和の2乗であることから証明できる。

817 名前:132人目の素数さん投稿日:01/09/27 07:10
>>814
子供二人だけでイカダに乗せるのは危ない。

818 名前:132人目の素数さん投稿日:01/09/28 04:24
半径1の円に内接する正十七角形
の面積を求めよ

819 名前:132人目の素数さん投稿日:01/09/28 04:43
1,2,3,4,5,6,7,8,9の数と加減乗除の記号を用いて100を作れ、というのは小町算として有名であるが、
指数も使ってよいとしたらいくつあるだろうか。

また100だけでなく、1000や10000などの10^nに拡張しても解はあるだろうか。

820 名前:132人目の素数さん投稿日:01/09/28 10:34
2cos(2π/7)=(-1+(7(2+ω))^(1/3)+(7(2+ω^2))^(1/3))/3
を示せ

821 名前:化学投稿日:01/09/29 14:35
海水に最も多く含まれている物質の物質名または化学式を答えよ

822 名前:132人目の素数さん投稿日:01/09/30 19:15
age

823 名前:132人目の素数さん投稿日:01/09/30 20:52
>821
H2O ですがなにか?

824 名前:821投稿日:01/10/01 00:15
>>823
正解ですが何か?

825 名前:132人目の素数さん投稿日:01/10/01 22:13
0〜1の実数を適当にn個選んだ時のk番目に大きい数の期待値は?

826 名前:132人目の素数さん投稿日:01/10/02 00:10
>>825
「適当に」ってどうやって選ぶんだ?

827 名前:132人目の素数さん投稿日:01/10/02 03:40
>>825
k番目におおきいあたいをあらわす確率変数をX[k],X[0]=0,X[1]=1とするとき
E(X[k+1]-X[k])=1/(n+1)を利用するやつね。

828 名前:825投稿日:01/10/02 15:09
>>826
やっぱり「適当に」ってのを正確に定義しなきゃ駄目ですか?
アバウトな感じで済ませたかったんですけど…
>>827
>X[1]=1
ってのはX[n+1]=1ですよね?
自分は面倒くさいやり方使っちゃったんで827さんのやり方で正解ということで。

829 名前:132人目の素数さん投稿日:01/10/03 22:22
age

830 名前:132人目の素数さん投稿日:01/10/07 01:45
2より大きいすべての偶数は、2つの素数の和で表わせることを証明してください。

831 名前:プレスリー投稿日:01/10/08 22:29
3は素数ですが,どうやって2つの素数の和にできるのかな?

832 名前:投稿日:01/10/08 22:31
3は偶数ですか?

833 名前:132人目の素数さん投稿日:01/10/08 23:43
>>830 4は? 1+3?  1は素数じゃないぞ

834 名前:おちこぼれ投稿日:01/10/09 00:01
ゼータ関数の s=2 は π2乗/6 ですよね。

これを三角逆関数の無限級数展開を使って、すっきりと解く方法があります。
実は私も大学時代に明らかに解いた記憶はあるんだけど、
解く方法を忘れちゃいました。
一生懸命思いだそうとしてるんだけど、だめなんです。
もうとしだなあ。

だれか教えて下さい。
(ただしここに書き込むのはたいへんだから、どんな関数を展開して、どんな特異解かを教えて下さい)
よろしく!

835 名前:132人目の素数さん投稿日:01/10/09 00:03
>>833
2は素数じゃないとでも?
異なる素数の和とは書いてないが?

836 名前:132人目の素数さん投稿日:01/10/09 00:10
>>835
最初からそう書けヴォケ

837 名前:132人目の素数さん投稿日:01/10/09 00:12

836 名前:132人目の素数さん 投稿日:01/10/09 00:10
>>835
最初からそう書けヴォケ

838 名前:132人目の素数さん投稿日:01/10/09 00:13
837 名前:132人目の素数さん :01/10/09 00:12

836 名前:132人目の素数さん 投稿日:01/10/09 00:10
>>835
最初からそう書けヴォケ

839 名前:132人目の素数さん投稿日:01/10/09 00:13
838 名前:132人目の素数さん :01/10/09 00:13
837 名前:132人目の素数さん :01/10/09 00:12

836 名前:132人目の素数さん 投稿日:01/10/09 00:10
>>835
最初からそう書けヴォケ

840 名前:132人目の素数さん投稿日:01/10/09 00:14
>>836
異なると書いてなければ異なる必要はない。

841 名前:おちこぼれ投稿日:01/10/09 00:21

>833

 その問題の解答はどこにでもあるよ。

842 名前:132人目の素数さん投稿日:01/10/10 17:44
>>841
ゴールドバッハ予想って解決したんですか?
さらにどこにでも証明方法があるとは…

843 名前:おちこぼれ投稿日:01/10/10 22:41
「ゴールドバッハ予想」はたぶんまだ解決してないと思う。(最近数学誌や論文を読んでないので自信ない)

「どこにでも証明方法があるよ」って言ったのは、この問題は数論の中ではあたりまえに出てきて証明も載ってると思う。

844 名前:132人目の素数さん投稿日:01/10/10 23:34
>>843
証明って何の証明?

845 名前:132人目の素数さん投稿日:01/10/10 23:48
√2を2進法で表したときに小数点n桁目までに
1が出てくる回数をf(n)回とする。

lim(n→∞)f(n)/nは収束するか?収束するのならその値を求めよ。

846 名前:KARL ◆gjHKPQSQ 投稿日:01/10/11 02:48
すでででごめん。

下に述べるような生成規則による次の二重数列を考える。

1 1 1 1 1 1 1 1 1 1 1 1 1 ...
1 2 3 4 5 6 7 8 9 10 11 12 13 ...
1 2 4 7 11 16 22 29 37 46 56 67 79 ...
1 2 4 8 15 26 42 64 93 130 176 232 299 ...
1 2 4 816 31 57 99 163 256 386 562 794 ...
.... ずっずれてる(^_^;
生成規則:
 a) 1行目と1列目はすべて1とする。
 b) それ以外の各項はその左の項xと、xの上の項yの和とする。
   つまり、
 y
 x □
こういう配置のとき□=x+y とする。
このとき、第1行第2列の1から桂馬とびに第2行第4列、第3行第6列、、、と
項を拾っていくと、4の何とか乗となっている。このことを証明せよ。

「わからない問題は、どんなものでも俺に聞け!」スレッドの先生が答えて
くださらなかったもので...もっとも自作問題なので「わからない問題」で
はないのを見抜かれたのかもしれない。とにかく面白い問題でしょ。

847 名前:132人目の素数さん投稿日:01/10/11 03:44
>>846
農{m=0〜n-1}{2n-1}Cm=2^{2n-2}
かな(記号が・・・)

848 名前:KARL ◆gjHKPQSQ 投稿日:01/10/12 01:12
>>847
nCmをC(n,m)と書くことにします。

Σ[m=0 to n-1]C(2n-1,m)
=C(2n-1,0)+C(2n-1,1)+C(2n-1,2)+...+C(2n-2,n-1)
=2^(2n-2)

という意味ですね。この式の左辺はどのようにして出てくるのでしょう。
またこの等式はどのように証明されるのでしょう。

849 名前:847投稿日:01/10/12 01:40
次の様に書き直すと見やすいかな?
1 1 1 1 1 1 1 1 1 1 1 1 1 ...
1 2 3 4 5 6 7 8 9 10 11 12 13 ...
1 2 4 7 11 16 22 29 37 46 56 67 79 ...
1 2 4 8 15 26 42 64 93 130 176 232299 ...
1 2 4 8 16 31 57 99 163 256 386 562 794 ...

各数字は「その地点から上か左に歩き、いずれかの列の左端に行き着く方法」
の総数に等しい(厳密には帰納法)。
これよりn列2n行の数字は
Σ[m=0 to n-1]C(2n-1,m)=Σ[m=0 to 2n-1]C(2n-1,m)/2=2^{2n-2}

850 名前:847投稿日:01/10/12 01:44
失敗・・・苦肉の策・・・
* * * * 01 01 01 01 001 001 001 001 001 001 001 001 001 ...
* * * 1 02 03 04 05 006 007 008 009 010 011 012 013 ...
* * 1 2 04 07 11 16 022 029 037 046 056 067 079 ...
* 1 2 4 08 15 26 42 064 093 130 176 232 299 ...
12 4 8 16 31 57 99 163 256 386 562 794 ...

851 名前:132人目の素数さん投稿日:01/10/12 08:12
問題

   8本の連続した直線ですべての点を結んでください。
  ただし途中で引き返したり同じ点を二度通ってはいけない。

      ・   ・   ・ ・   ・
    
    
      ・   ・   ・ ・   ・


      ・   ・   ・ ・   ・


・   ・   ・ ・   ・


・   ・   ・ ・   ・

852 名前:132人目の素数さん投稿日:01/10/12 08:25
ミスった
   ・  ・  ・  ・  ・
 
   ・  ・  ・  ・  ・
 
   ・  ・  ・  ・  ・
 
   ・  ・  ・  ・  ・
 
   ・  ・  ・  ・  ・

853 名前:132人目の素数さん投稿日:01/10/18 20:04
>>851
解けたよ、多分。
アタック25と同じ数字配列として、八本の線は次の順に点を通る。

1:24,23,22,21
2:16,12,8,4
3:5,10,15,20
4:25,19,13,7,1
5:2,9
6:14,18
7:17,11
8:6,3

合ってる?

854 名前:132人目の素数さん投稿日:01/10/18 20:22
アタック25なんかより行列成分の添え字と同じように行番号列番号で書いた方がわかりやすい

855 名前:132人目の素数さん投稿日:01/10/18 20:51
>>852
7本でできない限り、このパズルはつまんない。
8本だと答えのバリエーションがありすぎる。

856 名前:132人目の素数さん投稿日:01/10/18 21:02
>>853 これって15本じゃないか?

857 名前:132人目の素数さん投稿日:01/10/18 21:10
正方行列ではある次数をこえると対称性が現れる、この最小次数を
求めよ。

答え 11次。

こころ  $a_{111}$

858 名前:855投稿日:01/10/18 21:09
>>185-855
ふと思いついたので 予言(予想)します。
『NxN個の配列の場合、線の数は 2N−2が最小』
--------------------------------------
反例・反証 求む!

859 名前:132人目の素数さん投稿日:01/10/18 23:16
>>858
>反例・反証 求む!
つまり最初から成り立たないと思っているわけね。

860 名前:132人目の素数さん投稿日:01/10/18 23:38
>>855
> 8本だと答えのバリエーションがありすぎる。
それ教えて。
俺思いつかんわ

861 名前:855投稿日:01/10/19 00:42
1: 25-19-13-7-1
2: -2-3-4-5
3: -10-14-18-22
4: -21-16-11-6
5: -8-9
6: -15-20
7: -24-23
8: -17-12

1: 25-19-13-7-1
2: -2-3-4-5
3: -10-15-20
4: -24-18-12-6
5: -11-16-21
6: -22-14
7: -9-8
8: -17-23    ・・・など

862 名前:853=860投稿日:01/10/19 01:12
>>861
問題文読み直し

863 名前:855投稿日:01/10/19 05:29
>>862
逝きました

864 名前:852投稿日:01/10/19 08:32
>>853 正解
>>858 N=2だと成立しないじゃないか。

865 名前:晴れのちうぐぅ投稿日:01/10/19 12:52
問題です。
a,b,cはabc=1を満たす実数、nは2以上の自然数とする。
このとき、以下の不等式が成立することを示せ。

1/a^n(b+2c)+1/b^n(c+2a)+1/c^n(a+2b)≧1

あまりひねりの利いてない問題ですいません。

866 名前:なし投稿日:01/10/19 13:12
>>865
曖昧な書き方だなあ。(b+2c)はどこに作用している?

867 名前:晴れのちうぐぅ投稿日:01/10/19 13:54
>>866
(aのn乗)×(b+2c)の逆数と(bのn乗)×(c+2a)の逆数と
(cのn乗)×(a+2b)の逆数の和が1以上であることを証明して
くださいという意味です。条件に関する訂正ですが、a,b,cは正
の実数であるという条件を追加します。

868 名前:132人目の素数さん投稿日:01/10/19 15:20
>>858
3≧N について 証明完了

869 名前:132人目の素数さん投稿日:01/10/20 22:42
同じ点を二度通っちゃいけないだけなのね…

『線が折れている点以外では2つの線が交わらない』
って条件を入れると2N-2は不可能だよね?

870 名前:132人目の素数さん投稿日:01/10/21 06:02
4x4板のオセロの必勝法を記述せよ。
6x6板のオセロの必勝法を記述せよ。

871 名前:132人目の素数さん投稿日:01/10/22 07:45
いいやまず2×2板のオセロの必勝法を記述してもらおう。

872 名前:132人目の素数さん投稿日:01/10/22 07:50
>871
解無し

873 名前:132人目の素数さん投稿日:01/10/22 12:16
>>871
棋譜
-------
-------
以上 2-2 引き分け
と出ましたが何か?

874 名前:132人目の素数さん投稿日:01/10/22 12:17
a^3-a^2=?

875 名前:132人目の素数さん投稿日:01/10/23 00:05
age

876 名前:876投稿日:01/10/23 00:24
論理学の先生が生徒であるあなたに言いました。

「私は100円玉と10円玉を持っています。
これから私がすることをあなたが見事に言い当てたら、
100円玉か10円玉のどちらかをあげます。
はずれたらどちらもあげません。
私は矛盾するような行動は決してとらないことを約束します。
またどうしても矛盾せざるをえないような予想はしないでください。」

さてあなたはなんと予想すべきか?

この問題はかなり奥が深く面白いです。じっくり考えてみて下さい。
(答えを知ってる人はあんまり早く答え書かないでね。)

877 名前:数学じゃないけど・・・投稿日:01/10/23 01:10
有名な問題ですが…
A、B、C、Dの4人がクイズに挑戦します。
ただし、間違えた人は殺されてしまいます。
4人のうち2人は赤の帽子、残る2人は白の帽子をかぶっていますが、
自分の帽子の色はわかりません。
クイズというのは、自分の帽子の色を当てるというものです。
A 壁 B C D
という順に並んでいます。
Aは隔離されているので誰からも見られないし、誰を見ることもできません。
Bからは壁だけが見えます。
CからはBが見えます。
DからはB、Cが見えます。
4人は赤2つ、白2つという情報だけをもっています。
A→赤、B→白、C→赤、D→白
の帽子をかぶっているのですが、少ししてから
自分の帽子の色を当てた人がいます。それはだれでしょう?
理由も。あてずっぽうだったとかはだめです。
もしはずれたら殺されるのでみんな慎重です。

878 名前:数理くるめ投稿日:01/10/23 01:29
>>876
「先生は僕に10円玉をくれない」かな?

879 名前:876投稿日:01/10/23 01:38
>「先生は僕に10円玉をくれない」かな?

ざんねん。はずれです。

880 名前:876投稿日:01/10/23 01:39
何がどうハズレなのかは敢えて説明しません。
それを言っちゃうとかなり絶大なヒントになってしまうから。
もう少し考えてみてください。

881 名前:132人目の素数さん投稿日:01/10/23 01:45
>>877
Cかな。
理由はDが即答しなかったから。
つまりDから見ると赤と白の帽子が見えていると
Cにはわかることになる。
CにはBが白をかぶっているのはわかるから
自分が赤だとわかる。

ってとこでどうでしょう?

882 名前:21世紀投稿日:01/10/23 01:58
マッチ棒を6本使って正三角形を4つ作ってください。

883 名前:132人目の素数さん投稿日:01/10/23 02:09
正四面体。

884 名前:132人目の素数さん投稿日:01/10/23 02:11
>「先生は僕に10円玉をくれない」かな?
>ざんねん。はずれです。

なんで違うんだゴルァ!

885 名前:132人目の素数さん投稿日:01/10/23 02:24
数学とはちょい違うかもしれないけど、1つ。

「日本には上り坂と下り坂、どっちが多い?」

886 名前:876投稿日:01/10/23 02:24
>884
もっといい解答があるってことですよ。
俺は今日この問題知ったんだけど、
答え聞いたときはまじでびびったよ。

887 名前:132人目の素数さん投稿日:01/10/23 02:26
>885
逆から見れば上り坂は下り坂。
同じ。

888 名前:132人目の素数さん投稿日:01/10/23 02:29
>>887
あたりです。
くだらない問題、スイマセンでした。

889 名前:132人目の素数さん投稿日:01/10/23 02:34
>885
その問題、小学校に入りたての頃、小学館の学年誌についてた
ドラえもんのなぞなぞ?のような付録で見たぞ

890 名前:132人目の素数さん投稿日:01/10/23 05:31
することを当てるのにくれないという答はセーフですか?
「少なくともどちらか片方はくれない」
いや100円ゲットしたくて878なんだろうなー
「10円玉は持ったまま何かする」
何かって何?

891 名前:132人目の素数さん投稿日:01/10/23 09:55
>876
「先生は私に10万円金貨と100円玉をセットでくれるか、
なにもくれないかのどちらかであろう」は如何?

892 名前:132人目の素数さん投稿日:01/10/23 12:19
or条件ってありなのかな?
そしたら
>>891でも良さそうな気がするけど。
なんでおまけに10万円金貨がついてるのか分からんが。

893 名前:892投稿日:01/10/23 12:27
とにかく、
・外したら何もくれない
・矛盾した行動は取らない
んだから、
「外れたときに何か貰える」条件を付ければ
それは矛盾するので先生は選択できなくなる。

つまり、
「外れたときに何か貰える」→「先生は私に何もくれない」
という条件をorで付加すれば答えになるよなぁ。

でもこれじゃ何か面白くない。

もっと良い答えがあるんでしょ?

894 名前:892投稿日:01/10/23 12:46
じゃ、簡単で有名な問題を一つ。
とある地域に住んでいる原住民には2つの部族があり、
絶対に本当のことしか言わない「正直族」と
絶対に本当の反対のことしか言わない「嘘吐き族」がいました。
両族の見た目の区別はできません。

あるとき、2人の原住民と会いました。
もちろん、どちらがどの部族か分かりません。
2人に共通の1つの質問をして、2人の部族を見分けてください。

895 名前:877投稿日:01/10/23 14:27
>>881
素晴らしい。正解っす。
すぐに解かれるとは悔しい・・・
おいらは1時間くらいかかったよ。

896 名前:892投稿日:01/10/23 15:31
>>895
つーかがいしゅつ

897 名前:881投稿日:01/10/23 15:50
>>895
正解でしたか。やったー。

>>896
がいしゅつでしたか。
過去ログ読んでなかったんで・・。

898 名前:ぶんぶん科学省投稿日:01/10/23 16:46
∠A=45°の鋭角僊BCがあり,その外接円の半径は√2である。
(1) BCの長さを求めよ。
(2) 僊BCの垂心をHとする。線分AHの長さを求めよ。

899 名前:892投稿日:01/10/23 20:45
>>877,881

問題の条件設定に難ありと思われる。
Cは「DがBとCを見れる」という情報を持ってないと、
その答えに行きつかない。

以前にも書かれていたことなので、>>167以降をご覧あれ。

900 名前:881投稿日:01/10/23 21:26
>>892
過去ログをざっと読みました。
おっしゃるとおりですね。
そこまで深く考えていなかったです。
まだまだ修行がたりませんな。

901 名前:>499投稿日:01/10/23 22:37
>>499
問)
「全ての生徒はペンを持っている。」の反対の意味を持つ文を書け。

「ある生徒がいてペンを持っていない」は否定。
反対の意味ならば
「全ての生徒はペンを持っていない」が正解。
否定と反対は違う。その不正解にされた98人は可哀想。

902 名前:877投稿日:01/10/23 23:01
>>899
以前すでに出てたんですね…
ざっと見ていたつもりだったんですが・・・
すみません。これからは注意します。

903 名前:132人目の素数さん投稿日:01/10/24 00:06
全ての項が3桁の整数である等比数列の内、最も多くの項を含むものを答えよ。

小学生なら答えだけ、中学生以上は証明も。

904 名前:数理くるめ投稿日:01/10/24 00:15
>>876
「奥の深い答え」をそろそろ教えてくれませんか?
ちなみに>>884さんは私ではないです。

905 名前:132人目の素数さん投稿日:01/10/24 00:21
>>877は初めてみた。
ただ、この系統の考え方は有名だね。
「3人に黒の帽子を被せて一つの部屋に入れた。
3人には『誰か一人の帽子は黒であり、また黒と白以外の帽子は存在しない』
と説明し、自分の帽子の色が分かったら部屋から出ても良いとした。
すると3人は同時に部屋を出たが、この時の3人の考えを説明せよ。
ただし、当然ながら3人とも自分の帽子は見えず、残る2人の帽子は見える」
とか。

906 名前:132人目の素数さん投稿日:01/10/24 00:31
>>905
パズルの本質とは外れるけど、この種の問題を見ると、いつも
「登場する3人の知能(推理力)が、そこそこ同程度ってことが大前提なんだな」と思う。
そうじゃなければ、ヘタすれば、全員連鎖的に死刑になっちまう。

907 名前:132人目の素数さん投稿日:01/10/24 00:47
>>906
ワラタヨ。
確かに、一人ボケがいるとやばいわな(w

908 名前:132人目の素数さん投稿日:01/10/24 01:15
age

909 名前:892投稿日:01/10/24 01:32
>>905
> 3人には『誰か一人の帽子は黒であり、また黒と白以外の帽子は存在しない』

黒3白2って設定じゃないと解けないと思うけど・・・

910 名前:892投稿日:01/10/24 01:38
念のため回答を

Cが「自分が白」だと仮定し、Bの立場になって考える。
B「Cは白だ。もし自分が白ならAは逃げるはず、でも逃げないということは自分は黒だ」
つまり、Cが白ならばBは逃げられるはずだ。でもBは逃げなかった。
つまり、Cの仮定は正しくなかった、つまり、Cのかぶっている帽子は黒である。

三人が同時にこの結論にたどり着けば、揃って脱出できる。

911 名前:892投稿日:01/10/24 01:39
>>905
よく考えたら「誰か一人が黒」でも同じ事だった。
連投スマソ

912 名前:132人目の素数さん投稿日:01/10/24 03:18
age

913 名前:132人目の素数さん投稿日:01/10/24 03:48
帽子の問題は 668 684 691 693 でも出てるよ。

914 名前:876投稿日:01/10/24 13:07
891は惜しいねー。
>「先生は私に10万円金貨と100円玉をセットでくれるか、
>なにもくれないかのどちらかであろう」は如何?
俺が先生だったらハズレって言いながら1円あげちゃう。
ハズレの場合「10円玉と100円玉のどちらもあげない」だよ。
問題文をよく見てね☆

915 名前:晴れのちうぐぅ投稿日:01/10/24 13:08
大数の宿題をヒントに作った問題です。

自然数nに対して、n×{5^(1/2)}の小数部分をA(n)とします。
このとき、任意の異なる自然数i,jについて以下の不等式を満たす0より大きい
定数Cが存在することを証明してください。

     |(i−j){A(i)−A(j)}|≧C


関係ないけど、青木亮二の問題はなかなかいけてると思いますが皆さんはどう?

916 名前:876投稿日:01/10/24 13:10
んじゃ答えいくねー。

>論理学の先生が生徒であるあなたに言いました。
>「私は100円玉と10円玉を持っています。
>これから私がすることをあなたが見事に言い当てたら、
>100円玉か10円玉のどちらかをあげます。
>はずれたらどちらもあげません。
>私は矛盾するような行動は決してとらないことを約束します。
>またどうしても矛盾せざるをえないような予想はしないでください。」
>さてあなたはなんと予想すべきか?

答えは「先生は10円玉、100円玉、1億円のいずれも私にくれない」です。
この答えすばらしくない?
俺むちゃくちゃ感動したよ。

917 名前:892投稿日:01/10/24 13:36
>>916
これなら確実に1億円が貰えるって寸法かぁ。
「貰えるお金の最大値=100円」で思考が止まってしまったら負けなんだな・・・
なるほど、こりゃ目から鱗が落ちた。

でもどうせなら1兆円とかにしようよ(w

918 名前:132人目の素数さん投稿日:01/10/24 13:37
>>916
すまん,どう感動すれば良いか,厨房のおれにも教えちくり。

919 名前:892投稿日:01/10/24 13:48
>>918
私が先生の行動を言い当てたと仮定した場合:
・言い当てより、先生は私に10円も100円も1億円もくれない。
・前提条件より、言い当てたら10円玉か100円玉のどちらかをくれる。
この2条件は矛盾するので、先生は私の言い当て通りに行動できない。
つまり、私の言い当ては必ず外さざるを得ない。

で、言い当てが外れた場合:
・言い当ての否定より、先生は私に10円か100円か1億円をくれる。
・前提条件より、言い当てが外れたら10円玉も100円玉も貰えない
この2条件を満足するのは、先生が私に1億円くれる場合だけ。
従って、先生は私に1億円をあげざるを得ない。

こんなところで理解できますか?
ドモルガンは知ってる?

920 名前:918投稿日:01/10/24 13:49
ああ,なるほど,でもちょっとなっとくできないな。

921 名前:918投稿日:01/10/24 13:52
>>919
あ,なっとくできたかもしんない

922 名前:892投稿日:01/10/24 13:54
ちょっと訂正
・言い当ての否定より、先生は私に10円、100円、1億円のうち少なくともどれか一つはくれる。

923 名前:918投稿日:01/10/24 14:06
なぜちょっと納得できないといったかというと,
>これから私がすることをあなたが見事に言い当てたら、
の解釈なんだけど,「これから私がすること」というのは,この論理学の先生が
問題を出し,生徒がそれに答え,その次に何かこの先生が行動を取る,例えば,
「1円を生徒にくれた」とすると,ここで,生徒が予想すべき範囲は終わり,と
思えちゃうところです。だから,「1円を先生がくれ」て,生徒の予想があたり,
「その後先生が10円くれる」のは先生の言う「これから私がすること」の範囲に
含めないのが普通の解釈なのではないかと,私が早合点した事だな。
でも,日常ではそう解釈してもおかしくないよね。

924 名前:876投稿日:01/10/24 14:16
>923
あー確かにそれは正論だなー。
それに関しては
「お前、論理パズルに慣れてねーよ!」
って言うしかないかなあ。
ごめんなさい。悪気はないです(w

925 名前:918投稿日:01/10/24 14:32
>「お前、論理パズルに慣れてねーよ!」
>って言うしかないかなあ。
ごめん,その通り。それに一言いうの忘れてた。
”私も答えを聞いて感動しました。”

(誤解しちゃうような茶々
>>923でスマソ)

926 名前:876投稿日:01/10/24 14:52
俺の読んだ本(最近出版された本)では、
こういった論理は脅迫論理と名付けられています。
著者の甥かなんかが最近発見したそうです。
どうりで論理パズル好きの俺が知らない問題なわけだ。

それはともかく、脅迫論理ナンパ応用編。(笑)

「ねえねえ彼女〜。俺が今から君のする事を当てたら拍手してよ。
外れたら拍手しないでね。い〜い?」
「いいよ〜」
「うんとね〜。君は俺に拍手もキスもしない! どうだ?」
「・・・・チュ♪」

欠点:ナンパされてすぐキスするような女は、この論理を理解してくれない(w

927 名前:132人目の素数さん投稿日:01/10/24 15:06
>>926
>>917と同じような感想を抱いたぞ。
「どうせなら○○とかにしようよ」(藁

928 名前:名無しの歌が聞こえてくるよ♪投稿日:01/10/24 15:25
森永乳業の社長が100万円を三井信託銀行に100ヶ年満期の預金をしたが、
半年で3.94%の利子がつくとしていくらになるか。

929 名前:132人目の素数さん投稿日:01/10/24 15:25
7:7
3:3
1:1

930 名前:132人目の素数さん投稿日:01/10/24 15:28


931 名前:132人目の素数さん投稿日:01/10/24 15:54
>>923=918
「先生は私が予想した解答を聞く」
だったら、そういう解釈でも対応できる気はするけど。
解答するまでに何らかの行動をとられたら駄目だし、
あたっても10円か100円というのもメリットはないか・・・。

932 名前:132人目の素数さん投稿日:01/10/24 17:21
>>525の砂時計の答えは
7分砂時計をリセットしないと出来ないんじゃ?
初めの2分を計った時で7分砂時計は2分と5分に分かれてる状態

933 名前:932投稿日:01/10/24 17:22
あれ?アホダ逝ってきます

934 名前:132人目の素数さん投稿日:01/10/24 23:31
>>916感動age

935 名前:892投稿日:01/10/24 23:44
926のように単純化すると分かりやすいよなぁ。

936 名前:KARL ◆gjHKPQSQ 投稿日:01/10/25 01:15
3角形ABCの内接円をOとします。円Oと3辺BC,CA,ABとの接点を
P,Q,Rとします。線分AQ,AR,円Oに同時に接する円と円Oとの接点をL,
線分BR,BP,円Oに同時に接する円と円Oとの接点をM,線分CP,CQ,円O
に同時に接する円と円Oとの接点をNとします。このときPL,QM,RNは一点で
交わることを証明して下さい。

937 名前:132人目の素数さん投稿日:01/10/25 02:12
>>894
の回答教えてください。

938 名前:132人目の素数さん投稿日:01/10/25 02:20
>>937
「貴方はウソをつきますか?」と聞かれたら、正直族も嘘つき族も
「いいえ」と答える。
ここで二つの族の見解が一致しているのでそれを利用する。

「「貴方はウソをつきますか?」と聞かれたら貴方は「いいえ」と答えますか?」
と聞くと、正直族は「はい」、嘘つき族は「いいえ」と答える。
・・・というのが本来の解答。

でも、よく考えたら自分の髪の毛の色を聞けばいい。
「僕の髪の毛は黒いですか?」と聞けば・・・

939 名前:132人目の素数さん投稿日:01/10/25 04:56
894の問題は『頭の体操』(出版社ど忘れ)とかいうのに収録されてる。
でもこの挿し絵がかなり不気味でよなか一人でみるものじゃなかった(藁
スレと関係ないのでsage

940 名前:892投稿日:01/10/25 08:05
>>938
いろんな答えが考えられるけどね。
別解。
「貴方の隣に要る人は、貴方と同じ種族ですか?」

941 名前:892投稿日:01/10/25 08:19
「頭の体操」で思い出したけど、
昔、この手のクイズを集めた「IQエンジン」という問題と回答を放映するだけの番組があった。
「頭の体操」の問題がかなり出展として使われていた。とんち系の問題もかなりあったけどね。

オープニングのキャッチ「夜中ですが、頭をお使いください」が好きだった。

久々にもう一度みたいなぁ・・・

942 名前:数理くるめ投稿日:01/10/25 10:28
>>876
解答さんくす。
>さてあなたはなんと予想すべきか?
を「どうすれば100円玉をゲットできるか」
と解釈したのがいけませんでした。

943 名前:132人目の素数さん投稿日:01/10/26 12:59
直線4本の一筆書きで全ての点を通過せよ。

 ・   ・ ・   ・
   ・     ・
 ・   ・ ・   ・

解答用の番号は
 1 2 3 4
  5   6
 7 8 9 10
とする。

944 名前:132人目の素数さん投稿日:01/10/26 13:00
あ、ずれてる。スマソ。サイコロの5が二つ並んでると考えてね。

945 名前:132人目の素数さん投稿日:01/10/26 13:24
>>943
直線14と76の交点をaとする。
1-2-3-4-a-6-7-8-9-10-5

946 名前:132人目の素数さん投稿日:01/10/26 13:26
>>944
回答1
1:[1]から開始し、[2][3][4]を通過。このままもう少し伸ばす。
2:そこから左下に[6]を通過し、[7]まで線を引く。これが二本目。
3:[7]から[8][9]を通過し、[10]で止める。
4:そこから左上に[5]目指し最後の直線。

回答2:
遠慮無く且つ豪快にそれでいて正確に,太い直線を使用。そぅすれば直線一本

947 名前:132人目の素数さん投稿日:01/10/26 13:27
正解

948 名前:KARL ◆gjHKPQSQ 投稿日:01/10/27 01:43
1から2nまでの整数の中からn+1個の整数を任意に選び出す。このn+1個の整数の中に
は、どちらか一方が他方の約数になっているようなペアが必ず存在することを証明せ
よ。

949 名前:132人目の素数さん投稿日:01/10/27 01:47
>>948
1−2−4−8−...
3−6−12−...
5−10−20−...
7−14−28−...
9−18−36−...
...。

950 名前:はなう投稿日:01/10/27 01:55
>>949
なるほど。とてもわかりやすい。

951 名前:KARL ◆gjHKPQSQ 投稿日:01/10/27 21:50
それぞれの列(行というべきか)を1つの箱と考えるわけですね。
列の数はn,選ぶ数はn+1だから2つの数をふくむ列が必ず存在する、
というわけか。なるほど...でもすぐにはわからなかったよ。とほほ。

952 名前:KARL ◆gjHKPQSQ 投稿日:01/10/27 22:16
古人いわく「魔方陣は数学の基礎だ」なんちって...
けっこう知られている問題だったらごめんなさい。

*ABC
*DEFG
IJKLM
*NOPQ
*RST

上のアルファベットのある位置に1から19までの数字を書き入れて
A+B+C=D+E+F+G=I+J+K+L+M=N+O+P+Q
=R+S+T=A+D+I=B+E+J+N=C+F+K+O+R
=G+L+P+S=M+Q+T=C+G+M=B+F+L+Q
=A+E+K+P+T=D+J+O+S=I+N+R
となるようにせよ。正解ちゃんとあります。

953 名前:KARL ◆gjHKPQSQ 投稿日:01/10/27 22:21
恐れていた事態がおきた。ずれた。
思い切って、これで
*ABC
*DEFG
*IJKLM
*NOPQ
*RST
どうだ。

954 名前:132人目の素数さん投稿日:01/10/27 23:42
>>949-951
証明問題は厳密に書かないとだめなのではないでしょうか。
948の内容自体は当然のことですが、それをいかにして説明するか、ということで。
n列目までの中に2nまでの全ての整数が含まれることはどう証明するのでしょう。

955 名前:132人目の素数さん投稿日:01/10/27 23:46
>>954
では
n列目までの間に2nまでのすべての整数が現れることと
2度現れる数が存在しないことを証明した後
引き出し論法(鳩の巣原理)でOKかな?

956 名前:132人目の素数さん投稿日:01/10/27 23:50
>>954
1以上2n以下の奇数はn個。

957 名前:132人目の素数さん投稿日:01/10/28 01:29
>2度現れる数が存在しないことを
ん?なんで?
2度でも3度でも現れるよ。

958 名前:132人目の素数さん投稿日:01/10/28 14:45
>>957
949は奇数*2のn乗って形だよ。
奇数*nって形じゃないよ

959 名前:132人目の素数さん投稿日:01/10/28 17:10
>>955
同じ数が2回以上でないことの証明はいらない。

960 名前:132人目の素数さん投稿日:01/10/28 23:08
誕生日が判明する問題を聞いた事があるんですが、
どなたかご存知ないでしょうか?

961 名前:892投稿日:01/10/29 00:52
>>960
どーせこの辺じゃないの?単なる数字遊びだと思われ
http://www.google.com/search?q=%92a%90%B6%93%FA%93%96%82%C4&hl=ja&lr=

962 名前:132人目の素数さん投稿日:01/10/29 22:04
ある直線上において、静止している老婆AへトラックBが100m先から
100km/hの速度で接近してきたとする。

老婆AとトラックBは必ず直線上に存在するとした時
AとBが重ならない場合の条件を答えよ。

963 名前:132人目の素数さん投稿日:01/10/29 22:09
老婆Aが時速100qの自動車の中で静止している。

964 名前:132人目の素数さん投稿日:01/10/29 22:11
老婆の股下はトラックが潜るのに十分であった

965 名前:132人目の素数さん投稿日:01/10/29 22:15
重なる=合体?

966 名前:132人目の素数さん投稿日:01/10/29 22:29
老婆がトラックを破壊するに十分な火力を有している

967 名前:132人目の素数さん投稿日:01/10/29 22:32
未来の老婆からターミネーターが送られてきた。

968 名前:132人目の素数さん投稿日:01/10/29 22:50
トンネル効果。婆さんすり抜けた。

969 名前:132人目の素敵さん投稿日:01/10/29 22:51
老婆はリンボーダンスでトラックの下を潜った

970 名前:132人目の素数さん投稿日:01/10/29 22:53
道路は2車線だった

971 名前:132人目の素数さん投稿日:01/10/29 23:13
撥ねられるから重ならない。

972 名前:132人目の素数さん投稿日:01/10/29 23:51
同一直線上ではないのでセーフ

973 名前:132人目の素数さん投稿日:01/10/31 03:58
お答えどもども、っていうかこういうネタの方がレス多いのは何故?(藁

解答「老婆Aが100km/h超過の速度でトラックから離れた」

始めて解答を知った時に何故に?とオモタけどよく考えると
数学ならこういう答えはありなんだよな〜

974 名前:132人目の素数さん投稿日:01/10/31 05:37
>>973
問題をよく読め!
ばあさんは静止と書いてあるぞ!

975 名前:132人目の素敵さん投稿日:01/10/31 07:55
>>974
よく読めって・・・出題者だろ。
多分>>963みたいな事を言いたいんじゃないの?

976 名前:132人目の素数さん投稿日:01/10/31 21:59
>>KARL氏

今日、過去ログ探してたら
・0<f(0)<1,f(n+1)=f(n)*(1-f(n))のときlim(n→∞)nf(n)を求めよ
って問題をあなたが出していたのだが、答えは1か?
証明はまだ厳密でないので書けないが…

977 名前:132人目の素数さん投稿日:01/11/01 03:10
時間がある人へひまつぶし問題。

11235831459437...

この数列の法則を見出して次の数字を答えてください。
そして、この数列は何桁で1巡するでしょうか?

978 名前:132人目の素数さん投稿日:01/11/01 03:12
0707070707・・・
0が出たらおしまい。
フィボナッチ数列の1のケタの表示。

979 名前:132人目の素数さん投稿日:01/11/01 03:18
mod5で周期5*4=20とmod2で周期3で計60

980 名前:KARL ◆gjHKPQSQ 投稿日:01/11/01 03:21
>>976
1で正解。
0<f(0)<1,f(n+1)=f(n)*(1-f(n)*f(n))のとき
lim(n→∞)sqrt(n)*f(n)
も考えてみてください。

981 名前:132人目の素数さん投稿日:01/11/01 03:52
>>980
※x_n,y_nがn→∞でx_n→0,y_n→0となり、
 x_n+1=f(x_n),y_n+1=g(y_n)の時にf'(0)=g'(0)となるなら、
 lim(n→∞)x_n/y_n=1となる。

もし↑のが成り立つのなら
976の場合は
x_n+1=f(x_n)=x_n*(1-x_n)
y_n+1=g(y_n)=y_n/(1+y_n)
980の場合は
x_n+1=f(x_n)=x_n*(1-x_n*x_n)
y_n+1=g(y_n)=y_n/sqrt(1+y_n*y_n)
とすれば両方とも極限値はlim(n→∞)x_n/y_nとなるので1になりますけど…

※を証明するのが難しい。まんまロピタルの定理使うわけにもいきませんしね…
でなおしてきます。

982 名前:132人目の素数さん投稿日:01/11/01 04:40
>>981
0<aとして
x(1)=1
x(n+1)=x(n)/(1+x(n)^(1/a))^a
とすると
x(n)=1/n^a
なので
a=1のときy(n)=x(n),a=2のときz(n)=x(n)とすれば
y(n)/z(n)=(1/n)/(1/n^2)=n
なので正しくない。

983 名前:132人目の素数さん投稿日:01/11/01 12:19
>>978

あちゃ〜残念。07の次は0+7で7です。
その次は7+7で4が来ます。まだまだ続くよ。

父母夏地は合ってるから△をあげやう。

984 名前:132人目の素数さん投稿日:01/11/01 14:02
フィボナッチがらみで二つ。

a_1=1、a_2=1、a_(k+2)=a_(k+1)+a_k (1≦k)でフィボナッチ数列を作る。
1)Σ(k:1->∞)a_k*10^(-k)はどんな循環小数になるでしょう。
2)Σ(k:1->N)a_k*10^(k-1)の下N桁はNが大きくなると循環を始めますが
循環節の長さはいくつでしょう。

うまい方法ないっすかね。

985 名前:132人目の素数さん投稿日:01/11/03 23:41
age

986 名前:リュウネンジャー投稿日:01/11/04 00:15
誰か教えてください!
昔、computer(コンピューター)をある法則に
基づいて数字変換をし、ある法則に基づいて計算すると
答えが666になると聞きました。666は知る人ぞ知る
オーメンらしく、よって、コンピューターは呪われたもの
だと言われました。確か、a=1、b=2、c=3…だったような
気がします。どこの板にいけばいいのかわからなかったので
とりあえず、数学板にきました。よろしくお願いします。

987 名前:132人目の素数さん投稿日:01/11/04 01:02
>>983
978とは別人だけど
112358314594370774156178538190998752796516730336954932572910
で丁度60の周期だね。
a[k+2]≡a[k+1]+a[k]mod(n)で初期値やnを変えてみてるがあまり面白い結果が出ない。
a[k+m]≡(0≦i≦m-1)a[k+i]mod(n)の場合うちのパソコンじゃm=6あたりで悲鳴上げやがる。情けない

988 名前:132人目の素数さん投稿日:01/11/04 01:03
>>986
このような物発見。
N=90でなくN=6の根拠不明ですが。

http://piza.2ch.net/occult/kako/979/979392775.html

9 名前: あなたのうしろに名無しさんが・・・ 投稿日: 2001/01/14(日) 00:19

アルファベットに6をプラスしていくと次のようになる。

A=6 B=12 C=18 D=24 E=36 F=42 G=48
H=54  I=60 J=66 K=72 L=78 M=84 N=6
O=90 P=96 Q=102 R=108 S=114  T=120
U=126 V=132 W=138 X=144 Y=150 Z=156

これを"COMPUTER"にあわせるとその総計は"666"になる!

C=18
O=90
M=84
P=96
U=126
T=120
E=36
R=108
 ̄ ̄ ̄ ̄
  666

つまりコンピューターは世界支配の獣そのものだ!!!

989 名前:132人目の素数さん投稿日:01/11/04 01:31
computerでも出来るんだ。
無理矢理数当てはめて666にするのって昔からあるよね。
法王→神の子の代理人→Vicarius Filii Dei→
5+1+100+1+5+1+50+1+1+500+1=666とか

990 名前:132人目の素敵さん投稿日:01/11/04 03:22
>>988
> このような物発見。
> N=90でなくN=6の根拠不明ですが。

つーか、書き間違ってるだけでしょ。
COMPUTERの各文字を数字に直して足すと111になるからねぇ。

991 名前:リュウネンジャー投稿日:01/11/04 05:37
>>988
>>989
>>990
色々あるんですね・・・。
勉強になりました。
ありがとうございます〜☆

992 名前:キリ番ゲッター投稿日:01/11/04 10:30
1000間近スレッド発見

993 名前:キリ番ゲッター投稿日:01/11/04 10:30
1000狙うからな
覚悟しておけ!!!!!!>ALL

994 名前:キリ番ゲッター投稿日:01/11/04 10:31
俺様が1000取ったら
ウラビデオ
さしあげます!!!!!!!!!!!!!!maji

995 名前:キリ番ゲッター投稿日:01/11/04 10:32
ウラビデオ配布場所はここ
http://www.megabbs.com/cgi-bin/readres.cgi?bo=douga&vi=991411619&rm=100

996 名前:キリ番ゲッター投稿日:01/11/04 10:32
いいか、邪魔したら ウラビデオ配布しないぞ!!!

わかったか この あんぽんたんめ

997 名前:キリ番ゲッター投稿日:01/11/04 10:33
俺様が配布予定のウラビデオのサンプル画像
http://www.romamezor.f2s.com/image/140.jpg

欲しくなりましたか

998 名前:キリ番ゲッター投稿日:01/11/04 10:34
ふっふっふ。。。。
ROM君のPCが
あぼーん
された模様

hehehe 今のうち〜〜〜〜〜〜〜〜!!!!

999 名前:キリ番ゲッター投稿日:01/11/04 10:35
ではでは
1000


だきます    oh yes!!!!!!!!! my god

1000 名前:キリ番ゲッター投稿日:01/11/04 10:35
1000

1001 名前:1001投稿日:Over 1000 Thread
このスレッドは1000を超えました。
もう書けないので、新しいスレッドを立ててくださいです。。。


DAT2HTML 0.26 Converted.